Sunteți pe pagina 1din 87

Anul VI, Nr.

Iulie Decembrie 2004

RECREAII
MATEMATICE
REVIST DE MATEMATIC PENTRU ELEVI I PROFESORI

e i = 1

Editura Crengua Gldu


IAI, 2004

Semnificaia formulei de pe copert:


i
ntr-o form concis, formula e = 1 leag cele patru ramuri fundamentale ale matematicii:
ARITMETICA
GEOMETRIA
ALGEBRA
ANALIZA MATEMATIC

reprezentat
reprezentat
reprezentat
reprezentat

de
de
de
de

i
e

Redacia revistei :
Petru ASAFTEI , Temistocle BRSAN, Dan BRNZEI, Ctlin - Cristian BUDEANU,
Constantin CHIRIL, Eugenia COHAL, Adrian CORDUNEANU, Mihai CRCIUN
(Pacani), Paraschiva GALIA, Paul GEORGESCU, Dumitru GHERMAN (Pacani),
Gheorghe IUREA, Lucian Georges LDUNC, Mircea LUPAN, Dan tefan
MARINESCU (Hunedoara), Gabriel MRANU, Andrei NEDELCU, Gabriel POPA,
Dan POPESCU (Suceava), Florin POPOVICI (Braov), Maria RACU, Ioan ERDEAN
(Ortie), Dan TIBA (Bucureti), Lucian Tuescu (Craiova), Adrian ZANOSCHI.

Responsabili de numr :
Temistocle BRSAN, Gabriel POPA, Paul GEORGESCU, Gheorghe IUREA,
Lucian Georges LDUNC, Mircea LUPAN, Andrei NEDELCU

Adresa redaciei:
Catedra de Matematic Universitatea Tehnic Gh. Asachi Iai
Bd. Carol I, nr.11, 700506, Iai
Tel. 032 213737 / int. 123
E-mail: acord@math.tuiasi.ro

EDITURA CRENGUA GLDU


Toate drepturile rezervate
ISSN 1582 - 1765
Bd. N. Iorga, Bl. K2, ap. 4, IAI
Tel. / Fax: 032 - 230598

Anul VI, Nr. 2

Iulie Decembrie 2004

RECREAII
MATEMATICE
REVIST DE MATEMATIC PENTRU ELEVI I PROFESORI

e i = 1
Apare cu sprijinul
FILIALEI IAI a SOCIETII de TIINE MATEMATICE
i INSPECTORATULUI COLAR al JUDEULUI IAI

IAI, 2004

C
atre cititori
dup
a cinci ani de aparitie a revistei
Fugit irreparabile tempus
Revista "Recreatii matematice", cu doua aparitii pe an (exceptnd primul an, 1999,
n care a fost publicat un singur numar), a intrat n al saselea an al existentei sale.
Numele ei deriv
a din cel al revistei "Recreatii stiintifice", care a ap
arut la Iasi n
perioada 1883-1888 si care este prima publicatie din tara noastr
a destinat
a tineretului studios. Revista "Recreatii stiintifice" a publicat materiale din toate domeniile
stiintei, dar cu precadere articole, note si probleme de matematica. Este o cutezanta
faptul de a ne lega numele de aceast
a veche si prestigioas
a revist
a si este de o mare
r
aspundere ncercarea noastr
a de a o continua prin asumarea obiectivelor acesteia
(v. RecMat - 1/2003, pp. 1-5), ramase actuale si acum, la mai mult de o suta de ani.
Membrii fondatori ai revistei "Recreatii Matematice" sunt cei prezenti n redactia
primului num
ar: Temistocle Brsan, Catalin Calistru, Alexandru Carausu, Constantin Cocea, Adrian Corduneanu si Gheorghe Iurea. Redactia revistei s-a modificat si s-a largit cu timpul; actuala redactie (prezenta pe coperta, interior) este formata din profesori, nvatatori si elevi, care fac din entuziasm si cu pasiune o munca
necompensat
a de vreo r
asplat
a material
a. Nu-l vom uita pe Alin Spuma, exemplu
de abnegatie, pasiune si competenta.
n acesti primi ani de existenta, eforturile redactiei s-au dirijat spre cristalizarea
unei identitati a revistei si dobndirea unui impact favorabil al acesteia cu publicul
interesat de matematica elementar
a. ntr-adev
ar, revista are n prezent o form
a
grafic
a de prezentare definitivat
a si un continut structurat pe un num
ar de rubrici
bine conturat. Pe de alta parte, punctele de distributie a revistei sunt raspndite pe
o arie ntinsa, iar colaboratorii cu note si probleme originale provin din toata tara.
Aceste preocup
ari sunt n leg
atur
a strns
a cu atingerea obiectivelor revistei, care
constituie n fapt ratiunea aparitiei sale.
Revista se adreseaza elevilor de la cei mici, din clasele primare, pna la absolventii liceelor studentilor preocupati de viitoarea lor munca la catedra, profesorilor
si tuturor celor ce ndr
agesc matematica elementar
a.
Prima parte a fiec
arui num
ar ce ocup
a aproape jum
atate din spatiul ei este
destinata articolelor de informare, studiilor si notelor originale, chestiunilor metodice
si din istoria matematicii. A fost publicat n acesti cinci ani un numar de aproximativ
o sut
a de articole de acest fel, care a oferit cititorilor un material variat, pentru toate
nivelurile de preg
atire, n bun
a parte accesibil si elevilor. Credem c
a existenta acestui
spatiu larg de publicare a stimulat posibilitatile creative ale cititorilor nostri.
n scopul cultivarii gustului pentru matematica, au fost publicate un numar de
portrete de matematicieni ilustri (Fermat, Abel, Kolmogorov etc.) sau au fost
prezentate unele probleme celebre (postulatul V al lui Euclid, marea teorema a lui
Fermat, problema celor patru culori, trisectia unghiului etc.). Au fost evocate personalitati remarcabile ale matematicii romnesti (Spiru Haret, Gh. Vr
anceanu etc.),
reviste cu un aport major n cultura stiintific
a a tarii (Recreatii stiintifice, Revista stiintifica "V. Adamachi"), institutii cu un rol important n nv
atamntul si cercetarea
romneasca (Seminarul matematic "Al. Myller", Observatorul astronomic din Iasi ),
85

momente de vrf ale matematicii din tara noastr


a (Al V-leaCongres international al
matematicienilor romni ), ct si multe figuri de matematicieni ieseni, disparuti sau
n viata (I. Creang
a, Gh. Gheorghiev, R. Miron, C. Corduneanu etc) si figuri
de prestigiu ale nv
atamntului liceal (N. Colibaba).
Mention
am n mod special rubrica "Nota elevului", care a devenit permanent
a
ncepnd cu nr. 1/2001 al revistei si care a gazduit deja 11 articole. Elevii cei mai
buni au n aceasta rubrica un spatiu destinat ncercarilor lor pe un anume subiect sau
pe o idee propice. La recomandarea redactiei revistei, Fundatia culturala "Poiana"
(director d-l Dan Tiba) ofer
a anual premii n bani tinerilor autori ai celor mai bune
astfel de note. Pna n prezent au fost premiati un numar de cinci elevi.
Partea a doua cea mai dinamica parte a revistei este destinata concursurilor,
problemelor propuse si solutiilor acestora si se ncheie cu o list
a a rezolvitorilor.
Sunt publicate cu regularitate subiectele date la urm
atoarele concursuri initiate si
organizate de ieseni: Concursul "Al. Myller" (concurs national, cl. VII-XII), Concursul "F. T. Cmpan" (concurs interjudetean, cl. IV-VIII), Concursul "Recreatii
matematice" (concurs n cadrul Taberei nationale de matematic
a, cl. VII-XI), Concursul "A. Haimovici" (concurs interjudetean pentru liceele economice, industriale
si agricole, cl. IX-XII). Tot cu scopul informarii elevilor competitivi, sunt prezentate
si concursuri organizate n alte centre: Concursul "R. Miron" (Vaslui), Concursul
"Unirea" (Focsani), Concursul "O. Onicescu" (Botosani) toate fiind interjudetene.
Pentru elevii din gimnaziu talentati au fost publicate enunturile si solutiile problemelor date la OBM (juniori), ct si problemele aflate n atentia juriului acesteia.
Subiectele date la examenul de admitere n cteva facult
ati din universit
atile
iesene ofer
a o orientare candidatilor care opteaz
a pentru facult
atile respective.
Desigur, n centrul demersului nostru se afl
a rubricile "Probleme propuse" si
"Solutiile problemelor propuse" prin care se urmareste atragerea elevilor pentru
studiul matematicii si o bun
a preg
atire a celor care reusesc s
a devin
a rezolvitori
constanti si pasionati. n acesti cinci ani de aparitie, s-au publicat 73 probleme pentru clasele primare si cte 50 probleme pentru fiecare clas
a de gimnaziu sau de liceu.
Sunt rezolvate toate problemele propuse dupa un an de la publicarea lor, se dau mai
multe solutii (atunci cnd acestea apar) si se mentioneaz
a autorii acestora.
Rubrica "Probleme pentru pregatirea concursurilor", deschis
a n nr. 2/2001 al
revistei si mp
artit
a n dou
a nivele (gimnazial si liceal), pune la dispozitia elevilor cu
nclinatii speciale probleme cu un grad de dificultate sporit.
Revista se ncheie cu "Pagina rezolvitorilor", o list
a a elevilor rezolvitori de probleme menit
a s
a-i ncurajeze si s
a-i ambitioneze. Orice elev mentionat de trei ori n
aceast
a rubric
a primeste din partea redactiei o diplom
a si un premiu n c
arti.
Printre note si probleme au fost presarate si un numar de "Recreatii . . . matematice" pentru a dovedi c
a matematica poate lumina fetele celor pasionati nu numai
cu idei inspirate, ci si cu zmbete.
Dup
a acesti cinci ani de aparitie, desprindem o concluzie: s
a continu
am cu aceeasi
pasiune pentru a ndrepta ceea ce am gresit, a completa ceea ce am facut multumitor
si a tinde spre perfectiune n acele directii n care am reusit s
a facem ceva bun.
Redactia revistei "Recreatii matematice"
86

Alexandru Myller, ctitorul scolii matematice iesene


(3 decembrie 1879 - 4 iulie 1965)
La 3 decembrie a. c. se mplinesc cinci
sferturi de veac de la nasterea lui Alexandru Myller, savant de reputatie internationala si eminent profesor al Universitatii din Iasi.
S-a nascut n Bucuresti, unde a urmat
scoala primar
a, liceul (1896) si Facultatea
de S
tiinte (1900), avnd ca profesori pe
ilustrii matematicieni romni S. Haret, E.
Pangrati, N. Coculescu si D. Emmanuel.
Dup
a un scurt stagiu ca profesor la
Liceul "V. Alecsandri" din Galati, pleac
a,
n 1902, la studii la Gttingen, unde a avut
ca profesori pe celebrii matematicieni Felix
Klein si David Hilbert. Prelund creator
noua teorie a lui Hilbert asupra ecuatiilor
integrale, Myller publica un ciclu de lucrari
printre care si teza de doctorat (1906) elaborat
a sub ndrumarea lui Hilbert.
Lucr
arile lui Myller n domeniul analizei
matematice, elaborate la Gttingen si continuate la Bucuresti, au marcat, prin ideile,
metodele si rezultatele obtinute, un moment important n dezvoltarea si afirmarea
matematicii romnesti pe plan national si international. Acestea, precum si titlul de
elev al lui Hilbert, l-au consacrat ca matematician de prima marime. La Iasi, Myller
si schimba directia cercetarilor matematice, trecnd la geometrie, domeniu care l-a
fascinat nc
a din tinerete.
A obtinut numeroase rezultate n domeniul teoriei ecuatiilor diferentiale si
integrale prin: extinderea unor rezultate ale lui Hilbert la cazul unor ecuatii diferentiale de ordin arbitrar; ecuatii integrale cu nucleu antisimetric; probleme bilocale, la
limita si de periodicitate pentru ecuatii diferentiale ordinare si cu derivate partiale;
utilizarea metodelor functionale n rezolvarea unor probleme de fizica matematica.
n domeniul geometriei diferentiale contributiile sale se refera la: geometrie
algebrica si geometrie riglata; definirea notiunii de concurenta a vectorilor contravarianti ca o generalizare a paralelismului Levi Civita; paralelismul ntr-un sistem
de plane, care a condus la notiunea de configuratie Myller; dezvoltarea, mpreuna cu
O. Mayer a geometriei diferentiale centroafine. Aceste cercetari au fost declansate si
impulsionate de aparitia teoriei relativitati generale, unde se folosea n mod consistent teoria conexiunilor afine si alte notiuni geometrice legate de conexiuni. Aceste
preocupari au marcat momentul intrarii scolii de matematica de la Iasi pe arena
mondiala a cercetarii stiintifice.
87

n domeniul istoriei matematicii a reusit s


a repun
a n valoare contributiile
originale ale unor precursori precum D. Asachi, St.
Botez,
E. Bacaloglu etc.

Numit n 1910 profesor titular la catedra de geometrie analitica a Universitatii din


Iasi, Alexandru Myller pune bazele nvatamntului matematic modern la aceasta
institutie de nvatamnt prin: fondarea vestitei biblioteci de specialitate (18. X. 1910)
ca fundament al cercetarilor originale; ncadrarea unui corp profesoral de mare valoare; atragerea unor tineri cu care creeaza prima scoala romneasca de matematica,
cunoscuta sub numele de Seminarul Matematic din Ia
si; initierea n premiera
la Iasi a unor studii de istoria matematicii romnesti si universale; introducerea
unor cursuri libere de specialitate si a lucrarilor de licenta; aporturi originale n
geometria diferentiala, care au lansat scoala ieseana n competitie internationala.
Aceste evenimente s-au petrecut, prima oar
a n 1922 cnd au fost publicate lucr
arile
legate de paralelismul lui Levi Civita, a doua oar
a, n 1933, cnd a ap
arut memoriul
de geometrie diferentiala centroafina elaborat n colaborare cu O. Mayer.
Ca profesor, Al. Myller a fost un maestru n arta comunicarii cu studentii. A
aplicat, pentru prima oar
a n tara noastr
a, metoda euristic
a n predarea matematicii
la nivel universitar. Lectiile sale erau adev
arate momente de creatie n care profesorul
ghida pe studenti sa descopere, mpreuna, adevarurile stiintei predate. A mpartasit
cu dragoste si generozitate fiec
arei generatii de studenti tot ce a acumulat din punct
de vedere stiintific si metodic.
Sub ndrumarea sa, a fost obtinut n 1920 primul doctorat n matematici pure, la o
universitate romneasca, de catre Octav Mayer, iar n 1925 a fost obtinut doctoratul
n matematici de c
atre Silvia Creanga, care devine prima femeie doctor n matematici
la o universitate romneasc
a.
Dup
a r
azboi, a functionat ca rector n dificila perioad
a 1944-1945, reusind s
a
deschida cursurile universitare n mai 1945, dupa refugiul n Transilvania. A fost
preocupat de reconstructia cl
adirii, de revenirea la Iasi a laboratoarelor si bibliotecilor, precum si a profesorilor si studentilor.
n 1947 iese la pensie, dar r
amne legat de activitatea Seminarului Matematic,
unde continua cercetarile stiintifice, se preocupa de instruirea tinerilor doctoranzi
si de bunul mers al bibliotecii urm
arind cu asiduitate obtinerea unor publicatii mai
greu de procurat.
A mai desf
asurat variate activit
ati si la Institutul de Matematic
a de la Filiala Iasi
a Academiei Romne. A continuat sa lucreze pna la stingerea sa din viata survenita
la 4 iulie 1965.
Pentru meritele sale de exceptie, Academia Romn
a l-a ales membru titular n 1938. n 1959 Universitatea Humbold din Berlin i-a decernat titlul
de Doctor Honoris Causa pentru straduinti deosebite de a creea o matematica
romneasca de sine statatoare. A primit numeroase distinctii si decoratii din partea
statului romn.
La mplinirea a 125 ani de la nasterea savantului Alexandru Myller, elevii si
elevii elevilor sai l omagiaza n semn de adnca pretuire pentru opera sa nchinata
dezvolt
arii stiintei si nv
atamntului modern n tara noastr
a.
Prof. dr. Gheorghe BANTA
S
Prof. dr. Vasile OPROIU
88

Henri Poincar
la 150 de ani de la nasterea sa
La pense nest quun clair,
au milieu dune longue nuit,
mais cest cet clair qui est tout.
H. P.
Este anevoie pentru un om obisnuit
sa compare colosii; adncimi nebanuite tulbur
a vederea detaliilor si ratiunea
nu reuseste s
a completeze o imagine
de nteles. Exista consensul ca Henri
Poincar a fost ultimul matematician
universal si s-a dovedit genial n tot ce
a ntreprins. Acum, la 150 de ani de la
nasterea sa, zabovim un pic gndind la
el.
Henri Poincar s-a n
ascut la
Nancy, pe 29 aprilie 1854, ntr-o distins
a
familie de intelectuali. Ambidextru si
miop, a suferit de o slaba coordonare
muscular
a. Din scoala elementar
a a excelat n compozitii scrise. n 1862 este
nscris la Liceul din Nancy si timp de
11 ani s-a dovedit aici un elev stralucit la toate materiile. Profesorul s
au de
matematic
a l considera un monstru al
matematicii. La concursurile generale ale liceelor din Franta a cstigat premiile nti. n 1873 este admis la cole Polytechnique pe care o absolva n 1875, depasindusi clar colegii n domeniile matematicii. Citea mult si variat, realiznd conexiuni
neasteptate si dovedind o memorie vizual
a excelent
a. si completeaz
a cu mult interes
studiile la cole des Mines si profeseaza ca inginer n timp ce si elaboreaza lucrarea
de doctorat, sub conducerea stiintifica a lui Charles Hermite, n domeniul ecuatiilor
diferentiale. Devine doctor n matematic
a n 1879, convingnd dar si uimind comisia.
Pred
a analiz
a matematic
a 2 ani la Caen, din 1881 primeste catedr
a la Facultatea de S
tiinte din Paris, din 1886 trece la Sorbona pe Catedra de ecuatiile fizicii
matematice si probabilitati, preda si la cole Polytechnique. Lectiile sale acopera
domenii variate, mereu n schimbare, dar nu sunt usor de urm
arit de c
atre studenti
din cauza abundentei de idei. Este ales n Academia de Stiin
te n 1887; caz unic,
este ales n fiecare din cele cinci sectii ; n 1906 devine secretar permanent al acesteia.
n 1908 este ales n Academia Franceza al carei secretar permanent devine n anul
prematurului s
au deces, la 17 iulie 1912.
Activitatea sa de creatie stiintific
a se desf
asura cu precizie: ntre 10 si 12 dimineata,
ntre 5 si 7 dupa amiaza. Dupa ora 7 se informa. Pornea cte o lucrare fara un
89

plan prestabilit, f
ar
a calcule preliminare; dup
a primii pasi urmau natural urm
atorii.
Aborda si abandona subiecte din unghiuri de vedere diferite dar subconstientul sau
continua studiile, ntregind imaginea. Dificultatea de a opri cercetarea l determina
s
a nu ntreprind
a lucr
ari importante dup
a ora 7 seara, spre a nu-si tulbura somnul.
A dezvoltat studiul functiilor automorfe dup
a o idee ce sustine c
a i-a venit f
ar
a
nici o pregatire prealabila n momentul cnd urca ntr-un autobuz; folosea n context
complet diferit transformarile din geometria neeuclidiana. Prin lucrarea Analysis
situs, publicat
a n 1895, ntemeiaza domeniul topologiei algebrice n care clasica sa
conjectur
a este nc
a actual
a. Este considerat fondatorul teoriei functiilor analitice
de mai multe variabile complexe. A adus contributii esentiale n geometria algebrica, dezvoltnd metode ce au permis deduceri directe ale unor rezultate profunde
ce se bazau pe o idee de demonstratie gresit
a. n 1901 a rezolvat o problem
a crucial
a de teoria numerelor: c
autarea punctelor de coordonate rationale pe o curb
a
algebrica f (x, y, z) = 0 cu coeficienti rationali. Geometria hiperbolica, creata de
catre Lobacevski si Bolyai, a devenit de nteles pe modelul de univers imaginat de
catre Poincar. Este considerat al
aturi de Einstein (si Lorentz ) fondator al teoriei
relativitatii.
Retinem atentia cu cteva detalii referitoare la problema celor 3 corpuri. Se
presupun date la un moment initial 3 corpuri date prin trei puncte de mase invariabile
prin pozitia lor, viteze si acceleratii. Se presupune c
a asupra lor nu intervine nici
o forta exterioar
a, dar c
a ele evolueaz
a respectnd legea atractiei universale. Se
cere sa se evalueze comportarea lor n timp. Problema abordata de catre Poincar
n 1889 era ct pe ce s
a fie compromis
a de o eroare comis
a de editorul de la Acta
Mathematica. Un intens schimb de scrisori cu Mittag-Leler a l
amurit chestiunea
si un memoriu al lui Poincar a ap
arut n 1890, fiind considerat act de nastere a
teoriei haosului. Motivul este ca aceasta problema a celor 3 corpuri revine la un
sistem de ecuatii diferentiale (cu necunoscutele cele 9 functii ce dau coordonatele
punctelor) care nu este stabil. Aceasta nseamn
a c
a modific
ari infinitezimale ale
datelor problemei conduc la modific
ari substantiale ale traiectoriilor.
H. Poincar a adus contributii esentiale n numeroase domenii ale matematicilor aplicate: mecanica cereasca, cosmologie, mecanica cuantica, optica, electricitate, hidrodinamica, telegrafie, termodinamica, teoria elasticitatii, electromagnetism,
capilaritate.
n special, modul diferit de a concepe matematica la Poincar si Hilbert a condus
la filosofii distincte ale matematicii. n opozitie cu punctul de vedere logicist si
formalist, H. Poincar a sustinut un punct de vedere intuitionist: prin logica
demonstram, dar prin intuitie creem, logica ramne sterila fara a fi fertilizata de
intuitie.
H. Poincar este un maestru al populariz
arii stiintei; c
artile sale: Stiin
ta si
ipoteza (1902), Valoarea stiintei (1905), Stiin
ta si metoda (1908), Ultimele gnduri
(1908, postum) sunt disponibile si n limba romn
a.
La funeraliile sale s-a spus: ... a fost matematician, geometru, filosof,
scriitor, poet al infinitului, bard al stiintelor.
Prof. dr. Dan BRNZEI
90

Trecerea planetei Venus prin fata Soarelui


Planeta Venus, a doua planeta dupa Mercur ca departare de Soare, cunoscuta si
sub numele de Luceafarul de dimineata sau de seara, trece din cnd n cnd prin fata
Soarelui (se spune c
a n acel moment planeta se afl
a n conjunctie inferioara ). Acest
"din cnd n cnd" reprezint
a un interval de timp de cel putin 100 de ani. Pe data
de 8 iunie a acestui an fenomenul se va desfasura din nou, iar noi contemporanii
fenomenului vom avea prilejul sa-l vedem, daca cerul va fi favorabil observatiilor.
Trecerea lui Venus zeita frumusetii si dragostei la romani, numit
a la greci
Afrodita, ca fiica lui Zeus si Dionei a prezentat o importanta deosebit
a n istoria
astronomiei si de ce nu si n istoria civilizatiei. Omul, aceasta creatie extraordinara a
legilor lumii materiale, prin existenta si natura sa, s-a ntrebat de multa vreme cum a
ap
arut; s-a n
ascut aici pe P
amnt, a venit de undeva, a fost creat de cineva, care este
locul s
au n lumea Universului observabil? Iat
a ntreb
ari, devenite fascinante timp
de secole, la care astronomia, desi a raspuns doar partial, totusi a dat raspunsuri mai
mult sau mai putin convingatoare.
Pozitia P
amntului n Univers poate fi determinat
a dac
a se cunoaste distanta
Pamnt - Soare. Aceast
a distanta, m
asurat
a prin procedee astronomice, este de
149 597 870 km. Ei bine, aceasta distanta a fost ameliorata treptat prin observarea
timp de cteva secole si atunci cnd Venus trecea prin fata discului solar. Acum
exist
a metode ce permit determinarea distantei P
amnt - Soare cu precizie de cteva
zeci de kilometri, ns
a pentru acele vremuri metoda trecerii a reprezentat un succes
remarcabil de la trecere la trecere.
Nici acum observarea fenomenului nu este lipsita de interes daca observatiile se
fac cu suficient
a precizie (precizia n timp n determinarea fenomenului, trebuie s
a
fie de cel putin o sutime de secund
a; important este s
a se cunoasc
a la fel de precis
pozitia locului din care se fac observatiile).
Trecerea planetei Venus prin fata Soarelui are loc o data la 115 ani dupa care
urmeaza o alta trecere dupa 8 ani. Urmatoarea trecere urmnd sa aiba loc dupa 122
de ani, urmata de o alta dupa nca 8 ani. Asa se succed aceste treceri. Dac
a planul
traiectoriei lui Venus ar fi acelasi cu planul orbitei terestre, atunci trecerea lui Venus
ar avea loc dupa fiecare 584 de zile si 22 de ore. Acest interval de timp reprezinta
perioada sinodica a lui Venus (intervalul dintre dou
a pozitii asem
an
atoare ale planetei
n raport cu Soarele si P
amntul).
ntr-o viata de om, trairea acestui eveniment nu poate fi dect de doua ori, o
singura data sau deloc, cu exceptia celor ce au o longevitate mai mare de 115 ani
sau 122 de ani. Penultima trecere vizibila la noi a avut loc n anul 1874, fenomen
urm
arit de profesorul Neculai Culianu, fost decan al Facult
atii de S
tiinte si apoi
rector ntre anii 1888 - 1898 al prestigioasei universitati iesene. Ultima traversare a
putut fi observata dupa 8 ani (1882).
Fenomenul se ncadreaza n categoria eclipselor de Soare cnd Luna, Mercur sau
Venus si Pamntul sunt n linie dreapta. Spre deosebire de eclipsele de Soare, cnd
Luna interpunndu-se ntre Pamnt si Soare poate sa-l acopere complet, trecerea lui
91

Venus nu are aceast


a calitate. Venus fiind mult dep
artat
a de P
amnt (n acel moment
departarea va fi de 43 228 162 km) si mica n raport cu Soarele, va lasa o umbra ca
un mic disc ntunecat reprezentnd a 33-a parte din diametrul Soarelui.
Intrarea peste discul solar (primul contact exterior) va avea loc la ora 8 h 19 m
47 s ca dup
a 19 m si 27 s Venus s
a intre complet peste discul solar la ora 8 h 39 m
14 s (primul contact interior). Urmatoarele etape vor fi: ultimul contact interior la
ora 14 h 03 m 39 s, iar ultimul contact exterior va avea loc la ora 14 h 22 m 49 s.
Momentele caracteristice mentionate mai sus au fost calculate de c
atre Institutul
de Mecanica Cereasca si Calculul Efemeridelor din Franta, pentru Bucuresti. Pe
teritoriul tarii noastre aceste momente vor diferi de la localitate la localitate, cu nu
mai mult de cteva secunde.
NASA a f
acut calcule si pentru Iasi si iat
a momentele caracteristice: 8 h 19 m
34 s primul contact exterior, 8 h 39 m 11 s primul contact interior, 14 h 03 m
15 s ultimul contact interior si 14 h 22 m 37 s ultimul contact exterior.
nceputul intrarii lui Venus va avea loc prin partea stnga (partea estica) din
vecin
atatea marginii inferioare a discului solar asa cum se poate vedea n desenul al
aturat. Tot n desen se arat
a traiectoria umbrei si momentele caracteristice mentionate
mai sus. Durata fenomenului, de la primul contact interior la ultimul contact interior
92

va fi de 5 h 24 m 25 s.
Pentru observarea fenomenului trebuiesc luate masuri de protectie a ochilor. Recomandam ca atunci cnd se ndreapta privirea spre Soare sa se foloseasca o sticla
afumat
a sau o sticl
a ce se utilizeaz
a la ochelarii pentru sudur
a electric
a. Cel mai la
ndemn
a ar fi folosirea foliei metalizate utilizat
a drept ambalaj de c
atre vnz
atoarele
de flori. Aceasta ultima protectie trebuie confectionata din cel putin doua sau trei
folii suprapuse, functie de sensibilitatea ochilor celor ce vor sa observe fenomenul.
Observatii se pot face si cu o lunet
a dotat
a cu un ocular prin care s
a se proiecteze
Soarele pe un ecran situat la o distanta convenabil
a. Imaginea Soarelui ar putea
ajunge la dimensiunile unui cerc cu raza 10 - 15 cm si chiar mai mult. Evident, luneta
trebuie instalata pe un trepied pentru a-i asigura stablitate si urmarire comoda a
Soarelui. O urm
arire asem
an
atoare a fenomenului poate fi f
acut
a proiectnd imaginea
Soarelui ca mai sus, folosind un binoclu.
Amatorii care vor sa aduca contributii la observarea trecerii, avnd n vedere ca
unele observatoare specializate pot sa nu aiba vreme favorabila observatiilor, urmeaza
s
a tin
a seama de urm
atoarele: 1) s
a posede un ceas cu cronometru si pus la or
a dup
a
un post de radio national, 2) s
a urm
areasc
a momentul primului contact exterior si
sa-l cronometreze si la fel sa procedeze pentru celelalte trei momente caracteristice
(primul contact interior, al doilea contact interior si al doilea contact exterior). n
plus fata de aceste recomand
ari, trebuie s
a apeleze la o cunostinta ce posed
a un
aparat numit GPS cu ajutorul c
aruia s
a se determine coordonatele locului unde a
facut observatiile (longitudinea si latitudinea geografica). Daca masuratorile vor fi
f
acute cu o eroare n timp de plus sau minus 0,1 secunde, iar coordonatele locului vor
fi determinate cu o eroare de maximum 8 metri n jurul punctului unde s-au f
acut
m
asur
atorile, atunci observatiile pot fi luate n consideratie.
Observatiile pot fi transmise n scris la Observatorul Astronomic din Iasi, Aleea
Sadoveanu nr. 5, textul urmnd s
a contin
a: mijloacele cu care s-au f
acut observatiile,
momentele caracteristice m
asurate si coordonatele locului.
Operatia, pe plan mondial, este coordonat
a de c
atre Observatorul European de
Sud cu sediul n Garching (Germania) iar calculele centralizate vor fi facute la Institutul de Mecanica Cereasca si Calculul Efemeridelor cu sediul la Paris.
Urm
arirea fenomenului nu este lipsit
a de interes dac
a avem n vedere raritatea
lui, importanta stiintific
a, ncrederea si seriozitatea ce se poate acorda calculelor
astronomice, spre deosebire de ideile avansate de prezicatori si astrologi, care dau
interpret
ari apocaliptice unor astfel de fenomene. Din anul 3000 . H. si pn
a n
prezent, au avut loc 64 de treceri si alte evenimente astronomice deosebite, dar nu s-a
constatat, n afara unor fenomene naturale izolate, uneori devastatoare (cutremure,
inundatii, razboaie provocate de om etc.), sa se fi produs acel eveniment apocaliptic
mult proferat de prezic
atori.

Iulian BREAHNA
Membru al Uniunii Astronomice Internationale,
ex-director al Observatorului Astronomic din Iasi

93

Cteva noi aplicatii ale unei idei consacrate


Gabriel DOSPINESCU 1
Nu credem c
a exager
am spunnd c
a 99% din inegalit
atile care sunt propuse ca
probleme de concurs pot fi demonstrate direct, aplicnd alte inegalit
ati deja consacrate. Exist
a ns
a o categorie aparte de probleme foarte greu de rezolvat n acest
mod. Aceste probleme au solutii frumoase, bazate pe rationamente indirecte, dar
extrem de eficiente.
Vom prezenta n continuare ideea comuna aflata n spatele tuturor acestor solutii.
S
a presupunem c
a avem de demonstrat o inegalitate de forma g (x1 ) + g (x2 ) +
+ + g (xn ) 1 n ipoteza c
a variabilele x1 , x2 , . . . , xn verific
a o relatie de tipul
f (x1 ) + f (x2 ) + + f (xn ) = 1.
Presupunem prin reducere la absurd ca g (x1 ) + g (x2 ) + + g (xn ) < 1. Notnd
1
a S =
S = g (x1 ) + g (x2 ) + + g (xn ), obtinem c
pentru un anumit k > 1.
k
Punem kg(xi ) = ai , i = 1, n, rezolvam ecuatiile n xi si introducem aceste valori
asim c
a variabilele ai ,
ca functii de ai n relatia f (x1 ) + f (x2 ) + + f (xn ) = 1. G
i = 1, n satisfac de asemenea o relatie de forma h(x1 ) + h(x2 ) + + h(xn ) = 0
precum si a1 + a2 + + an = 1. Problema se reduce acum la a dovedi ca egalitatea
h(x1 ) + h(x2 ) + + h(xn ) = 0 este imposibila, lucru care se realizeaza deseori
demonstrnd c
a h(x1 ) + h(x2 ) + + h(xn ) > 0 (sau < 0) pentru orice numere a1 ,
a2 , . . . , an cu sum
a 1.
Credem ca noua problema este adesea mai usoara dect cea initiala si pentru a
exemplifica acest lucru vom rezolva cteva probleme de concurs utiliznd strategia
de mai sus.
Prima dintre ele, Problema 3, OM China 2003, este faimoasa datorita dificultatii
sale, fiind de asemenea nrudit
a cu o alt
a binecunoscut
a problem
a care va fi discutat
a
n cele ce urmeaz
a, Problema 2, OIM 2001.
Exemplul 1. Fie x1 , x2 , . . . , xn (0, /2) astfel ca tg x1 tg x2 . . . tg xn = 2n/2 .
Determinati cel mai mic kn pentru care inegalitatea cos x1 +cos x2 + +cos xn kn
este ntotdeauna adevarata.
Solutie. Substituind tg2 xi = 2ai , problema revine la a determina supremumul
1
1
1
+
+ +
pentru a1 , a2 , . . . , an > 0 si cu
expresiei
1 + 2a1
1 + 2a2
1 + 2an
produsul egal cu 1.
Nu vom discuta cazul n = 1, acesta
fiind trivial. Pentru n = 2, ramne de gasit
1
x
+
, x > 0. Studiind ceea ce se ntmpla pentru
valoarea maxima a lui
1 + 2x
x+2
2
x = 1, x si x 0, deducem c
a valoarea c
autat
a este . Pentru a dovedi acest
3

1
x
lucru, este suficient s
a se studieze monotonia functiei f (x) =
+
,
1 + 2x
x+2
1

Student, Facultatea de Matematic


a si Informatic
a, Bucuresti

94

x > 0, cu ajutorul derivatei.


Sa studiem ce se ntmpla pentru n > 2. Daca toti ai , i = 1, n, sunt egali cu 1,
n
valoarea expresiei de mai sus este . Acum, dac
a ncerc
am s
a "apropiem" ct mai
3
multi ai de 0, observam ca putem "apropia" de 0 cel mult n 1 dintre ei, caz n care
n
valoarea expresiei va tinde la n 1. Deoarece n 1 > , este clar c
a va trebui s
a
3
1
1
1
dovedim ca
+
+ +
n 1.
1 + 2a1
1 + 2a2
1 + 2an
Este usor de v
azut c
a rationnd ca n problema precedent
a ajungem repede ntrun impas. Observ
am ns
a c
a este suficient s
a demonstr
am inegalitatea de mai sus
doar pentru n = 3. De ce acest lucru? Daca n > 3 si a1 a2 an = 1, atunci putem
alege ai , aj , ak al caror produs este cel putin 1. Atunci
1
1
1
1
1
1

+
+ +
< n3+
+p
+
.
1 + 2a1
1 + 2a2
1 + 2an
1 + 2ai
1 + 2ak
1 + 2aj

Dar

1
1
1
1
1
1

+p
+

+p
+q
2,
1 + 2ai
1 + 2ak
1 + 2ai
1 + 2aj
1 + 2aj
1 + 2 ai1aj

presupunnd c
a inegalitatea este adev
arat
a pentru n = 3, iar din relatia precedent
a
rezult
a c
a ea este adev
arat
a si pentru n oarecare.
Sa dovedim deci inegalitatea pentru n = 3. Presupunem ca ea nu este adevarata
1
1
1
2
+
+
= , cu p < 1.
pentru o tripleta a1 , a2 , a3 . Atunci
p
1 + 2a1 1 + 2a2 1 + 2a
3
3
3
Q
Q
p
1
1
p2
1
, obtinem c
a1=

<
. Este
Notnd xi =
2
2
2
2
2 1 + 2ai
k=1 8xk
k=1 8xk
1
deci suficient sa demonstram ca pentru orice 0 < x, y, z < cu x + y + z = 1 avem
2

1
1
1
1
1
1

1, inegalitate echivalenta cu
ca
8x2
2
8y 2 2
8z 2 2
(1 2x)(1 2y)(1 2z)(1 + 2x)(1 + 2y)(1 + 2z) 83 x2 y 2 z 2 .

O alta schimbare de variabila este acum necesara. Desigur, aceasta este 12x = a,
1 2y = b, 1 2z = c. Ramne deci sa demonstram ca, pentru orice a, b, c > 0 cu
suma 1, avem
8(1 a)2 (1 b)2 (1 c)2 abc(2 a)(2 b)(2 c).
Avem ns
a c
a

2
8
8(1 a)2 (1 b)2 (1 c)2 = 8(b + c)2 (c + a)2 (a + b)2 8 (a + b + c)(ab + bc + ca) =
9
512
512
512
=
(ab + bc + ca)2
abc (a + b + c) =
abc.
81
27
27
512
, ceea ce este trivial,
Este suficient, deci, sa demonstram ca (2 a)(2 b)(2 c) <
27
deoarece (2 a)(2 b)(2 c) < 8, iar problema este acum rezolvata.
95

A fost mentionat
a anterior, n trecere, frumoasa Problema 2, OIM 2001, propus
a
de Hojoo Lee. Problema n cauza fiind att de mult discutata si popularizata,
s-ar putea crede ca nu mai este nimic nou de spus despre ea. Totusi, credem ca
urm
atoarea demonstratie a unei generaliz
ari a problemei este nou
a.
Exemplul 2. Demonstrati ca daca a1 , a2 , . . . , an sunt numere reale strict pozitive astfel nct a1 a2 an = 1, iar k > 1 este un numar natural, atunci
1
1
1
p
+ p
+ + p
1.
k
k
k
k
k
1 + (n 1) a1
1 + (n 1) a2
1 + (nk 1) an

Vasile Crtoaje
Solutie. Presupunem c
a
1
1
1
1
p
+ p
+ + p
= ,
k
k
k
k
k
k
p
1 + (n 1) a1
1 + (n 1) a2
1 + (n 1) an
p
pentru un anumit p > 1. Notam p
= xi , i = 1, n si obtinem ca
k
1 + (nk 1) ai

n
n
k
n
Q
Q
pk
1
n 1 =

1
>
1 , iar x1 + x2 + + xn = 1.
k
k
i=1 xi
i=1 xi

n
Q
1
Pentru a ajunge la o contradictie, vom dovedi ca

1
nk 1
k
i=1 xi
pentru orice x1 , x2 , . . . , xn > 0 astfel nct x1 +x2 + +xn = 1. Pentru demonstrarea
acestei inegalitati, sa observam ca
n
n
Q
Q

(x1 + + xi1 + xi+1 + + xn )


1 + xi + + xk1
n
i
Y
1
i=1
i=1

1
=
(1)
xki
(x1 x2 . . . xn )k
i=1

si, din inegalitatea mediilor,


n
Y
(x1 + x2 + + xi1 + xi+1 + + xn ) (n 1)n x1 x2 . . . xn .

(2)

Sumnd inegalitatile de mai sus, obtinem


n
Y


n
1 + xj + + xk1
1 + G + G2 + + Gk1 ,
j

(3)

i=1

Desigur, pentru minorarea celuilalt produs din formula (1), aplicarea direct
a a inegalit
atii mediilor nu mai este la fel de eficient
a. Conform inegalit
atii mediilor, avem
q
n
n
X
n (x1 x2 . . . xn )i
xij
qQ

, i = 0, k 1.
n
k1
n
1 + xj + + xk1
j
j=1
j=1 1 + xj + + xj

j=1

unde G =

n
x1 x2 . . . xn . Din (1), (2) si (3) deducem c
a
n

n
Y
(n 1)n Gn 1 + G + G2 + + Gk1
1
1
,
Gnk
xki
i=1
96

iar pentru a finaliza solutia problemei este suficient s


a demonstr
am c
a
1
1 + G + G2 + + Gk1
1 + n + + nk1 , ceea ce este trivial, deoarece G .
Gk1
n
Este acum momentul s
a discut
am o alt
a problem
a deosebit
a, propus
a la Concursul
anual al Gazetei Matematice de catre Vasile Crtoaje. Solutia autorului se bazeaza
pe aplicarea succesiva a ctorva identitati, fiind aproape imposibil de gasit n mod
independent. Credem c
a solutia ce urmeaz
a este mai natural
a din acest punct de
vedere.
Exemplul 3. Demonstrati ca pentru orice a, b, c, d > 0 astfel ca a2+b2+c2+d2 = 1
este satisfacuta inegalitatea (1 a)(1 b)(1 c)(1 d) abcd.
(1 a) (1 b) (1 c) (1 d)
Solutie. S
a presupunem c
a
= p4 , cu p < 1. Fie
abcd
P
P
1
1b
1c
1d
1
1a
>
,
= x,
= y,
= z,
= t. Atunci 1 =
2
pa
pb
pc
pd
(1 + px)
(1 + x)2
iar xyzt = 1. Problema se reduce, deci, la a demonstra ca
1
1
1
1
+
+
+
1
(1 + x)2 (1 + y)2
(1 + z)2
(1 + t)2
pentru orice x, y, z, t > 0 verificnd xyzt = 1.
Profitnd de faptul c
a enuntul problemei utilizeaz
a 4 numere, vom separa numerele n 2 grupe si demonstra c
a
1
1
1
1
1
1
si
2 +
2 1 + xy
2 +
2 1 + zt .
(1 + x)
(1 + y)
(1 + z)
(1 + t)
Prima inegalitate se reduce la xy (x y)2 + (1 xy)2 0, care este n mod
evident adevarata, pentru a doua rationndu-se n mod analog. Prin sumarea celor
dou
a inegalit
ati si tinnd seama de faptul c
a xyzt = 1, rezult
a concluzia.
n ncheierea acestui articol vom discuta alte doua probleme deosebite, carora li se
pot da solutii rapide utiliznd ideile de mai sus. Prima dintre ele a fost publicata n
revista "American Mathematical Monthly", fiind propus
a de c
atre Vasile Crtoaje.
1
1
Exemplul 4. Fie x1 , x2 , . . . , xn > 0 astfel nct x1 + x2 + + xn =
+ +
x1 x2
1
1
1
1
. Demonstrati ca
+
+ +
1.
+ +
xn
n 1 + x1
n 1 + x2
n 1 + xn
1
1
1
1
Solutie. Presupunem ca
+
+ +
= ,
n 1 + x1
n 1 + x2
n 1 + xn
p
p
1
, i = 1, n si deducem c
a ai <
p < 1. Not
am ai =
, i = 1, n, iar
n 1 + xi
n1
a1 + a2 + + an = 1. Conditia din enunt se transcrie sub forma
X
n
n
X
p
1
n+1 =
.
p
ak

n+1
ak
k=1

k=1

Deoarece p < 1, putem scrie


X
X
n
n
n
X
1
p
n+1 >
n+1 =
ak
ak
k=1

p
k=1 ak

k=1

97

X
1
>
n+1

1
k=1 ak

1
.
n+1

R
amne, deci, s
a demonstr
a pentru
oricena1 , a2 , . . . , an > 0 cu a1 +a2 + +an = 1
am c
n
P
P
1
1
are loc inegalitatea
n+1
, care este echivalent
a cu
1
k=1 ak
k=1 ak n + 1
n
n 1 (n 1) a
P
P
ak
k

.
1

(n

1)
a
a
k
k
k=1
k=1
O alta substitutie este acum necesara, anume 1 (n 1)ak = bk , k = 1, n. Se
observ
a c
a avem de asemenea b1 + b2 + + bn = 1 si r
amne deci s
a dovedim
n 1b
n
P
bk
k
2 P
c
a
(n 1)
, rezultat care se poate obtine prin sumarea a n
bk
k=1
k=1 1 bk
P bk
bk
1
bk
=P

.
inegalit
ati de forma
2
1 bk
b
(n 1) i6=k bi
i6=k i

n final vom discuta o alta frumoasa problema de concurs, propusa la Barajul de


selectie a lotului Romniei pentru OIM, 1999, de c
atre Gheorghe Eckstein.
Exemplul 5. Demonstrati ca daca x1 , x2 , . . . , xn > 0 satisfac x1 x2 . . . xn = 1,
1
1
1
+
+ +
1.
atunci
n 1 + x1 n 1 + x2
n 1 + xn
Solutie. Ca mai sus, problema se reduce la a demonstra ca pentru
2, . . . ,
orice x1 , x
n
Q
1
1
xn <
n + 1 1.
astfel nct x1 +x2 + +xn = 1 are loc inegalitatea
n1
i=1 xi
Cu substitutia bi = 1 (n 1) xi problema de demonstrat se reduce la
n

(1 b1 ) (1 b2 ) . . . (1 bn ) (n 1) b1 b2 . . . bn .

Conform inegalitatii mediilor,


n
n X
n h
Y
Y
Y
(1 bi ) =
bj
(n 1)
i=1

i=1

j6=i

i=1

sY

n1

j6=i

i
n
bj = (n 1) b1 b2 . . . bn ,

ceea ce trebuia demonstrat.


n ncheiere, tin s
a multumesc prof. Marian Tetiva pentru lectura acestui articol
si observatiile pretioase facute, care au contribuit la mbunatatirea formei finale.

ERRATA
n articolul "Combinatorica . . . algebrica" de Gabriel Dospinescu, publicat n
nr. 2/2003 al revistei, forma corecta a Lemei de la pag. 19 este:
2
2
Fie n numar natural prim si = cos
+ i sin . Are loc egalitatea
n
n
a0 + a1 + + an1 n1 = 0, a0 , a1 , . . . , an1 Q,

daca si numai daca a0 = a1 = = an1 .


Diferentele ce apar ne-au fost semnalate de prof. Sergiu Romascu, Vaslui; acestea nu afecteaz
a cu nimic restul articolului. De aceast
a neglijenta se face vinovat
a
redactia si nu autorul articolului.
98

Cteva propriet
ati ale medianelor
Temistocle BRSAN 1
1. Ne propunem s
a indic
am un num
ar de propriet
ati ale medianelor la nivelul
de ntelegere al unui elev bun de gimnaziu. Instrumentele principale de lucru vor fi
teorema lui Thales si teorema lui Menelaus.
Pentru o exprimare scurt
a, vom folosi notiunile de puncte izotomice si puncte
conjugate armonic. Fie A, B, X, Y patru puncte coliniare. Punctele X si Y sunt
izotomice n raport cu A si B daca sunt simetrice fata de mijlocul segmentului [AB]
(evident, X si Y sunt fie interioare, fie exterioare acestui segment); aceasta conditie
revine la egalitatea AX = BY . Punctele X si Y sunt conjugate armonic n raport
XA
YA
cu A si B daca mpart segmentul [AB] n rapoarte egale:
=
(evident, X
XB
YB
este interior segmentului si Y exterior sau invers).
Propozitia 1. Fie ABC un triunghi oarecare, A0 , B 0 , C 0 mijloacele laturilor
[BC], [CA], respectiv [AB] si M, N BC doua puncte izotomice n raport cu vrfurile B si C, cu M diferit de mijlocul segmentului [A0 C]. Atunci, sunt adevarate
afirmatiile:
1 dreptele B 0 M si C 0 N se intersecteaza ntr-un punct X AA0 ;
2 paralela prin M la AC si paralela prin N la AB se intersecteaza ntr-un punct
0
X AA0 ;
3 punctele X si X 0 sunt conjugate armonic n raport cu A si A0 .
Demonstratie. Avem patru situatii distincte ilustrate n figurile de mai jos;
demonstratia este nsa aceeasi.
X
A

A
C X

C
C M

M
N B

A
B

A M

Deoarece M nu-i mijlocul lui [A0 C] rezulta ca N nu-i mijlocul lui [A0 B] si,
ca urmare, dreptele B 0 M si C 0 N intersecteaz
a AA0 . Fie {X} = AA0 B 0 M si
{X1 } = AA0 C 0 N . Conform teoremei lui Menelaus, aplicat
a la 4AA0 C si transversala B 0 M si la 4AA0 B si C 0 N , avem
X1 A N A0 C 0 B
XA M A0 B 0 C

=
1
s
i

= 1,
XA0 M C B 0 A
X1 A0 N B C 0 A
1

Prof. dr., Catedra de matematic


a, Univ. Tehnic
a "Gh. Asachi", Iasi

99

XA
MC
NB
X1 A
si
=
=
. Punctele M , N fiind izotomice n raport cu
0
0
0
XA
MA
X1 A
N A0
X1 A
XA
B si C, avem M A0 = N A0 si M C = N B. Rezult
=
, deci punctul
a c
a
XA0
X1 A0

X1 coincide cu X. Afirmatia 1 este dovedita.


Fie X 0 , X10 intersectiile dreptei AA0 cu paralela prin M la AC, respectiv paralela
MC
X 0A
si
=
prin N la AB. Conform teoremei lui Thales, au loc relatiile:
0
0
XA
M A0
0
NB
X1 A
=
. Din acestea si din izotomia punctelor M si N fata de B si C,
X10 A0
N A0
deducem egalitatea rapoartelor din membrii din stnga. Ca urmare, X10 coincide cu
X 0 , deci 2 este demonstrata.
MC
MC
X 0A
XA
si 0 0 =
=
, deci X si X 0 mpart [AA0 ]
Mai sus s-a aratat ca
0
0
XA 0
MA
XA
M A0
n acelasi raport, adic
a X, X sunt conjugate armonic fata de A, A0 . Afirmatia 3 ,
deci si propozitia, este demonstrat
a.
de unde

Observatie. Sa urmarim deplasarea punctului X pe AA0 , atunci cnd M parcurge BC. Not
am cu G centrul de greutate al triunghiului ABC si cu A mijlocul
0
medianei [AA ]. Dac
a M este n C, atunci X coincide cu A. Dac
a M se ndep
arteaz
a
de C, la "dreapta" acestuia, atunci X (AA ). Daca M se apropie de B, din
"stnga" acestuia, atunci X (A G). Pentru M situat n B, X coincide cu G. Daca
M (BA0 ], atunci X (GA0 ]; punctul M n pozitia A0 coincide cu X. Pentru M
ntre A0 si mijlocul segmentului [A0 C], X parcurge semidreapta de origine A0 ce nu
contine A. n sfrsit, daca M este ntre mijlocul lui [A0 C] si C, atunci punctul X
este situat pe semidreapta de origine A ce nu contine A0 si se apropie de vrful A.
O pozitie particular
a interesant
a a punctului M este semnalat
a n urm
atorul
Corolar. Daca sunt ndeplinite conditiile din Propozitia 1 si n plus M C =
a
= N B = , atunci X este izotomicul punctului G n raport cu A si A0 , iar X 0 este
2
simetricul lui A0 fata de A.
XA
MC
a/2
1
=
=
Demonstratie. Avem X [AA0 ] si
= , adica
XA0
M A0
a/2 + a/2
2
1
1
a XA = GA0 si
XA0 = 2XA sau XA = AA0 . Cum avem si GA0 = AA0 , rezult
3
3
0
XA
XA
1
= ,
prima afirmatie este dovedita. Pe de alta parte, X 0 [AA0 ] si 0 0 =
0
XA
XA
2
deci 2X 0 A = X 0 A0 , de unde X 0 A = AA0 . Asadar X 0 si A0 sunt simetrice fata de A.
2. Aplicatii. Fie D, Da , Db , Dc punctele de tangenta a cercurilor nscris, Aexnscris, B-exnscris respectiv C-exnscris triunghiului ABC cu dreapta BC. Amintim c
a au loc egalit
atile [1], p.30:
DB = Da C = p b si Db C = Dc B = p a (2p = a + b + c);

a
primele spun c
a punctele D si Da sunt izotomice fata de B si C, iar ultimele c
aceast
a proprietate o au si punctele Db si Dc . Putem presupune c
a AB 6= AC
pentru a evita cazul trivial n care 4ABC ar fi isoscel cu vrful A.
100

Propozitia 2. Relativ la punctele Db , Dc , sunt adevarate afirmatiile:


1 B 0 Db si C 0 Dc se intersecteaza ntr-un punct X (AA );
2 B 0 Dc si C 0 Db se intersecteaza ntr-un punct Y (A G);
YA
XA
+
= 2;
3
0
XA
Y
A0

4 X este izotomicul lui G n raport cu A si A0 daca si numai daca 2a = b + c.


Demonstratie. 1 si 2 decurg direct din Propozitia 1 si observatia de mai sus.
De asemenea, avem
YA
Db C
Dc C
pa
p
2p a
XA
+
=
+
=
+
=
= 2,
XA0
Y A0
Db A0
Dc A0
(p a) + a/2 p a/2
p a/2

a
adic
a are loc 3 . n sfrsit, avnd n vedere Corolarul, X este izotomicul lui G dac
a
a
si numai daca Db C = , adica p a = sau 2a = b + c.
2
2
Observatie. Triunghiurile ce satisfac conditia
A
2a = b+c (o latur
a este media aritmetic
a a celorlaltor
doua) sunt speciale, cu multe proprietati cunoscute
X
(n [2], p. 242, sunt date opt propriet
ati). Afirmatia
B
C
Y

4 indic
a o nou
a proprietate caracteristic
a lor.
U
Un rezultat similar se obtine dac
a lu
am punctele
Dc B
D A Da C Db
izotomice D, Da n locul punctelor Db , Dc .
Propozitia 3. Daca 4ABC satisface conditia
a
|b c| 6= , atunci relativ la punctele D, Da avem:
2
1 B 0 D si C 0 Da se intersecteaza n U AA0 ;
V
2 B 0 Da si C 0 D se intersecteaza n V AA0 ;
U
A
V
A
3

= 2 (+ n cazul b > c si n cazul b < c).


U A0
V A0
Omitem demonstratia, ce urmeaza pe cea din Propozitia 2, dar mentionam ca
a
a fie mijlocul segmentului [A0 C].
prin conditia |b c| 6= se evit
a ca D sau Da s
2
Observatie. Punctele X 0 , Y 0 , U 0 , V 0 conjugatele armonic ale punctelor X, Y ,
U , V n raport cu A si A0 sunt alte patru puncte pe dreapta AA0 care pot fi puse n
conexiune cu punctele de tangenta D, Da , Db , Dc , asa cum se indica n Propozitia 1.
R
amne n seama cititorului examinarea lor.
Bibliografie
1. T. Lalescu - Geometria triunghiului, Ed. Tineretului, Bucuresti, 1958.
2. V. Gh. Vod
a - Vraja geometriei demodate, Ed. Albatros, Bucuresti, 1983.

101

O constructie geometric
a a mediilor (II)
Claudiu - Stefan
POPA1

n [2] am prezentat o constructie geometric


a a mediilor armonic
a, geometric
a,
aritmetica, patratica si ponderata a lungimilor bazelor unui trapez, ca segmente cu
capetele pe laturile neparalele ale trapezului si paralele cu bazele lui. Ne propunem
n continuare dezvoltarea acestor idei, fapt ce va conduce la o serie de consideratii
cu interesante aplicatii geometrice.
Propozitie. Fie ABCD un trapez cu AB k CD si punctele A0 , B 0 , C 0 , D0 astfel
nct A (A0 B), B (AB 0 ), C (C 0 D), D (CD0 ), iar AA0 = BB 0 = CC 0 = DD0 .
Notam {E} = A0 C BD0 , {F } = AC 0 B 0 D, {K} = EF AD, {L} = EF BC.
n aceste conditii, EF k AB, EF = AA0 , iar KE = F L.

Demonstratie. Deoarece AA0 k CC 0 si


C
D D
C
a c
a AC 0 CA0 este paraleloAA0 = CC 0 , rezult
gram, deci AC 0 k A0 C, AC 0 = A0 C. Analog,
K
BD0 k B 0 D, AD k A0 D0 si BC k B 0 C 0 , cu egaliK
L
E
F
t
atile de segmente corespunz
atoare. Unghiurile
0 A0 E
\ au laturile respectiv paralele si vor
\
si DAF
D
0 D 0 E ADF
\. Avem nc
fi congruente; la fel, A\
a
0 0
A D = AD, prin urmare 4A0 D0 E 4ADF , A A
B
B
deci A0 E = AF . nsa AE 0 k AF si atunci AF EA0
este paralelogram. Rezulta ca EF k AA0 si EF = AA0 , primele doua afirmatii ale
concluziei.
Fie {K 0 } = EF A0 D0 . Deoarece A0 C si BD0 sunt diagonale n trapezul A0 BCD0 ,
iar K 0 L este paralela la baze prin punctul de intersectie a diagonalelor trapezului,
urmeaza ca EL = EK 0 . nsa KK 0 = AA0 + EF , deci EL EF = EK 0 KK 0 , adica
F L = KE, ceea ce ncheie demonstratia.
Observatie. Concluzia se p
astreaz
a, cu demonstratie asem
an
atoare, n cazul n
care punctele A0 , B 0 , C 0 , D0 se afl
a pe semidreptele [AB, [BA, [CD, respectiv [DC.
n cele ce urmeaz
a vom folosi notatiile: AB = a, CD = b, AA0 = x. Studiem problema variatiei lungimii segmentului [KL] functie de x; vom gndi lungimea segmentului [AA0 ] ca fiind pozitiv
a n cazul n care A (A0 B) si negativ
a pentru A0 (AB.

2ab
Pentru x = 0, avem KL =
(= mh ), iar pentru x = ab (= mg ), avem
a+b
KL = mg , dupa cum s-a demonstrat n [2]. Desennd figurile pentru cteva valori
ale lui x > 0, observam ca segmentul [KL] "coboara" pe masura ce x creste, fara a
"atinge" ns
a linia mijlocie a trapezului. n momentul n care vom demonstra riguros
acest lucru, vom avea o (probabil) nou
a demonstratie pentru inegalitatea mediilor a
doua numere reale pozitive.
n trapezul A0 BCD0 , segmentul [K 0 L] are ca lungime media armonica a bazelor
2 (a + x) (b + x)
.
A0 B = a + x si CD0 = b + x, deci K 0 L =
a + b + 2x
1

Profesor, S
coala "Alecu Russo", Iasi

102

Atunci, avem
ma x + m2g
2(a + x)(b + x)
x(a + b) + 2ab
x + mh
= ma
.
x =
=
a + b + ax
2x + (a + b)
x + ma
x + ma
Suntem astfel condusi la studiul functiei
x + mh
y
.
f : R\ {ma } R, f (x) = ma
x + ma
Aceasta este o functie omografica, strict
ma
crescatoare pe (, ma ) si pe (ma , +),
mg
al c
arei grafic este o hiperbol
a de asimptote
mg
mh
ma
y = ma si x = ma . Prin calcul, stabilim c
a
mh O m
valorile x = mg sunt puncte fixe ale functiei.
x
g
Deoarece f ([0, )) = [mh , ma ), iar restrictia
mg
lui f la [0, ) este strict cresc
atoare, justificarea afirmatiilor anterioare este complet
a.
Consideratiile precedente conduc la urm
atoarele interpretari geometrice:
1. Faptul ca f (mh ) = 0 arata ca, daca ABCD este trapez cu AB k CD,
A0 [AB, D0 [DC sunt astfel nct AA0 = DD0 = mh , iar {E} = A0 C BD0 ,
atunci paralela prin E la bazele trapezului dat trece prin punctul de concurenta al
prelungirilor laturilor neparalele ale acestuia.
2. Faptul ca f (mg ) = mg se interpreteaza geometric astfel: daca ABCD este
un trapez cu AB k CD, iar A0 [AB, D0 [DC sunt astfel nct AA0 = DD0 = mg ,
atunci A0 C, BD0 si AD sunt concurente ntr-un punct E, iar paralela prin E la baze
intersecteaz
a BC n F astfel nct EF este media geometric
a a lungimilor bazelor.
a + kb
a kb
3. n sfrsit, sa observam ca pentru x =
, obtinem f (x) =
1+k
1k
(calculul se efectueaza cu usurinta). Prin urmare, considernd segmentul [AA0 ] de
lungime egal
a cu media ponderat
a a bazelor cu ponderile 1 si k, segmentul [KL] va
reprezenta media ponderat
a a bazelor, cu ponderile 1 si k. Acest fapt ne permite
sa construim cu rigla si compasul conjugatul armonic al unui punct; cititorul poate
dezvolta singur ideile.
KL = K 0 LKK 0 =

Bibliografie
1. L. Constantinescu - O interpretare geometrica a inegalitatii mediilor, R.M.T. 1/1982, 30.
2. C. - S
t. Popa - O constructie geometrica a unor medii, Rec. Mat. - 2/2003, 13-14.

103

O generalizare a teoremelor de baz


a
ale calculului diferential
Florin POPOVICI 1
Nota de fata si propune sa extinda teoremele de baza ale calculului diferential prin
impunerea conditiei de derivabilitate bilateral
a n locul conditiei clasice de derivabilitate. Rezultatele obtinute se exprim
a sub forma unor conditii de apartenenta,
care pot fi, nsa, interpretate geometric.
Teorema 1 (Teorema lui Fermat generalizata ). Fie f : [a, b] R o functie data.
Daca c (a, b) este un punct de extrem local al functiei f si aceasta este derivabila
bilateral (la stnga si la dreapta)
0 n punctul
c, atunci
0

0
0
0 min f
(c) , f+
(c) , max f
(c) , f+
(c) .
(1)
Demonstratie. Fie c punct de maxim local. Asadar, > 0 astfel nct avem:
f (x) f (c)
0
(c) 0,
x (c , c) [a, b] f (x) f (c)
0 f
xc
f (x) f (c)
0
x (c, c + ) [a, b] f (x) f (c)
(c) 0
0 f+
xc

0
0
si, ca urmare, 0 f+ (c) , f
(c) , adic
a are loc2 (1).
Exemplu. Functia f (x) = max x, 1 x , x 0, este
y

51
, pentru care
derivabil
a cu exceptia punctului x0 =
2

0
0
f (x0 ) = 1 5 si f+ (x0 ) = 1. Relatia (1) revine la 0

1 5, 1 , adic
a semitangentele la grafic n M0 (x0 , x0 ) sunt
x
x0
de parti diferite fata de paralela prin acest punct la axa Ox.
Teorema 2 (Teorema lui Rolle generalizata ). Daca f : [a, b] R este o functie
continua pe [a, b], derivabila bilateral pe (a, b) si f (a) = f (b), atunci exista un punct
c (a, b) astfel nct are loc relatia (1).
Demonstratie. Conform teoremei lui Weierstrass, functia f este m
arginit
a si si
atinge marginile, adica c1 , c2 [a, b] astfel nct f (c1 ) f (x) f (c2 ), x [a, b].
Daca {c1 , c2 } {a, b}, atunci rezulta ca f este functie constanta si (1) are loc
pentru orice c (a, b). Dac
a {c1 , c2 } 6 {a, b}, fie c {c1 , c2 } \ {a, b}; conform
Teoremei 1 pentru acest punct c are loc (1).
Teorema 3 (Teorema lui Lagrange generalizata ). Daca f : [a, b] R este
o functie continua pe [a, b] si derivabila bilateral pe (a, b), atunci exista un punct
c (a, b) astfel nct

0
f (b) f (a)
0
0
(c) , f+
(c) , max f
(c) , f+
(c) .
(2)
min f
ba
Demonstratie. Se aplica Teorema 2 functiei auxiliare g : [a, b] R definite prin
f (b) f (a)
g (x) = f (x)
x, x [a, b].
ba
Corolarul 1 (Teorema lui Lagrange pentru functii convexe). Fie f : I R ( I
interval deschis) o functie convexa. Atunci, pentru orice puncte a, b I, a < b,
exista c (a, b) astfel nct
1

Profesor, Liceul Teoretic "N. Titulescu", Brasov

104

f (b) f (a)
0
(c) .
f+
ba
Demonstratie. Deoarece f este convex
a pe I (deschis), rezult
a c
a f este continu
a
0
0
(x) f+
(x), x I. Prin aplicarea Teoremei 3
pe I, derivabil
a bilateral pe I si f
pe intervalul [a, b], obtinem rezultatul cerut.
Corolarul 2. Fie f : I R ( I interval deschis) o functie continua si derivabila bilateral pe I. Atunci functia f este crescatoare pe I daca si numai daca este
satisfacuta conditia
0

0
min f
(x) , f+
(x) 0, x I.
(3)
Demonstratie. Necesitatea conditiei este usor de dovedit. Pentru suficienta, fie
x1 , x2 I cu x1 < x2 . Conform Teoremei 3, aplicat
a restrictiei functiei f la [x1 , x2 ],
0

f (x2 ) f (x1 )
0
min f
(c) , f+
(c) .
c (x1 , x2 ) astfel nct
x2 x1
De aici si din (3), deducem c
a f (x1 ) f (x2 ); ca urmare, functia f este cresc
atoare.
Teorema 4 (Teorema lui Cauchy generalizata ). Daca f, g : [a, b] R sunt doua
functii continue pe [a, b], derivabile bilateral pe (a, b) si

0
0
0
/ min g
(x) , g+
(x) , max g
(x) , g+
(x) , x (a, b) ,
(4)
atunci g (a) 6= g (b) si exista un punct c (a, b) astfel nct

0
0
f (c) f+
f (c) f+
(c)
(c)
f (b) f (a)
min
,
,
max
,
.
(5)
0 (c) g 0 (c)
0 (c) g 0 (c)
g (b) g (a)
g
g
+
+
Demonstratie. Daca am avea g (a) =
2, ar
conform
Teoremei
exista
g0 (b), atunci,
0
0
0
(c) , g+
(c) , max g
(c) , g+
(c) , ceea
un punct c (a, b) astfel nct 0 min g
ce contrazice (4). Deci are loc g (a) 6= g (b).
Consider
am acum functia h : [a, b] R definit
a prin
f (b) f (a)
h (x) = f (x)
g (x) , x [a, b] .
g (b) g (a)
Observam ca h este continua pe [a, b], derivabila bilateral pe (a, b) si avem
f (a) g (b) f (b) g (a)
h (a) =
= h (b). Conform Teoremei 2, exista c (a, b) astfel
(b) g (a)

g
nct 0 min h0 (x) , h0+ (x) , max h0 (x) , h0+ (x) , adica avem
f (b) f (a) 0
f (b) f (a) 0
0
0
(c)
(c)
(6)
g (c) 0 f+
g (c)
f
g (b) g (a)
g (b) g (a) +
sau
f (b) f (a) 0
f (b) f (a) 0
0
0
(c)
(c)
(7)
g (c) 0 f
g (c) .
f+
g (b) g (a) +
g (b) g (a)
Presupunem ca are loc (6) (se procedeaza analog, daca ar avea loc (7)). Datorita
ipotezei (4), putem scrie

0
0
0
(c) , g+
(c) > 0 (8)
sau
max g
(c) , g+
(c) < 0 (9) .
min g
Dac
a are loc (8), atunci (6) ia forma
0
(c)
f
f 0 (c)
f (b) f (a)

+
0
0 (c) ,
g (c)
g (b) g (a)
g+
deci (5) este adevarata. Daca are loc (9), atunci (6) se scrie
0
f 0 (c)
(c)
f+
f (b) f (a)


0
0 (c) ,
g+ (c)
g (b) g (a)
g
deci (5) este adev
arat
a si n acest caz. Demonstratia este complet
a.
0
f
(c)

105

Asupra unei inegalit


ati
Alexandru NEGRESCU 1
La a V-a editie a Concursului interjudetean de matematica "Radu Miron", noiembrie 2003, elevilor clasei a IX-a li s-a propus urm
atoarea problem
a:
1
1
1
Fie x1 , x2 , . . . , xn (2, +) astfel nct
+
+ +
= 1.
x1 1
x2 1
xn 1
n
Demonstrati ca x1 x2 . . . xn (n + 1) .
Vom da 5 demonstratii acestei inegalit
ati.

Solutia I. Notam xi 1 = ai , i = 1, n; deci ai > 1. Aplicam inegalitatea lui


Huygens:

n
(1 + a1 ) (1 + a2 ) (1 + an ) (1 + n a1 a2 an ) , a1 , a2 , . . . , an 0.
Obtinem

n
p
(1)
x1 x2 . . . xn 1 + n (x1 1) (x2 1) (xn 1) .

Dar, conform cu inegalitatea dintre mediile armonica si geometrica, avem


p
n
n (x1 1) (x2 1) (xn 1)
1
1
1
+
+ +
x1 1 x2 1
xn 1
sau, tinnd seama de conditia din enunt,
p
n n (x1 1) (x2 1) (xn 1).
Din (1) si (2) rezulta ca x1 x2 . . . xn (1 + n)n , q.e.d.
Solutia II. Scriem

(2)

s
n
xi 1
xi 1 xi 1
xi 1
n+1
.
xi = 1 + (xi 1) = 1 +
+
+ +
(n + 1)
n
n
n
n
Ca urmare,
r
n
n
n
n n+1 (x1 1) (x2 1) (xn 1)
.
(3)
x1 x2 . . . xn (n + 1)
2
nn
n
Din (2), avem (x1 1) (x2 1) (xn 1) n , deci
r
n
n
n
n+1 (x1 1) (x2 1) (xn 1)
1.
(4)
nn2
Combinnd (3) si (4), obtinem inegalitatea ceruta.
1
Solutia III. Not
am
= yi , i = 1, n; deci yi (0, 1) si y1 + y2 + + yn = 1.
xi 1
yi + 1
Rezult
a c
a xi =
, 1, n si avem:
yi
y1 + 1 y2 + 1
yn + 1

...
=
x1 x2 . . . xn =
y1
y2
yn
1

Elev, cl. a IX-a, Colegiul National "A. T. Laurian", Botosani

106

2y1 + y2 + + yn y1 + 2y2 + + yn
y1 + y2 + + 2yn

...

y1
y2
yn
q
n
n+1
(n + 1) n+1 (y1 y2 yn )
n

= (n + 1) .
y1 y2 yn

1
1
Solutia IV. Apelam la metoda lui Sturm. Deoarece xi = 1 +
,
xi 1
xi 1
i = 1, n, inegalitatea de demonstrat se scrie
"n
# Y
n
Y
1
1
(5)
1+
(n + 1)n .
x

1
x

1
i
i
i=1
i=1
=

1+a 1+b
Sa analizam comportarea produsului

, cu a, b > 0 si a + b este cona


b
stant
a sububitar
a, atunci cnd a si b "se apropie". Presupunem a < b si nlocuim
numerele a si b cu a + t si respectiv b t, unde 0 < t < b a. Atunci
t (1 + a + b) (a b + t)
1+a+t 1+bt 1+a 1+b

=
< 0,
a+t
bt
a
b
ab (a + t) (b t)

1+a 1+b
ceea ce arat
a c
a apropiind numerele a si b produsul

descreste.
a
b
1
1
1
Daca printre numerele
,
, ... ,
exista doua inegale, atunci
x1 1 x2 1
xn 1
1
1
1
1
si celalalt este strict mai mare ca ; fie
unul este strict mai mic ca
<
n
n
x1 1
n
1
1
1
1
1
1
1
1
si

> . nlocuim
si
prin si respectiv
+
.
x2 1
n
x1 1
x2 1
n
x1 1 x2 1 n
1
1
1
1
1
1
Suma numerelor ,
+
,
, ... ,
r
amne aceeasi,
n x1 1
x2 1
n x3 1
xn 1
dar membrul stng n (5), obtinut prin aceast
a nlocuire, este mai mic. n noul set
1
de numere avem unul egal cu , iar, daca printre celelalte exista doua inegale, se
n
1
procedeaza la fel pna cnd se obtine un set de numere egale cu . n acest caz
n
membrul stng are valoare minim
a, anume,

1 .1
n
= (n + 1) .
1+
n
n
Solutia V. Vom dovedi mai nti rezultatul urmator:
Lem
a. Daca ai , bi > 0, i = 1, n, atunci
v
v
v
un
un
un
uY
uY
uY
n
n
n
t
t
(ai + bi )
ai + t
bi .
i=1

i=1

i=1

Demonstratie. Inegalitatea se poate scrie astfel:


r
r
an
bn
a1
b1
n
1
...
+ n
...
a1 + b1
an + bn
a1 + b1
an + bn
107

(6)

si rezult
a aplicnd inegalitatea mediilor:

b1
a1
1
an
bn
1=
+ +
+ +
+

n
a1 + b1
an + bn
a1 + b1
an + bn
r
r
an
bn
a1
b1
n
...
+ n
...
.
a1 + b1
an + bn
a1 + b1
an + bn
Lund n (6) ai = xi 1 si bi = 1, i = 1, n, obtinem
(2)
p

n
x1 x2 . . . xn n (x1 1) (x2 2) (xn 1) + 1 n + 1,
de unde rezult
a inegalitatea dorit
a.
Observatie. Aceasta problema poate fi usor generalizata astfel:
1
1
1
Fie x1 , x2 , . . . , xn (2, +) astfel nct
+
+ +
= k.
x2 1
xn 1

n x1 1
k+n
Demonstrati ca x1 x2 . . . xn
.
k
Bibliografie
1. Gh. Andrei si colab. - Exercitii si probleme de algebra pentru concursuri si olimpiade scolare, Partea I, Constanta, 1990.
2. M. Ganga - Manual pentru clasa a IX-a, Profil M1,M2, Ed. Mathpress, Ploiesti,
2003.
3. M. Ganga - Probleme elementare de matematica, v. II, Ed. Mathpress, Ploiesti,
2003.
4. D. S
t. Marinescu, V. Cornea - Doua inegalitati si unele aplicatii ale acestora,
Gazeta Matematic
a, CVI (2001), nr. 3, 102-104.
5. I. Nedelcu - Probleme de matematica pentru liceu, Ed. Mathpress, Ploiesti, 2003.
6. L. Panaitopol, M. Lascu, V. B
andil
a - Inegalitati, Ed. GIL, Zalau, 1996.

1 (Problema de cnt
arire a lui Bachet). Care este cel mai mic num
ar de
greutati care pot fi folosite pentru a cntari cu o balanta ori numar ntreg de kilograme
de la 1 la 40?
2. G
asiti dou
a numere care se scriu n baza 10 numai cu ajutorul cifrei 1 si care
au suma egala cu produsul lor.
3. Un urs pleac
a din brlogul s
au 1 km spre sud, se ntoarce si parcurge 1 km
spre est, apoi 1 km spre nord, revenind astfel n punctul de plecare. Ce culoare are
ursul?
Not
a. R
aspunsurile la aceste probleme se g
asesc la p. 110 si la p. 123

108

Asupra problemei VII.41 din RecMat - 2/2003


n nr. 2/2003 al revistei Recreatii matematice este publicata urmatoarea problema,
propus
a de elevul Alexandru Negrescu din Botosani:
a
b
+
= 1.
b+1 a+1
Aceast
a problem
a a fost apoi propus
a elevilor de cl. a VII-a n cadrul Concursului
"Recreatii matematice", editia a III-a, 2003, Iasi.
Aceste mprejurari, ct si accesibilitatea problemei, au facut ca aceasta sa se
bucure de atentia elevilor. Ca rezultat, au fost date mai multe solutii distincte
sau variante ale lor. Nota de fata colecteaz
a aceste solutii. Cititorul va observa
entuziasmul si pasiunea cu care elevii au atacat problema VII.41.
VII.41. Rezolvati n N2 ecuatia

Solutia I (Alexandra Ciofu, eleva, Hrlau). Daca perechea (a, b) este solutie
a
b
a ecuatiei, atunci
1 si
1, de unde a b + 1 si b a + 1.
b+1
a+1
I Daca a b, rezulta ca 0 a b 1, deci a b {0, 1}. n cazul a b = 0,
2a
= 1. De aici, obtinem a = 1
urmeaz
a a = b si, deoarece (a, b) este solutie,
a+1
si, deci, perechea (1, 1) va fi solutie a ecuatiei date. n cazul a b = 1, nlocuind n
b
= 1, deci b = 0. Asadar, n acest caz
ecuatia dat
a pe a cu b + 1 obtinem 1 +
b+2
obtinem solutia (1, 0).
II Dac
a a b, rolurile numerelor a si b se schimb
a si (a, b) va fi (1, 1) sau (0, 1).
Rezumnd, multimea solutiilor ecuatiei date este {(0, 1) , (1, 0) , (1, 1)}.
Solutia II (Maria Cr
aciun, eleva, Hunedoara). Ecuatia data este echivalenta
a
cu a (a + 1) + b (b + 1) = (a + 1) (b + 1), deci cu a2 ab + b2 = 1. Aceasta din urm
poate fi scris
a sub forma a (a b) b (a b) = 1 ab sau (a b)2 = 1 ab. Asadar,
numarul 1 ab este un patrat perfect cel mult egal cu 1, daca perechea (a, b) este
solutie a ecuatiei. Ca urmare, 1 ab {0, 1}, adic
a ab {0, 1}.
Dac
a ab = 1, atunci perechea (a, b) va fi (1, 1) si aceasta verific
a ecuatia.
Dac
a ab = 0, atunci a = 0 sau b = 0. Dac
a a = 0, nlocuind n ecuatie obtinem
b = 1; perechea (0, 1) verifica ecuatia din enunt. Daca b = 0, obtinem n mod
asem
an
ator solutia (1, 0). Dac
a a = 0 si b = 0, verific
am c
a perechea (0, 0) nu-i
solutie.
n concluzie, multimea solutiilor ecuatiei este {(0, 1) , (1, 0) , (1, 1)}.
Solutia III (Bogdan-Alexandru Burican, elev, Hrlau). Ecuatia este echiva2
lent
a cu a2 ab + b2 = 1 si apoi cu a2 + b2 + (a b) = 2. Rezult
a c
a a2 2, deci
a {0, 1}.
Daca a = 0, din ecuatia data obtinem b = 1.
Dac
a a = 1, din relatia a2 ab + b2 = 1 obtinem b2 b = 0, de unde b = 0 sau
b = 1.
Prin urmare, perechile (0, 1), (1, 0) si (1, 1) sunt solutiile ecuatiei date.
Solutia IV (Diana Prodan, eleva, Iasi). Ecuatia a2 ab + b2 = 1, echivalenta
109

cu ecuatia din enuntul problemei, se pune n forma 3a2 + (a 2b) = 4. De aici,


avem 3a2 4, deci a {0, 1}. Se continua ca n Solutia III.
Solutia V (Adrian Hamciuc, elev, Iasi). Ca mai sus, obtinem ecuatia
a2 ab + b2 = 1. Cum a2 + b2 2ab, rezulta ca ab 1, adica ab {0, 1}. Se
ncheie ca n Solutia II.
Solutia VI (Diana Timofte, eleva, Iasi). Fie (a, b) o solu
2tie a ecuatiei date, deci
2
si a ecuatiei a2 ab + b2 = 1. Ca urmare, ecua
t
ia
b

ab

a 1 = 0, considerat
a

n b, are solutii reale si atunci = a2 4 a2 1 0. Rezulta ca 3a2 4, deci


a {0, 1}. Se continu
a ca n Solutia III.
Solutia VII (Alexandru Negrescu, elev, Botosani). Conform inegalit
atii
Cauchy-Buniakovski-Schwarz, avem

a
b
[(b + 1) + (a + 1)]
a+ b ,
+

b+1 a+1

a + b (b + 1) + (a + 1), adica a + b + 2 ab a + b + 2 sau ab 1.


de unde
De aici ab {0, 1} etc.
Observatie. Solutiile prezentate mai sus au comun faptul ca n prima parte se
obtine, cu tehnici diverse de calcul, una dintre relatiile a {0, 1}, a b {0, 1},
ab {0, 1}. n partea a doua sunt utilizate aceste relatii (una dintre ele) pentru
determinarea multimii solutiilor ecuatiei.

Solutia Problemei de cntarire a lui Bachet (p. 108)


Dac
a greut
atile se pun pe un singur taler, sunt necesare sase greut
ati: 1 kg, 2 kg,
4 kg, 8 kg, 16 kg, 32 kg. ntr-adevar, orice greutate de la 1 la 40 poate fi atinsa
astfel:
1 = 1,
6 = 4 + 2,
2 = 2,
...............
3 = 2 + 1, . . . . . . . . . . . . . . .
4 = 4,
39 = 32 + 4 + 2 + 1,
5 = 4 + 1, 40 = 32 + 8.
Daca greutatile pot fi puse pe ambele talere, sunt necesare numai patru greutati:
1 kg, 3 kg, 9 kg, 27 kg. ntr-adevar, avem:
1 = 1,
6 = 9 3,
2 = 3 1,
...............
3 = 3,
...............
4 = 3 + 1,
39 = 27 + 9 + 3,
5 = 9 3 1, 40 = 27 + 9 + 3 + 1.
110

Asupra unei probleme de concurs


Dumitru MIHALACHE, Marian TETIVA1
n aceast
a not
a ne propunem s
a prezent
am dou
a modalit
ati de abordare a unei
probleme de geometrie si s
a obtinem o generalizare a sa (rostul ghilimelelor se va
vedea la vremea potrivita). Problema a fost propusa de C. Apostol la Concursul
National de Matematica Laurentiu Duican (ajuns, iata, la cea de a XI-a editie:
felicit
ari si succes n continuare!) si enuntul ei poate fi citit n [1]; iat
a acest enunt
(modificat pentru a retine doar esentialul):
b = 80 , m(B)
b = 120 ,
Problema 1. n patrulaterul convex ABCD avem m(A)

b
b
b
m(C) = 50 , m(D) = 110 . Sa se arate ca, daca (BD este bisectoarea unghiului B,
b
atunci (AC este bisectoarea unghiului A.

ntr-o prima faza, negasind nici o alta idee, am ncercat o rezolvare a problemei
bazata pe calcule trigonometrice. Rezultatul acestei cautari este
Metoda I de rezolvare a problemei. Din
C
\ = m(DBC)
\ =
D
ipotez
a rezult
a c
a m(DBA)
N
\ = 40 , iar
= 60 si, apoi, ca m(ADB)
\ = 70 . S
m(BDC)
a consider
am punctele
M AB (A (M B)) si N (BC) astfel nct triunghiurile M BD si BN D sa fie
P
\
echilaterale (prin urmare m(M
DA) = 20 si
\
m(N
DC) = 10 ). Fie a lungimea laturilor M
A
B
acestor triunghiuri. Cu teorema sinusurilor n
triunghiul M AD obtinem
MA
a
a sin 20
a sin 20
=

M
A
=
=
,
sin 20
sin 100
sin 100
sin 80
iar din triunghiul N CD,
a
a sin 10
NC
=

N
C
=
.
sin 10
sin 50
sin 50
Atunci
2 sin 30 cos 50
cos 50
sin 80 sin 20
=a
=a
AB = M B M A = a

sin 80
sin 80
sin 80
si
cos 20
sin 50 + sin 10
=a
,
BC = BN + N C = a

sin 50
sin 50
prin urmare
2 cos 20 sin 80
sin 40
BC
cos 20 sin 80

=
=
2
cos
20
=
=
AB
sin 50 cos 50
sin 100
sin 20
(este evident c
a am aranjat acest raport, stiind unde vrem s
a ajungem).
1

Profesori, Colegiul National "Gh. Rosca Codreanu", Brlad

111

\ deci m(BCA)
\ = 60 x; tot cu teorema sinusurilor
Acum, fie x = m(BAC),
(acum n triunghiul ABC) si tinnd seama de calculul anterior, avem
sin x
sin 40
sin x sin 20 = sin (60 x) sin 40 ;
=
sin (60 x)
sin 20
transformam produsele n sume si avem
deci

cos (x 20 ) cos (x + 20 ) = cos (20 x) cos (100 x) ,

cos (100 x) cos (x + 20 ) = 0 sin 60 sin (40 x) = 0.


Cum, evident, 0 < x < 80 , de aici rezulta x = 40 si problema este rezolvata.
Desigur, asemenea calcule nu sunt pentru cl. VII-a (iar problema a fost propusa
elevilor acestei clase); s
a vedem asadar si o solutie a problemei la acest nivel.
Metoda a II-a se bazeaz
a pe observatia urm
atoare: dac
a P (BC) este punctul

\
pentru care m(P DC) = 50 , atunci triunghiul P DC este isoscel cu P D = P C
\
(evident), iar triunghiul ADP este echilateral. ntr-adev
ar, unghiul BP
D, se vede
imediat, are masura de 100 , de aceea patrulaterul ABP D este inscriptibil (avnd
\
\ = 60 , iar, pe de
dou
a unghiuri opuse suplementare). Atunci m(AP
D) = m(DBA)

\) = m(ADC)
\ m(P
\
alt
a parte, m(ADP
DC) = 110 50 = 60 , deci triunghiul

ADP are dou


a unghiuri de m
asur
a 60 .
Rezult
a c
a avem AP = DP = P C si triunghiul P AC este tot isoscel; prin urmare
[
180 m(AP
C)
m(P[
AC) = m(P[
CA) =
= 20 ,
2
\ = 40 si demonstratia se ncheie.
de unde rezult
a imediat m(BAC)
F
ar
a ndoial
a, aceast
a a doua variant
a de solutionare a problemei este mult mai
simpl
a dect prima; totusi, ideea de a alege punctul P nu vine prea usor. n schimb,
rolul sau n problema este fundamental; de fapt, noi credem ca problema a fost
construita pornind de la cele doua triunghiuri isoscele lipite, ADP si CDP . Ca
triunghiul ADP este chiar echilateral nu este esential si de asta v
a puteti convinge
rezolvnd urm
atoarea generalizare a primei probleme.
b = + , m(B)
b = 2 + ,
Problema 10 . Fie ABCD un patrulater n care m(A)

b = , m(D)
b = + , m(DBC)
\ = , unde , , > 0 si 2 + + = 180 .
m(C)
Atunci:
a) 90 > > .
b
b) (AC este bisectoarea unghiului A.
b
\ avem 2+ > si 2 < 2++ = 180
Solutie. a) Deoarece m(B) > m(DBC),
\ = 2+ si m(ADB)
\ = 180 (2 + + + ) =
< 90 . Apoi, m(ABD)
= , deci > .
b) Alegem un punct P (BC) astfel nct
\
\
m(P
DC) = m(P
DB) = .

\ = + . Mai departe
Acest punct exist
a pe segmentul (BC), deoarece m(BDC)
demonstratia decurge ca mai sus: observati ca triunghiul P CD este isoscel, apoi ca
112

\
ABP D este patrulater inscriptibil si c
a triunghiul P AD este isoscel (cu m(P
AD) =
\
= m(P DA) = ). Rezult
a triunghiul P AC isoscel, de unde se va putea calcula
\ = + , ceea ce ncheie rezolvarea.
masura lui P[
AC si apoi se ajunge la m(BAC)
2
Nu e o generalizare efectiv
a, totul se bazeaz
a pe aceeasi idee (Problema 1 se
a de ce am pus cuvntul ntre
reg
aseste pentru = 50 , = 60 si = 20 ): iat
ghilimele.
Iar pentru a ncheia lasam ca tema o problema nrudita cu cele de mai sus
(eventual ncercati si o generalizare a ei).
b = 110 si m(C)
b = 50 . ConsideProblema 2. Fie ABC un triunghi cu m(B)
\
\
ram punctele M (AC) si N (AB) astfel nct m(M
BC) = 70 si m(N
CB) = 30 .

\
Sa se calculeze m(AM
N ). (R
aspuns: 60 .)
Bibliografie
1. F. Diac - A XI-a editie a Concursului National de Matematica Laurentiu Duican,
Brasov, 2003, G. M. 11/2003.

LISTA MEMBRILOR FILIALEI IA


SI a S. S. M.
continuare din nr. 1/2000, 1/2001, 1/2002, 1/2003 si 1/2004
126.
127.
128.
129.
130.
131.
132.
133.
134.
135.
136.
137.
138.
139.
140.
141.
142.
143.
144.
145.
146.
147.

DIMITRIU Gabriel

RADUCANU
Petru
IUREA Gheorghe
Cristian
LAZAR
PETCU Alina Emilia
POPA Gabriel
AM
ANU

Laura
A
VAT
T
NEDELCU Andrei
AU
SU Alexandru
CAR
ROMAN Neculai

CALIN
Ionela
S Angelica
GOLAE
BEJAN Tinuta
BUZAC Gabriela - Tamara

PADURARU
Adriana
LUCHIAN Dorel
COZLAC Magda
LUCA TUDORACHE Rodica
POPA Antoaneta
ARBONE Dorina
IONESCU Mihaela
SAVA Radu

I. M. F., Iasi
Liceul "D. Cantemir", Iasi
Liceul "D. Cantemir", Iasi
Colegiul National, Iasi
Liceul Energetic, Iasi
Colegiul National, Iasi
S
coala Waldorf, Iasi
Liceul "Gr. Moisil", Iasi
Univ. Tehnic
a "Gh. Asachi", Iasi
S
coala "V. Alecsandri", Mircesti (Iasi)
Liceul "D. Mangeron", Iasi
Gr. sc. ind. usoar
a "Victoria", Iasi
S
coala "Al. Vlahuta", Iasi
Liceul Economic nr. 1, Iasi
S
coala "B. P. Hasdeu", Iasi
Liceul "M. Costin", Iasi
Liceul ind. nr. 7, Iasi
Univ. Tehnica "Gh. Asachi", Iasi
S
coala Mnzatesti (Iasi)
S
coala "Mircea cel B
atrn", Iasi
S
coala "I. Ghica", Iasi
Colegiul "C. Negruzzi", Iasi
(continuare la p. 128)
113

Exponentul num
arului natural a n produsul n!
1

Mihai CRACIUN

n cele ce urmeaz
a, date fiind a, n N , vom indica o formul
a de calcul a exponentului lui a n n!, notat expn! (a), mpreun
a cu cteva aplicatii.

n
n
n
Teorema 1. Daca p este prim, atunci expn! (p) =
+ 2 + 3 + .
p
p
p
Demonstratie. Mai nti, se observ
a c
a suma din enunt este finit
a si se continu
a
pn
a cnd termenul curent are partea ntreag
a egal
a
cu
0.

n
Dintre factorii
produsului
n!,
un
num
a
r
de
vor fi multipli de p. Dintre ei,
un

p
n
n
2
numar de 2 vor fi multipli de p , iar dintre acestia din urma, un numar de 3
p
p
vor fi multipli ai lui p3 etc. Suma numerelor indicate va fi exponentul cerut, deoarece
fiecare factor al produsului n! care este multiplu al lui pm f
ar
a a fi si multiplu al lui
pm+1 se socoteste n modul indicat de m ori, ca multiplu al lui p, p2 , . . . , pm .

50
50
50
50
Exemplul 1. exp50! (3) =
+
+
+
= 16 + 5 + 1 + 0 = 22.
3
9
27
81

expn! (p)

Teorema 2. Daca p este prim iar N , atunci expn! (p ) =


.

Demonstratia este evident


a.


7
exp50! 3
22
=
= 3.
Exemplul 2. exp50! 3 =
7
7
Teorema 3 (Legendre). Daca a N este descompus n factor primi sub forma
k
k
1
2
1 2
a = p
1 p2 pk , atunci expn! (a) = min {expn! (p1 ) , expn! (p2 ) , . . . , expn! (pk )} .
Demonstratia este imediat
a.
Exemplul 3.



exp253! 2
exp253! 3
exp253! (108) = min exp253! 22 , exp253! 33 = min
,
.
2
3
Deoarece exp253! 2 = 246 si exp253! 3 = 125, urmeaza ca exp253! (108) = 41.
Problema 1. Determinati cu cte zerouri se termina 1958!
Solutie. Num
arul de zerouri este dat de exp1958! (10). Se observ
a c
a
exp1958! (10) = min {exp1958! 2, exp1958! 5} = exp1958! (5) .


1958
1958
1958
1958
1958
+
+
+
= 467,
+
exp1958! (5) =
5
52
53
54
55
deci 1958! se termin
a cu 467 zerouri.
Problema 2. Demonstrati ca n! nu se divide cu 2n , n N.
Solutie. Fie n N si fie k N astfel ca 2k n < 2k+1 . Atunci

hni h n i
h n i n n
n
1
expn! (2) =
+ 2 + + k+1 + 2 + + k+1 = n 1
<n
2
2
2
2 2
2
2k + 1

Atunci

Profesor, Liceul "M. Sadoveanu", Pascani

114

deci expn! (2) < n si n! nu se divide cu 2n .

5n 1
Problema 3. Determinati cu cte zerouri se termin
a
!, unde n N .
4
n
5 1
. Ca mai sus, numarul de zerouri este dat de
Solutie. Sa notam N =
4
expN ! 5. n plus
n

n
5 1
5 1
5 1
+ +
+ =
expN ! 5 =
+
45
4 52
4 5k

n
n
5 5k + 5k 1
5 5+51
5 52 + 52 1
+ +
+ =
=
+
45
4 52
4 5k
5n1 1 5n2 1
5nn 1
5n 4n 1
=
+
+ +
=
,
4
4
4
16
5k 1
5k 1
deoarece
[0, 1). De aici, N ! se termina cu
N pentru k N , iar
4
4 5k
5n 4n 1
zerouri.
16
Problema 4. Factorialul caror numere se termina exact n 1000 zerouri?
Solutie. Fie n N un numar al carui factorial
a exact
h n i hsen termin
i
h n ni 1000 zerouri
k
k+1
si fie k N astfel ca 5 n < 5 . Atunci
+ 2 + + k+1 = 1000, deci

5
5
5h i
n
n
n
n
1
n
+ 2 + + k =
1 k > 1000 si n > 1000. n plus,
< 1000, deci
5
5
5
4
5
5
n < 5005,
ceea
a pentru
n 4004, n! se termin
a n cel

a k = 5.
Se
observ
ac

ce implic
4004
4004
4004
4004
4004
mult
+
+
+
= 999 zerouri, n vreme ce pentru
+
4
5
5
52
53
5 5

4010
4010
4010
4010
4010
n 4010, n! se termin
a n cel putin
+
+
= 1001
+ 2 +
5
5
53
54
55
zerouri. Numerele cerute sunt 4005, 4006, 4007, 4008, 4009.
(2m)! (2n)!
Problema 5. Demonstrati ca
N, unde m, n N.
m!n! (m + n)!
Solutie. Mai nti, se poate demonstra inegalitatea
[x] + [y] + [x + y] [2x] + [2y] , x, y R.
(1)
Exponentul
unui factor prim p din descompunerea canonica a numaratorului este
X n m m + n
s1 =
+ k +
, n vreme ce exponentul lui p din descompk
p
pk
X 2m 2n
k
punerea canonic
a a num
ar
atorului este s2 =
+ k
. Folosind (1)
pk
p
k
(2m)! (2n)!
N.
obtinem c
a s1 s2 si deci
m!n! (m + n)!
Probleme propuse.
101001!
?
1. Cu cte zerouri se termin
a num
arul
2004!
2. Sa se demonstreze ca numerele A = 111! 222! 333! 444! si B = 555! 666!
777! 888! sunt divizibile cu 10268 , respectiv 10715 .
3. Factorialul c
aror numere se termin
a cu exact 2004
zerouri?
4. Aflati exponentul lui k n 1 + k + k 2 + + kn !, unde k, n N , k 3.
115

Concursul de matematic
a Al. Myller
Editia a II-a, Iasi, martie 2004
Not
a (pentru clasele IV-VI ). Toate subiectele sunt obligatorii. Timp efectiv de lucru
90 min. Se acord
a din oficiu 30 puncte, cte 6 puncte pentru problemele 1-5, cte 8 puncte
pentru problemele 6-10 si cte 10 puncte pentru problemele 11-15.

Clasa a IV-a
1. Un elev rezolva fiecare dintre primele 5 probleme ale acestui test n cte 3
minute, iar pe fiecare dintre urm
atoarele 5 n cte 5 minute. Cte minute i trebuie
pentru a rezolva una dintre ultimele 5 probleme, presupunnd c
a fiecare din ele i
solicita acelasi timp? (timpul total de lucru este 1h 30 min.)
2. Calculeaza (100 99) + (98 97) + (96 95) + + (2 1).
3. Calculeaz
a (5 + 55 + 555 + 5555 + 55555) : (1 + 1 + 111 + 1111 + 11111) : 5.
4. Cei 41 de elevi ai unei clase urc
a n sir pe munte. Mircea observ
a c
a n fata
lui sunt un sfert dintre colegii sai. Al ctalea n sir este Mircea?
5. Delia calculeaza suma cifrelor pe care le afiseaza ceasul ei digital (de exemplu
la ora 14:28 ea obtine 1 + 4 + 2 + 8 = 15). Care este suma maxim
a pe care o poate
obtine?
6. Care sunt ultimele trei cifre ale numarului 1 2 3 4 5 . . . 2004 + 12?
7. Aflati suma dintre dempartit, mpartitor, ct si rest, stiind ca restul este cu
18 mai mic dect ctul, ctul este 25, demp
artitul este impar, iar mp
artitorul are o
singur
a cifr
a.
8. Un numar se mparte la 3 si da restul 2. Ctul se mparte din nou la 3,
obtinnd restul 2. Noul ct se mparte iar la 3 si gasim ctul 2 si restul 2. Care a
fost num
arul initial?
9. La un magazin se aduc 301 kg de mere n l
azi de 25 kg si 21 kg. Cte l
azi se
folosesc n total?
10. 58 de elevi sunt asezati pe 4 rnduri, fiecare rnd avnd cu 3 elevi mai putin
dect rndul din fata sa. Cti elevi sunt pe ultimul rnd?
11. Dan vrea s
a cumpere mingi. Dac
a ar cump
ara 5, i-ar mai r
amne 100 000 lei,
iar daca ar dori sa cumpere 7, ar mai avea nevoie de 220 000 lei. Ct costa o minge?
12. Marinarii de pe un vapor au hrana pentru 60 de zile. Ei gasesc pe o insula
30 de naufragiati si astfel hrana le va ajunge tuturor doar 50 de zile. Cti marinari
erau pe vapor?
13. 12 baeti si 8 fete sunt membri ai cercului de matematica. n fiecare saptamna,
nca 2 fete si 1 baiat sunt acceptati ca membri ai cercului. Cti membri va avea cercul
de matematic
a atunci cnd num
arul b
aietilor va fi egal cu num
arul fetelor?
14. Cte numere naturale de patru cifre au ultima cifr
a 3?
15. Pe o insula locuiesc numai arici, serpi si vulpi. Fiecare animal mannca o
data pe zi, astfel nct orice arici mannca la micul dejun cte un sarpe, orice vulpe
m
annc
a la prnz cte un arici, iar orice sarpe m
annc
a la cin
a cte o vulpe. La
sfrsitul zilei de miercuri, pe insul
a a r
amas un singur animal. Cte animale existau
pe insula luni, nainte de micul dejun?
116

Clasa a V-a
1. Sa se calculeze suma 4 + 8 + 12 + + 2000.
2. Determinati numerele naturale x astfel nct multimile A = {2x; 6x + 4; 3x + 5}
si B = {2x 1; 2x + 1; 5x + 6} s
a aib
a un singur element comun.
3. Care sunt numerele prime a, b, c pentru care a + 10 b + 12 c = 82?
4. Aflati suma cifrelor numarului A = 102004 1.
5. Sa se determine perechile de numere naturale (x, y) pentru care fractia
15
este echiunitar
a.
(x + 1) (y 4)
6. Sa se determine x N astfel nct numarul a = 5x + 5x+1 + 5x+2 + 5x+3 sa
aiba exact 48 divizori.
1 1
1
1
1
1
7. Sa se calculeze suma S = + +
+
+
+ +
.
2 6 12 20 30
360
3a + 16
8. Aflati numerele a N pentru care
N.
2a 5
9. Care este suma ultimelor trei cifre ale produsului 1 2 3 . . . 25?
10. S
a se afle cel mai mic num
ar natural de 2 cifre pentru care suma dintre p
atratul
si cubul lui este patrat perfect.
11. Asezati n ordine cresc
atoare numerele a = 250 , b = 247 + 224 , c = 245 + 244 +
23
22
48
23
+2 + 2 , d = 2 + 2 .
12. n pester
a erau dragoni rosii si dragoni verzi. Fiecare dragon rosu avea 6
capete, 8 picioare si 2 cozi. Fiecare dragon verde avea 8 capete, 6 picioare si 4 cozi.
n total dragonii aveau 44 cozi. Sunt, de asemenea, cu 6 picioare verzi mai putin
dect capete rosii. Cti dragoni rosii sunt n pester
a?
13. Dan spal
a o masin
a n 40 de minute, iar Ionut spal
a o masin
a n 2 ore. n ct
timp vor spala mpreuna 3 masini?
14. Dintre cei 101 de dalmatieni, 56 au o pat
a neagr
a pe urechea stng
a, 25 au
pat
a neagr
a pe urechea dreapt
a, iar 29 au ambele urechi albe. Cti dalmatieni au
pete negre pe ambele urechi?
15. Fie a = 2214 + 3143 si b = 3143 . Care dintre numere este mai mare?

Clasa a VI-a
7a 2b
2
b
=
, atunci este ..............................
5a + 4b
15
a
2. Numarul triunghiurilor din figura este ..............................
3. Diferenta dintre masurile suplementului si complementului aceluiasi unghi este ..............................
1
1
1
1
4. Rezultatul calculului
+
+
+ +
este .........................
13 35 57
2003 2005
0, 2 (4)
5. Doua unghiuri complementare au raportul masurilor lor egal cu
; atunci
0, 4 (2)
m
asura unghiului mai mare este ..............................

6. Rezultatul calculului ab0 ba : 99 este ..............................


7. n cte moduri putem aseza patru persoane ntr-un rnd?
8. Ultima cifr
a a num
arului 22004 + 32004 + + 92004 este ..............................
9. Avem la dispozitie timbre de 4000 lei si de 9000 lei. Pentru a expedia o
scrisoare sunt necesare timbre n valoare de 35000 lei. Numarul total de timbre
1. Dac
a

117

este ..............................
10. Suplementul unui unghi si complementul sau au masurile invers proportionale
cu 2 si 5. Suma masurilor lor este ..............................
11. Num
arul maxim de unghiuri n jurul unui punct cu m
asuri numere naturale
diferite este ..............................
12. Un produs se scumpeste cu 10% si apoi cu 20%. Acelasi produs se scumpeste
cu 20% si apoi cu 10%. n ce caz pretul final este mai mare?
13. Solutia ecuatiei 1, (1x) + 2, (2x) + + 9, (9x) = 50 este ..............................
14. Cte cifre de 0 are la sfrsit num
arul 1 2 3 4 . . . 2004?
15. Pe o tabla de sah 4 4 se trage o linie dreapta. Cel mai mare numar de
patratele 1 1 care pot fi taiate n doua parti este ..............................

Not
a (pentru clasele VII-XII ). Toate subiectele sunt obligatorii. Timp efectiv de lucru
3 ore. Pentru fiecare subiect se acord
a 7 puncte.

Clasa a VII-a
1. a) Demonstrati c
a dac
a m si n sunt numere naturale, atunci 25n 7m este
divizibil cu 3.
b) Determinati cel mai mic num
ar de forma |25n 7m 3m |, unde m si n sunt
numere naturale nenule.
Marius Ghergu, Slatina
2. Fie a, b dou
a numere reale avnd modulele cel putin 2. Demonstrati c
a
(a2 + 1)(b2 + 1) (a + b)(ab + 1) + 5. Cnd are loc egalitatea?
Marius Durea, Iasi
3. n triunghiul ABC se consider
a n
altimea [CM ], M AB, iar N este simetricul punctului M fata de BC. Paralela prin punctul N la CM intersecteaz
a BC
n P si AC n Q.
a) Demonstrati ca M Q AP daca si numai daca [AB] [AC].
b) Ar
atati cum pot fi obtinute pozitiile punctelor A, B, C atunci cnd cunoastem
doar pozitiile punctelor M , N , P .
Petru R
aducanu, Iasi
4. Se consider
a un triunghi ABC, n puncte distincte A1 , A2 , . . . , An pe latura
(BC), n puncte distincte B1 , B2 , . . . , Bn pe latura (AC) si n puncte distincte C1 ,
C2 , . . . , Cn pe latura (AB). Fie M multimea punctelor care se obtin la intersectia
a cel putin doua din segmentele (AAi ), (BBj ), (CCk ). Determinati numarul minim
si num
arul maxim de elemente pe care le poate avea multimea M .
Dan Brnzei, Iasi

Clasa a VIII-a
1. Determinati numerele ntregi a, b pentru care, oricare ar fi x real,
(2a a2 )x4 + (2a + 2b 4)x3 + (3b 3)x2 + (2b2 2b)x + b + 2 0.
Gheorghe Iurea, Iasi
118

2. Un zar este un cub de latur


a 1, pe fetele c
aruia sunt imprimate puncte ca n figura, astfel nct suma numerelor de puncte
de pe fetele opuse sa fie 7. Din opt zaruri alcatuim un cub de
latur
a 2.
a) Ce valori poate avea num
arul punctelor care sunt vizibile
pe fetele cubului de latura 2?
b) Este posibil sa asezam zarurile astfel nct oricare doua fete care sunt n contact
s
a aib
a un num
ar egal de puncte?
***

3. Fie ABCD o piramida n care AC = BC = 1 si AB = AD = BD = CD = 2.


Determinati distanta de la punctul A la planul (BCD).
***
4. Fie cubul ABCDA0 B 0 C 0 D0 de latura a si puncte M (AB), N (CC 0 ),
P (D0 A0 ), astfel ca AM = CN = D0 P = x.
a) Calculati M P .
b) Aratati ca triunghiul M N P are centrul de greutate pe segmentul [B 0 D].
Dan Brnzei

Clasa a IX-a
1. Fie n un numar natural si numerele reale a, b, c astfel nct an = a + b;
b = b + c si cn = c + a. S
a se arate c
a a = b = c.
Gheorghe Iurea, Iasi
2. Pe laturile [AB], [BC], [CD], [DA] ale unui patrulater convex ABCD se
MA
NB
PD
QA
considera punctele M , N , P , respectiv Q astfel nct
=
=
=
= k,
MB
NC
PC
QD
unde k 6= 1. Sa se arate ca S[ABCD] = 2S[M N P Q] daca si numai daca S[ABD] =
= S[BCD] .
Petre Asaftei, Iasi
3. Fie ABC un triunghi dreptunghic n A si D un punct apartinnd laturii
[BC]. Bisectoarele unghiurilor ADB si ADC intersecteaz
a laturile AB si AC n

si M N are m
punctele
M
s
i
N
.
S
a
se
arate
c
a
unghiul
dintre
dreptele
BC
asura

1 b

b daca si numai daca D este piciorul perpendicularei din A.


m(B) m(C)
2
Bogdan
Enescu, Buz
au
p
4. S
a se determine numerele reale x, x > 1 pentru care n [xn ]este ntreg, oricare
ar fi n 2. (Se noteaz
a cu [a] partea ntreag
a a num
arului real a).
Mihai Piticari, C-lung Moldovenesc, Dan Popescu, Suceava
n

Clasa a X-a
1. Fie A = {1, 2, 3, 4, 5}. Sa se determine numarul functiilor f : A A cu
proprietatea c
a nu exist
a numere distincte a, b, c A astfel nct f (a) = f (b) = f (c).
Adrian Zanoschi, Iasi
2. Se consider
a tetraedrul ABCD n care medianele din A n triunghiurile ABC,
ABD si ACD sunt perpendiculare doua cte doua. Sa se arate ca muchiile din A
sunt egale.
Dinu S
erb
anescu, Bucuresti
3. Se considera numerele reale x, y, z cu proprietatea ca cos x+cos y +cos z = 0 si
119

cos 3x + cos 3y + cos 3z = 0. S


a se demonstreze inegalitatea cos 2x cos 2y cos 2z 0.
Bogdan
Enescu,
au

Buz
4. Fie a 2 un numar natural. Consideram multimea A = { a, 3 a, 4 a, 5 a, . . .}.
a) S
a se arate c
a A nu contine o progresie geometric
a infinit
a neconstant
a.
b) S
a se arate c
a pentru orice n 3, exist
a n elemente din A care sunt n progresie
geometrica.
Bogdan Enescu, Buz
au

Clasa a XI-a
1. a) Fie (xn ) un sir de numere reale, cu proprietatea |xn+1 xn |

1
, n N.
2n

Ar
atati c
a sirul (xn ) este convergent.
b) S
a se construiasc
a un sir de numere reale (yn ), care s
a aib
a simultan proprietatile:
(i) lim (yn+1 yn ) = 0;
(ii) (yn ) este marginit;
(iii) (yn ) este divergent.
n
Eugen Popa, Iasi
2. Se da paralelogramul ABCD, cu laturi inegale. Vrful B se proiecteaza
pe AC n punctul E. Perpendiculara n E pe BD intersecteaza dreptele BC si
AB n punctele F , respectiv G. S
a se arate c
a EF = EG dac
a si numai dac
a
\ = 90 .
m(ABC)
Mircea Becheanu, Bucuresti
3. Fie matricele A, B M2 (Z) astfel nct AB = BA si det B = 1. S
a se arate
c
a dac
a det(A3 + B 3 ) = 1, atunci A2 = O.
Mircea Becheanu, Bucuresti
4. Fie f : R R o functie cu proprietatea lui Darboux. S
a se arate c
a dac
af
este cresc
atoare pe R \ Q, atunci f este continu
a.
Mihai Piticari, C-lung Moldovenesc

Clasa a XII-a
1. Fie p un numar prim, p > 3. Sa se arate ca ecuatia (x + y)1 = x1 + y 1 are
solutii n corpul Zp dac
a si numai dac
a 3 divide p 1.
Mihai Piticari, C-lung Moldovenesc
2. Sa se calculeze limita

X ln 1 + i ln 1 + j
n
n
p
lim
.
n
n4 + i2 + j 2
1i<jn

Gabriel Mrsanu si Andrei Nedelcu, Iasi


3. Fie n 3 un num
ar impar si A un inel comutativ cu 3n elemente. S
a se arate
c
a num
arul elementelor nilpotente ale lui A este cel mult n. (Elementul a A se
numeste nilpotent daca an = 0, pentru n > 0 convenabil).
***
4. S
a se arate c
a, pentru orice num
ar natural p,
Z 1
p+1
lim n
enx ln (1 + xp ) dx = p!.
n

Gheorghe Iurea, Iasi

120

Concursul de matematic
a Florica T. Cmpan
Etapa interjudetean
a, 8 mai 2004
Not
a. Toate subiectele sunt obligatorii. Timp de lucru: cl. a IV-a 90 de minute,
cl. V-VIII 2 ore.

Clasa a IV-a
1. Precizeaz
a regula de formare a numerelor urm
atoare si identific
a cifrele care
lipsesc n ultimul numar: 2798, 5783, 3574, 7862, 54 .
2. ntr-o urn
a sunt bile albe, galbene si rosii. Dac
a mp
artim num
arul bilelor albe
la numarul bilelor galbene obtinem ctul 2 si restul 2. mpartind numarul bilelor rosii
la suma celorlalte obtinem ctul 3 si restul 3. Aflati cte bile sunt n urn
a, stiind c
a
diferenta dintre num
arul bilelor rosii si dublul sumei celorlalte este 17.
3. n timpul unui campionat de sah, doi participanti care jucaser
a acelasi num
ar
de partide s-au mboln
avit si s-au retras, iar ceilalti au continuat turneul pn
a la
sfrsit. Este adevarat ca cei doi participanti au ajuns sa joace ntre ei, daca se stie ca
n total s-au jucat 23 de partide? (Turneul s-a jucat n sistemul "fiecare cu fiecare"
cte o singur
a partid
a.) Justificati r
aspunsul.

Clasa a V-a
1. Aflati vrstele tatalui, fiului si nepotului, stiind ca sunt exprimate prin trei numere prime, iar peste cinci ani vrstele lor vor fi exprimate prin trei numere naturale
p
atrate perfecte.
2. Determinati toate numerele de forma abbc, cifrele a, b, c fiind distincte, stiind
c
a sunt ndeplinite conditiile:
a) ba se divide cu 13; b) ab este num
ar prim; c) c este p
atratul unui num
ar natural.
Cte solutii are problema?
3. Pe trei jetoane asezate cu fata n jos sunt scrise trei numere naturale nenule
si distincte a caror suma este 13. Jetoanele sunt asezate n ordine crescatoare de
la stnga la dreapta. Ana ridic
a, prima, jetonul din stnga, apoi Dan pe cel din
dreapta, iar ultimul, S
tefan, pe cel din mijloc, declarnd fiecare, n aceast
a ordine,
ca nu are suficiente informatii pentru a descoperi celelalte numere. Dar tu, acum,
poti spune ce numar a vazut S
tefan?
Mihaela Cianga, Iasi

Clasa a VI-a
1. Se consider
a multimea A = {1, 2, 3, . . . , 2004}. Determinati:
a) probabilitatea ca, alegnd la ntmplare un element din A, acesta sa fie divizibil
cu 167.
b) probabilitatea ca, alegnd la ntmplare o submultime B 6= a lui A, produsul
tuturor elementelor multimii B s
a fie egal cu produsul tuturor elementelor multimii
A \ B.
Ioan Lungu, Vaslui
\ 90. Fie D (BC)
2. Se consider
a triunghiul ABC cu [AB] [AC], m(BAC)

\
astfel nct m(ADC) = 60 si fie punctul E astfel nct D (AE) si [AE] [CD].
\
Sa se determine m(EBD).
Mihai Gavrilut, Roman
121

3. Un pilot de avion parcurge o anumit


a distanta cu avionul si efectueaz
a urm
atorul calcul: aduna numerele naturale nenule ce reprezinta distanta (n km) cu viteza
(n km/h) si cu timpul (n h) si obtine 6008. Determinati ct timp a durat zborul.
Cristian Laz
ar, Iasi

Clasa a VII-a
1. S
a se arate c
a oricum am alege 51 de numere naturale distincte de la 1 la 100,
printre ele exista doua numere naturale distincte a si b astfel nct a | b.
2. Un casier de banca distrat, platindu-i un cec lui Lucian Georges, a ncurcat
euro cu centii (1 Euro = 100 centi), dndu-i acestuia euro n loc de centi si centi n
loc de euro. Dup
a ce si-a cump
arat o acadea de 5 centi, Lucian Georges a descoperit
c
a i-a mai r
amas exact o sum
a reprezentnd dublul sumei initiale de pe cec. Care
este suma scrisa pe cec?
C
at
alin Budeanu, Iasi
3. 26 de pisici sunt nchise ntr-un labirint care
are forma din figura alaturata (fiecare latura are
lungimea de 1 m). Dac
a dou
a pisici se afl
a la distanta mai mic
a de 1, 5 m se vor zgria! Ar
atati c
a
oricum am aseza cele 26 de pisici n labirint, macar
doua se vor zgria.
Monica Nedelcu, Iasi
1m

1m

Clasa a VIII-a
1. Fie tetraedrul ABCD. Vrfului A i asociem numarul natural n, iar vrfurilor
B, C, D le asociem num
arul 0. Numim "mutare" alegerea a dou
a vrfuri oarecare si
m
arirea numerelor asociate lor cu cte o unitate. S
a se arate c
a dup
a un num
ar finit
de "mutari" putem face ca fiecarui vrf sa-i fie asociat acelasi numar, daca si numai
daca n este numar par.
Gheorghe Iurea, Iasi
2. Pentru a R consideram functia fa : R R, fa (x) = ax + 2 a, x R si
fie Ma simetricul originii fata de graficul functiei fa .
a) Ar
atati c
a graficele functiilor fa trec printr-un punct fix, oricare ar fi a R.
b) Sa se arate ca
, oricare ar fi a si b R, lungimea segmentului Ma Mb este mai
mica sau egala cu 2 5.
Gabriel Popa, Iasi
3. Un corp gol n forma de tetraedru regulat este asezat cu o fata pe pamnt si
sufer
a dou
a r
asturn
ari instantanee, consecutive, pe alte dou
a fete ale sale. O bil
a neelastic
a, aflat
a initial n vrful ce nu atinge p
amntul, se misc
a sub actiunea atractiei
pamntului naintea primei rasturnari si pna dupa ultima rasturnare, imediat dupa
fiecare rasturnare plecnd spre alta fata din punctul n care a ajuns.
a) Ar
atati c
a bila cade de fiecare dat
a (nu se rostogoleste pe fete).
b)
Calcula
t
i
lungimea
parcursului
bilei,
dac
a latura tetraedrului are lungimea de

9 6
m.
32
Claudiu-
Stefan Popa, Iasi

122

Concursul Traian Lalescu


Editia a V-a, Iasi, 2004
Not
a. Fiecare subiect va fi notat cu cinci puncte. Timp de lucru: 2 ore.

1. Calculati: [62 + 8 (24 24 : 4 3)] : 5.


2. Aflati valoarea lui x din egalitatea: 1500 [(409 307) 4 + 315 : 15 x] : 3 =
= 1322.
3. Determinati cel mai mic num
ar natural n astfel nct: n+3n+5n+7n+9n >
> 2004.
4. Cte numere de trei cifre au suma cifrelor mai mare sau egala cu 25?
5. Pe tabla s-a scris de douazeci si trei de ori numarul 13 si de treisprezece
ori num
arul 23. Cte numere trebuie sterse de pe tabl
a pentru ca suma numerelor
r
amase s
a fie 464?
6. Gasiti cel mai mic numar natural care are suma cifrelor 102.
7. Multimea numerelor naturale diferite de zero se mparte n grupe astfel: (1),
(2, 3), (4, 5, 6), (7, 8, 9, 10), . . . . Cu ce num
ar ncepe grupa cu num
arul 51?
8. Suma anilor de nastere ai Raluc
ai si Elenei este 3961. S
tiind c
a la data de 8
mai 1998 Raluca era de sase ori mai mare dect Elena, cti ani are astazi Raluca?
9. De-a lungul unui gard sunt opt tufe de zmeura. Numarul fructelor de pe doua
tufe vecine difer
a cu 1. Num
arul fructelor de pe toate tufele poate fi:
a) 101; b) 213; c) 225; d) 229; e) alt num
ar.
10. ntr-o clas
a, elevii politicosi sunt de dou
a ori mai multi dect cei nepoliticosi.
Numarul fetelor politicoase si al baietilor nepoliticosi este de doua ori mai mare dect
cel al baietilor politicosi si al fetelor nepoliticoase, iar numarul fetelor politicoase este
egal cu num
arul tuturor b
aietilor. Cti b
aieti sunt n clas
a, dac
a aceasta are ntre 20
si 30 de elevi?
11. mparatul Rosu lasa mostenire celor patru fii herghelia de cai astfel: cel mai
mare ia o treime din herghelie, al doilea ia trei optimi din rest, al treilea trei cincimi
din noul rest, iar mezinul a primit cei 10 cai r
amasi. Cti cai are herghelia?
12. Pe distanta de 124 de metri s-au instalat 18 conducte de ap
a. S
tiind c
a n
depozit se gaseau 15 conducte de 6 m si 10 conducte de 8 m, determinati cti metri
masoara mpreuna conductele care au mai ramas n depozit.

Solutiile problemelor enuntate la p. 108.


1. Numerele cautate sunt 11 si 1, 1, caci 11 + 1, 1 = 11 1, 1.
2. Ursul se afla la Polul Nord. El are culoarea alba.
123

Concursul Adolf Haimovici, editia a VIII-a


pentru liceele economice, industriale si agricole

Faza interjudetean
a, Iasi, 8 - 9 mai, 2004
Not
a. Toate subiectele sunt obligatorii. Timp de lucru: 2 ore.

Clasa a IX-a
1. a) Se d
a graficul functiei f : R R, f (x) = x2 + ax + a, din figura 1. Ct
este a?
b) n figura 2 sunt reprezentate graficele a trei trinoame de gradul al doilea. Pot
s
a fie acestea urm
atoarele trinoame: ax2 + bx + c, cx2 + ax + b, bx2 + cx + a? Aceeasi
ntrebare pentru figura 3.
y

Figura 1.

Figura 2.

Figura 3.

c) Se consider
a trinoamele de gradul al doilea de forma x2 + px + q, p, q Z si
p + q = 30. Care dintre ele au r
ad
acini ntregi?
2. Se d
a triunghiul ABC n care AB = 3, AC = 4 si BC = 5.
a) Sa se calculeze aria triunghiului ABC.
b) Sa se afle raza cercului nscris si raza cercului circumscris triunghiului.
c) S
a se calculeze sin B, cos B, sin 2B, cos 2B.
d) S
a se arate c
a cos nB Q, n N.
e) Argumentati ca cos nB 6= 0 si ca exista o infinitate de numere naturale n cu
proprietatea ca cos nB > 0.
3. a) Sa se arate ca (n + 1)2 (n + 2)2 (n + 3)2 + (n + 4)2 = 4, n N.
b) S
a se arate c
a pentru orice k Z, exist
a n N si o alegere convenabil
a a
semnelor + si astfel nct k = 12 22 ... n2 .

Clasa a X-a

1. Fie z1 , z2 , z3 numere complexe astfel nct |z1 | = |z2 | = |z3 | = 2004 si


|z1 + z2 + z3 | = 2004. Atunci (z1 + z2 )(z1 + z3 )(z2 + z3 ) = 0.
2. Codul Morse, care a fost mult utilizat n comunicatii telegrafice, a utilizat
punctele si liniile pentru a codifica anumite caractere. De exemplu E se codifica
printr-un punct, T printr-o linie, zero prin 5 linii etc. Cte caractere se pot codifica
prin cel mult 5 semnale Morse (puncte sau linii)?
3. Se consider
a polinoamele f = 8X 3 6X 1, g = 4X 3 3X.
a) S
a se arate c
a f nu are r
ad
acini rationale.
b) Sa se afle ctul si restul mpartirii lui f la g.
124

c) Ar
atati c
a f cos
= 0.
9

d) Este cos irational? Argumentati.


9

e) Daca h Q[X] si h cos


= 0, sa se arate ca h .. f .
9

Clasa a XI-a
1. Se d
a graficul functiei
f : R R,

f (x) = ax4 x2 + bx + c.

Sa se afle semnele numerelor a, b si c.


2. Fie dreptele d1 si d2 perpendiculare n A. Un punct F asezat n interiorul unuia
din unghiurile formate de cele dou
a drepte, este situat la distantele 1 si respectiv 8
fata de acestea. S
a se determine lungimea minim
a a unui segment [BC] (B d1 ,
C d2 ) care trece prin punctul F .

a b c

3. Fie multimile M = c a b | a, b, c N si

b c a

K = n N | n = a3 + b3 + c3 3abc; a, b, c N .
a) S
a se arate c
a dac
a A, B M, atunci AB M.

b) Pentru orice m, n K, avem mn K.


a b c
c) Exist
a o matrice E M cu proprietatea c
a c a b = aI3 + bE + cE 2 ,
b c a
a, b, c N?
d) Fie A M, nesingular
a, cu d = det A. S
a se arate c
a

22004 1
2004

si
det . . . (A ) . . .
= d2
. . . (A ) . . . = d 3 A.
{z
}
|
{z
}
|
2004

2004

Clasa a XII-a

1. Pe multimea G = {a, b, c, d, e} operatia defineste


o structura de grup. Completati tabla grupului G.

a
b
c
d
e

b
d

e
2. Fie polinoamele f = 8X 3 6X 1 si g = 4X 3 3X.
b
a) Sa se arate ca f nu are radacini rationale.
si restul mp
b) S
a se calculeze
artirii lui f la g.
ctul

c) Ar
atati c
a f cos
= 0.
9

d) Este irational num


arul cos ?
9

= 0, sa se arate ca h .. f .
e) Daca h Q[X] si h cos
9
3. a) Calculati aria subgraficului functiei f : [1, e] R, f (x) = ln x.
b) Dac
a a, b, c sunt numere reale oarecare din intervalul [1, e] astfel nct a b c,
atunci (b a) ln a + (c a) ln b + (c b) ln c < 2.

125

Olimpiada Balcanic
a de Matematic
a (juniori), 2004
Editia a VIII-a, Novi Sad (Serbia si Muntenegru)
A. Problemele de concurs - enunturi si solutii
1. Pentru x, y R, nu simultan nule, s
a se demonstreze inegalitatea

x+y
2 2
p
.
x2 xy + y 2
x2 + y 2

2. Fie 4ABC isoscel (AC = BC), M mijlocul segmentului [AC], iar [CH]
n
altimea din C. Cercul ce trece prin B, C si M intersecteaz
a a doua oar
a dreapta
CH n Q. Sa se afle raza cercului circumscris 4ABC functie de m = CQ.

3. Fie x, y N astfel nct 3x + 4y si 4x + 3y s


a fie ambele p
atrate perfecte. S
a
se arate c
a numerele x si y sunt multipli de 7.
4. Se consider
a un poligon convex cu n 4 vrfuri. Descompunem arbitrar
poligonul n triunghiuri ale c
aror vrfuri sunt printre cele ale poligonului, astfel nct
oricare doua triunghiuri sa nu aiba puncte interioare comune. Coloram cu negru
triunghiurile ce au doua laturi care sunt si laturi ale poligonului, cu rosu triunghiurile
ce au exact o latur
a care este si latur
a a poligonului, celelalte triunghiuri l
asndu-le
albe. S
a se demonstreze c
a num
arul triunghiurilor negre este cu 2 mai mare dect
numarul triunghiurilor albe.
1. Observam ca cei doi numitori sunt ntotdeauna pozitivi. Daca x + y < 0,
inegalitatea este evident
a (si strict
a). Dac
a x + y > 0, se demonstreaz
a imediat c
a
p
x2 + y 2
x + y 2 (x2 + y 2 ), x2 xy + y 2
,
2
prin urmare
p

2 (x2 + y 2 )
x+y
2 2
x+y
p

.
x2 xy + y 2
x2 xy + y 2
x2 + y 2
x2 + y 2
2
Egalitate se atinge pentru x = y > 0.
C
2. Fie P si O centrele cercurilor circumscrise triunghiurilor BCM , respectiv ABC, K mijlocul lui
[CM ], iar {L} = CH P K. Evident c
a O CH si
L
K
cum OM si P K sunt perpendiculare pe AC, rezulta
P
M
ca OM k P K. Urmeaza ca LK este linie mijlocie n
O
4CM O, deci CL = LO.
[ ) = 90
Deoarece OP BC, avem ca m(LOP
Q
\
[ ) = m(CLK)
\ = 90
m(OCB).
Apoi, m(OLP
A
H
B
[
\ OCA,
[ deoarece n
m(OCA).
ns
a OCB
altimea
CH este bisectoare n 4ABC isoscel, deci P[
OL P[
LO si P L = P O. Observam ca
4P CQ este isoscel cu P C = P Q, prin urmare va rezulta c
a CL = OQ.
2
2
n concluzie, CL = LO = OQ, deci R = OC = CQ = m.
3
3

126

.
3. Fie m2 = 3x + 4y, n2 = 4x + 3y; atunci m2 + n2 = 7 (x + y), deci m2 + n2 .. 7.
Un p
atrat perfect d
a la mp
artirea prin 7 unul din resturile 0, 1, 2, 4 si atunci, pentru
ca o suma de patrate perfecte sa se divida cu 7, trebuie ca fiecare n parte sa se divida
.
.
.
.
cu 7. ns
a, dac
a m2 .. 7, atunci m2 .. 72 si analog n2 .. 72 , adic
a 7 (x + y) = m2 + n2 .. 49,
.
.
.
deci x + y .. 7. Urmeaza ca x = (4x + 3y) 3 (x + y) .. 7, y = (3x + 4y) 3 (x + y) .. 7.

4. Notam cu x, y, z numarul triunghiurilor colorate n negru, rosu, respectiv alb;


atunci x + y + z = n 2, acesta fiind num
arul triunghiurilor din descompunerea
poligonului. Deoarece fiecare latur
a a poligonului este latur
a a exact unui triunghi
din descompunere, avem ca 2x + y = n. Combinnd cele doua relatii, obtinem ca
x z = 2, ceea ce trebuia sa demonstram.

B. Probleme aflate n atentia juriului - enunturi


1. Fie a, b, p, q N cu p, q 3, iar a, b relativ prime si de parit
ati diferite. S
a
se arate ca numarul N = 2ap b 2abq nu poate fi patrat perfect.

2. Gasiti toate numerele naturale A care se scriu cu patru cifre n baza 10, cu
2
1
asturnatul lui A).
proprietatea c
a A + 2000 = A. (Cu A am notat r
3
3
3. G
asiti toate numerele naturale n 3 cu proprietatea c
a n divide (n 2)!.
4. Pentru a, b, c [1, +), demonstrati inegalitatea

1 1 1
(1 + abc)
+ +
3 + a + b + c.
a b
c
5. Pentru x, y, z R, demonstrati inegalitatea

y2 z2
z 2 x2
x2 y 2
+
+
(x + y + z)2 .
2x2 + 1 2y 2 + 1 2z 2 + 1

a
< b < 2a, sa se arate ca
b

2
2ab a2
2ab b2
1 a b
+
1+
.

7ab 3b2 2a2


7ab 3a2 2b2
4 b a
7. Dou
a cercuri C1 si C2 sunt secante n A si B. Un cerc C cu centrul n A taie
C1 n M si P , iar pe C2 n N si Q, astfel nct N si Q sunt situate de o parte si de
\
\
alta a dreptei M P , iar AB > AM . S
a se demonstreze c
aM
BQ N
BP .
6. Daca 0 <

8. Fie E, F dou
a puncte distincte n interiorul paralelogramului ABCD. Determinati numarul maxim posibil de triunghiuri avnd aceeasi arie, cu vrfurile n trei
dintre punctele A, B, C, D, E, F .
9. Fie 4ABC nscris n cercul C. Cercurile C1 , C2 , C3 sunt tangente interior
cercului C n punctele A1 , B1 , respectiv C1 si tangente laturilor [BC], [CA], [AB] n
punctele A2 , B2 , respectiv C2 , astfel nct A, A1 sunt de o parte si de alta a lui BC
etc. Dreptele A1 A2 , B1 B2 si C1 C2 intersecteaz
a a doua oar
a cercul C n punctele A0 ,
0
0
0
0
\
B , respectiv C . Dac
a {M } = BB CC , demonstrati c
a m(M
AA0 ) = 90 .
b = 90 si punctele D [AC], E [BC]. Spre interiorul
10. Fie 4ABC cu m(C)
triunghiului construim semicercurile C1 , C2 , C3 , C4 de diametre [AC], [BC], [CD],
127

respectiv [CE] si fie {C, K} = C1 C2 , {C, M } = C3 C4 , {C, L} = C2 C3 , {C, N } =


C1 C4 . Aratati ca punctele K, L, M , N sunt conciclice.
11. Pe o tabl
a dreptunghiular
a cu m linii si n coloane, n fiecare c
asuta se afl
a
scris semnul "". Putem efectua urm
atoarele operatii:
(i) schimbarea tuturor semnelor de pe o linie (din "+" n "", iar din "" n "+");
(ii) schimbarea tuturor semnelor de pe o coloana.
a) Dac
a m = n = 100, ar
atati c
a nu putem obtine 2004 semne "+" utiliznd de
un num
ar finit de ori operatiile descrise;
b) Daca m = 100, gasiti cea mai mica valoare a lui n > 100 pentru care putem
obtine 2004 semne "+".
(continuarea tabelului din p. 113)
148.
149.
150.
151.
152.
153.
154.
155.
156.
157.
158.
159.
160.
161.
162.
163.
164.
165.
166.
167.
168.
169.
170.
171.
172.
173.
174.
175.
176.
177.

PRECUPANU Codrin
DUREA Magdalena
BURGHELEA Diana
FARCAS Marius
BAGHIU Ciprian
ION Elena
BUCESCU Dominic
PASA Narcisa

SUFITCHI
Viorica
POPA Gabriela
RUSU Virginia
ALESCU

DASC
Diana
GHERGHELAS Liliana
Olimpia
FERENT
LEONTIES Rodica
PANAINTE Ecaterina Bronia
POPA Dumitru

CARBUNE
Ioan
BRESUG Constantin

CAILEANU
Sorin
DOCA Laurentia
Cristina
BRNZILA
SU Dan
PLAE
Valentin
CONTU
OLENIUC Mariana
Amalia - Patricia
ROMILA

GRADINARU Daniela
ARBONE Ion
NECHITA Remus
BUJOR Lorena

Colegiul "C. Negruzzi", Iasi


S
coala "T. Maiorescu", Iasi
S
coala "T. Maiorescu", Iasi
S
coala "T. Maiorescu", Iasi
Liceul "D. Cantemir", Iasi
Liceul "G. Ibr
aileanu", Iasi
S
coala "I. Creang
a", Iasi
Liceul de arta, Iasi
S
coala gen. nr. 26, Iasi
S
coala gen. nr. 43, Iasi
Liceul "M. Costin", Iasi
Liceul "M. Eminescu", Iasi
Gr. sc. "Victoria", Iasi
Gr. sc. "Victoria", Iasi
Liceul "Al. I. Cuza", Iasi
S
coala "Gh. I. Br
atianu", Iasi
S
coala "St. Brsanescu", Iasi
Liceul "I. Neculce", Tg. Frumos
Liceul "I. Neculce", Tg. Frumos
Liceul "I. Neculce", Tg. Frumos
Liceul "I. Neculce", Tg. Frumos
Liceul "D. Cantemir", Iasi
S
coala normala "V. Lupu", Iasi
S
coala "I. Cantacuzino", Pascani
Gr. sc. "D. Mangeron", Iasi
S
coala normal
a "V. Lupu", Iasi
Gr. sc. "A. Saligny", Iasi
informatician
Liceul "M. Costin", Iasi
S
coala "I. Teodoreanu", Iasi
(continuare la p. 165)
128

Cercul de matematic
a Leonard Euler
organizat la Universitatea Humboldt, Berlin
Probleme pentru clasa a VIII-a
Holger STEPHAN 1

Not
a. Rubrica "Corespondente" are ca scop informarea elevilor si profesorilor din
tara noastr
a cu privire la activitatea de performanta din alte colturi ale lumii. Pentru
orice "corespondenta", solutiile problemelor vor fi publicate n num
arul urm
ator al revistei.
Materialul de fata a fost obtinut prin str
adania d-lui Dan Tiba, cercet
ator principal I,
Institutul de Matematic
a al Academiei Romne.

1. Patru numere adunate doua cte doua dau sumele 4, 7, 9, 14, 16, 19. Care
sunt cele patru numere?
2. Demonstrati c
a prin "rotirea c
atre dreapta" a unui num
ar de 8 cifre, divizibil
cu 73, se obtine tot un num
ar divizibil cu 73. (Se spune c
a un num
ar natural este
"rotit catre dreapta", daca ultima cifra este mutata n fata primei cifre; exemplu:
1234 4123.)
3. Aflati cifrele necunoscute x, y, z din egalitatea
20 058 473 11! = x00yz0055046400.
4. Un num
ar x format din cinci cifre diferite si nenule este divizibil cu 9. Ar
atati
c
a suma tuturor numerelor de cinci cifre distincte ce se pot forma cu aceste cinci cifre
(inclusiv x) este divizibila cu 2399976.
5. Gasiti toate perechile de numere ntregi x si y care sunt solutii ale ecuatiei
diofantice 2x2 + 7xy + 3y 2 = 228.
6. G
asiti toate perechile de numere ntregi x si y care sunt solutii ale ecuatiei
diofantice 2x2 + 3y 2 = 77.
7. Consider
am num
arul natural n, 1000 n < 5000. Form
am num
arul (de 12
sau 13 cifre) obtinut scriind n ordine cifrele lui 3n, 2n si respectiv n. Aratati ca
acest numar este divizibil cu 28 + 1.
8. S
ase numere prime 7 < p1 < p2 < p3 < p4 < p5 < p6 formeaz
a un "sextet
de numere prime", dac
a p2 , p3 si p4 , p5 sunt numere prime gemene (adic
a p3 p2 =
= p5 p4 = 2), iar p2 p1 = p4 p3 = p6 p5 = 4. Demonstrati c
a suma lor este
divizibila cu 630.
9. Este posibil ca suma a sapte patrate perfecte succesive sa fie un patrat perfect?

Cercet
ator dr., Institutul Weierstrass, Berlin (e-mail: stephan@wias-berlin.de)

129

Solutiile problemelor propuse n nr. 2 / 2003


Clasele primare
P.54. Calculati a si b daca 46 a = 36 + a si b 3 = 17 b.
( Clasa I )
nv. Doinita Spnu, Iasi
Solutie. 46 a = 36 + a se scrie 36 + 10 a = 36 + a, de unde 10 a = a si
a + a = 10, deci a = 5. Analog, b 3 = 17 b se scrie b = 3 + 17 b, de unde
b + b = 20, deci b = 10.
P.55. n cte moduri pot fi aranjate n linie dreapta 9 mingi rosii si una galbena?
( Clasa I )
Georgiana Ciobanu, elev
a, Iasi
Solutie. Toate mingile ocup
a 10 locuri. Mingea galben
a poate ocupa fiecare loc
din cele 10. Sunt 10 moduri.
P56. Cu cinci ani n urma, suma vrstelor a trei copii era de 11 ani. Care va fi
suma vrstelor acelorasi copii peste 6 ani?
( Clasa a II-a)
nv. Rodica Rotaru, Brlad
Solutie. In prezent suma vrstelor copiilor este 11 + 3 5 = 26 ani, iar peste 6
ani aceasta va fi 26 + 3 6 = 44 ani.
P.57. n cte moduri pot fi mpartiti 8 baieti n doua echipe de cte 4 jucatori,
daca Petru vrea sa fie n echipa cu Mihai si Dan, dar nu vrea sa fie cu Avram?
( Clasa a II-a)
Adina Dohotaru, elev
a, Iasi
Solutie. n gruparea Petru, Mihai, Dan mai trebuie un singur b
aiat. Acesta
poate fi luat din restul b
aietilor, cu exceptia lui Avram. Al patrulea elev din echip
a
poate fi ales n patru moduri.
P. 58. Sa se arate ca suma 1 + 4 + 7 + . . . + 100 mpartita la 3 da restul 1.
( Clasa a III-a)
Alexandru - Gabriel Tudorache, elev, Iasi
Solutie. Suma se scrie 1 + (3 1 + 1) + (3 2 + 1) + . . . + (3 33 + 1). Restul
mp
artirii la 3 este acelasi cu restul mp
artirii sumei |1 + 1 + 1{z+ . . . + 1} la 3, care
34 ori
este 1.
P.59. Fie a si b doua numere consecutive. Suma acestor numere mpreuna cu
numerele obtinute marind cu 12 fiecare dintre vecinii lor este 939. Care sunt cele
doua numere?
( Clasa a III-a)
nv. Maria Racu, Iasi
a
Solutie. Consider
am b = a + 1. Folosim metoda figua
11
rativ
a. Primul num
ar este
13
a
[939 (11 + 13 + 1 + 1 + 11 + 1 + 13)] : 6 = 888 : 6 = 148.

Al doilea numar este 148 + 1 = 149.

939
11
13

P.60. Din 16 bile, una este mai grea dect celelalte 15, care au mase egale. Care
este cel mai mic numar de cntariri prin care se poate stabili bila mai grea?
( Clasa a III-a)
Carmen Ciolacu, elev
a, Iasi
Solutie. Asez
am cte 8 bile pe fiecare taler. Dup
a prima cnt
arire se determin
a
grupul de 8 bile care contine bila mai grea. Din acestea asez
am cte 3 pe fiecare
taler. Daca balanta este n echilibru, atunci bila mai grea se afla n perechea ramasa.
130

Printr-o nou
a cnt
arire se afl
a bila mai grea. Dac
a balanta nu este n echilibru,
atunci bila mai grea se afla ntr-o grupare de 3 bile. Aseznd cte o bila pe fiecare
taler se determina bila mai grea. Numarul minim de cntariri este 3.
P.61. Suma a doua numere este un numar de doua cifre al caror produs este 3.
Diferenta dintre cele doua numere este 7. Care sunt cele doua numere?
( Clasa a IV-a)
nv. Maria Racu, Iasi
Solutie. Suma celor dou
a numere poate fi 13 sau 31. Cum diferenta numerelor
este 7, n primul caz numerele sunt 3 si 10, iar n al doilea numerele sunt 12 si 19.
P.62. Doua ceasuri au nceput sa functioneze la aceeasi ora. Se constata ca la
fiecare 30 minute (fata de ora exacta) unul ramne n urma cu un minut iar celalalt
avanseaza cu un minut. La un moment dat orele indicate de aceste ceasuri sunt: 18
h 36 min si 19 h 24 min. La ce ora au nceput sa functioneze?
( Clasa a IV-a)
Felicia Amih
aiesei, elev
a, Iasi
Solutie. Cele dou
a ceasuri se abat cu acelasi num
ar de minute fata de ora exact
a,
unul prin lipsa iar celalalt prin adaos. n momentul citirii abaterea este (19 h 24 min
18 h 36 min) : 2 = 48 min : 2 = 24 min. Ceasurile au fost citite la ora 19 h 24 min
24 min = 19 h. Num
arul de ore n care ceasurile au functionat este 24 : 2 = 12.
Ceasurile au nceput s
a functioneze la ora 19 12 = 7.
P.63. Alege un numar format din trei cifre. Scrie la dreapta lui un numar format
din doua cifre. Scoate din numarul format de 99 ori numarul format din trei cifre.
Din rezultat scoate diferenta dintre numarul de trei cifre si numarul de doua cifre si
scrie rezultatul. Eu ti ghicesc numarul format din doua cifre. Cum se explica acest
lucru?
( Clasa a IV-a)
Prof. Petru Asaftei, Iasi
Solutie. Fie abc numarul de trei cifre si xy numarul de doua cifre. Avem abcxy =
= abc00 + xy = 100 abc + xy, 100 abc + xy 99 abc = abc + xy. n continuare
folosim metoda figurativ
a.

n urma efectu
arii operatiilor indicate n problem
a se obtine dublul num
arului de
doua cifre.

Clasa a V-a
V.41. Fie a numar natural compus astfel nct daca p | a, cu p prim, atunci
p + 1 | a. Sa se arate ca 12 | a si sa se afle cel mai mare numar a de trei cifre.
Ciprian Baghiu, Iasi
Solutie. Fie p un divizor prim al lui a, conform enuntului. Atunci p + 1 | a, deci
p (p + 1) | a si, cum p (p + 1) este num
ar par, deducem c
a 2 | a. Din 2 | a rezult
a
3 | a si apoi 4 | a; prin urmare 12 | a.
Cel mai mare num
ar de trei cifre divizibil cu 12 este 996 = 22 3 83; cum 84 - 996,
deducem ca 996 nu este solutie. La fel 984 nu este solutie. Numarul 972 = 22 35
verific
a cerintele problemei.
V.42. Se dau numerele xy, ab scrise n baza 10 astfel nct xy divide ab. Sa se
131

arate ca x = y daca si numai daca a = b.


Ioan S
ac
aleanu, Hrl
au
Solutie. Fie x = y. Atunci din xx = 11x | ab rezulta ca 11 | ab, deci ab {11, 22,
33, . . . , 99}, adic
a a = b.
Fie a = b. Atunci xy | 11a si, cum a este cifr
a nenul
a, deducem c
a xy se divide la
11. Ca mai sus, avem x = y.
V.43. Sa se afle cifrele a si b stiind ca a b = cd si ab = dc.
Romanta Ghita
si Ioan Ghita
, Blaj
Solutie. Evident, a {2, 3, . . . , 9}. Daca a = 2, atunci b 5 si cum 2 5 = 10 si
25 = 32, 2 6 = 12 si 26 = 64, iar 27 > 100 rezult
a c
a nu avem solutii. Dac
a a = 3,
urmeaz
a b 4 si din 3 4 = 12 si 34 = 81, iar 35 > 100 deducem c
a nici n acest caz
nu avem solutii. Considernd si verificnd toate posibilitatile, obtinem solutia a = 9
si b = 2.
V.44. Sa se afle x, y, z Q+ pentru care xn = yz, y n = xz, z n = xy, cu n N.
N. N. Hrtan, Iasi
Solutie. Avem xn+1 = y n+1 = z n+1 = xyz, deci, n mod necesar, x = y = z.
Conditiile initiale se reduc la xn = x2 , prin urmare, x = y = z = 1 daca n N \ {2}
si x = y = z Q+ daca n = 2.
V.45. Se dau sase urne, unele continnd bile. Fie operatia: se aleg trei urne si
se pune cte o bila n fiecare dintre ele.
a) Compozitia urnelor fiind 0, 0, 4, 6, 6, 8, sa se indice o succesiune de operatii n
urma carora toate urnele sa contina acelasi numar de bile.
b) Compozitia urnelor fiind 0, 1, 2, 3, 4, 4, sa se arate ca nu exista o succesiune de
operatii n urma carora toate urnele sa contina acelasi numar de bile.
Gheorghe Iurea, Iasi
Solutie. a) O succesiune de operatii ce atinge scopul este (0, 0, 4, 6, 6, 8)
(1, 1, 5, 6, 6, 8) (2, 2, 6, 6, 6, 8) (3, 3, 7, 6, 6, 8) (4, 4, 8, 6, 6, 8) (5, 5, 8, 7, 6, 8)
(6, 6, 8, 8, 6, 8) (7, 7, 8, 8, 7, 8) (8, 8, 8, 8, 8, 8).
b) Presupunem c
a dup
a n operatii urnele contin acelasi num
ar m de bile, rezult
a
ca numarul total de bile din urne este 6m = 3n + 14, imposibil.

Clasa a VI-a
VI.41. Pe opt cartonase sunt nscrise cte unul din numerele 1, 2, 22 , 23 , 3,
3 , 33 , 34 . Daca P (k) este probabilitatea ca, extragnd doua cartonase, numerele
obtinute sa aiba n total k divizori distincti, sa se rezolve inecuatia P (k) 1/7.
Dumitru Dominic Bucescu, Iasi
Solutie. Considernd cele 28 de posibilitati de extragere a cartonaselor si numa2
rnd divizorii distincti ai numerelor extrase, obtinem: P (1) = 0; P (2) =
;
28
5
8
7
3
2
1
si P (k) = 0,
P (3) =
; P (4) = ; P (5) = ; P (6) = ; P (7) = ; P (8) =
28
28
28
28
28
28
1
k 9. Prin urmare, multimea solutiilor inecuatiei P (k) este S = {3, 4, 5}.
7
VI.42. Fie x, y, z N pentru care 84x + 91y + 98z = 2002. Sa se afle valoarea
maxima a sumei x + y + z.
Adrian Zanoschi, Iasi
2

132

Solutie. Relatia din enunt este echivalent


a cu 12x + 13y + 14z = 286 sau
12 (x + y + z) = 286 (y + 2z). De aici, rezulta ca x + y + z este maxim daca
y + 2z este minim si 12 divide pe 286 (y + 2z). Obtinem x + y + z maxim
pentru y + 2z = 10. Valoarea maxim
a a expresiei este 23 si se obtine pentru
(x, y, z) {(18, 0, 5) ; (17, 2, 4) ; (16, 4, 3) ; (15, 6, 2) ; (14, 8, 1) ; (13, 10, 0)}.
Not
a. D-l Titu Zvonaru, Comanesti (Bacau), stabileste valoarea minima a
sumei x + y + z. ntr-adev
ar, dac
a x + y + z 19, atunci y + 2z 57 si am avea
12 (x + y + z) + y + 2z 12 19 + 57 = 285; prin urmare, x + y + z 20. Dac
a
x + y + z = 20, atunci y + 2z = 46. Pentru y < 15, z < 15, avem y + 2z < 45. Pentru
16 y 20, avem ca z 4, deci y + 2z 28. Pentru 16 z 20, avem y 4, deci
y + 2z 44. Rezult
a c
a valoarea minim
a a sumei x + y + z este 21, care se obtine,
de exemplu, pentru x = 2, y = 4, z = 15.
VI.43. Fie {a1 , a2 , . . . , an } Z pentru care k {1, 2, . . . , n}, i, j {1, 2, . . . , n},
i 6= j, astfel nct ak = ai + aj . Sa se arate ca n 6.
Petru Asaftei, Iasi
Solutie. Putem presupune c
a a1 < a2 < < an . Considernd num
arul an ,
exista i, j {1, 2, . . . , n}, i 6= j, astfel ca an = ai + aj . Daca ai < 0, aj < 0, atunci
an < ai , imposibil. La fel, ai < 0, aj > 0 implica an < aj , imposibil. Deci ai si aj
sunt pozitive si atunci multimea contine cel putin trei numere pozitive. Considernd
num
arul a1 deducem c
a multimea contine cel putin trei numere negative, prin urmare
n 6.
Un exemplu de multime cu 6 elemente este {3, 2, 1, 1, 2, 3} iar una cu n 6
elemente este {3, 2, 1, 1, 2, 3, . . . , n 3}.
VI.44. Fie ABCD un paralelogram si M AB, N AD triunghiuri echilaterale construite n exteriorul acestuia. Demonstrati ca [M N ] [BD] daca si numai daca
N D k M B.
Ciprian Baghiu, Iasi
Solutie. Presupunem [M N ] [BD]. Din conD
C
gruenta triunghiurilor BAD si M AN deducem ca
360 120
\ = m(M
\
m(BAD)
AN ) =
= 120 , deci N
2
B
\ ) = 180 , adic
A
a punctele B, A, N sunt colim(BAN
\
\
niare. Cum M
BN DN
B, rezulta M B k N D.
\
Presupunem M B k N D. Ca urmare, m(ABD)+

\
\
+m(ADB) = 180 120 = 60 , deci m(BAD) =
\
= 120 ; totodata m(M
AN ) = 120 si atunci triM
unghiurile M AN si BAD sunt congruente, de unde rezult
a c
a [BD] [M N ].
VI.45. Fie E, F picioarele naltimilor din B si C ale triunghiului ascutitunghic ABC. Daca P , N sunt mijloacele laturilor [AB], respectiv
[AC], iar {Q} =


b
\
= P E F N , sa se arate ca m(P QF ) = 180 3 m(A). (n legatura cu Q1086
din Parabola, nr. 3/2000 )
Titu Zvonaru, Bucuresti
133

Solutie. Fie Q exterior triunghiului ABC. Din


A
4AEB dreptunghic n E, cu (EP ) mediana, deducem
Q
[ P[
E
c
a triunghiul AEP este isoscel cu AEP
AE; deci
P
b La fel, din triunghiul dreptunghic
\
m(QP
F ) = 2m(A).
N
F
\
\
AF C cu mediana (F N ) deducem N
FA F
AN . Din
b S
\
4P QF rezult
a c
a m(P
QF ) = 180 3m(A).
a remarC
B
b
c
am c
a punctul Q este exterior dac
a m(A) < 60 . Cazul
b > 60 , se
n care Q este interior triunghiului ABC, ce corespunde situatiei m(A)

b = 60 , dreptele P E si F N sunt
trateaz
a folosind aceleasi argumente. Dac
a m(A)

\
paralele si putem considera ca m(P QF ) = 0 .

Clasa a VII-a

a
b
+
= 1.
b+1 a+1
Alexandru Negrescu, elev, Botosani
Not
a. Mai multe solutii ale acestei probleme sunt date n articolul prezent n
acest num
ar la p. 109.

VII.42. Sa se arate ca a2 + 1 b2 + 1 c2 + 1 (|a| + |b|) (|b| + |c|) (|c| + |a|),


a, b, c R.
Dorin M
arghidanu, Corabia
Solutie.
Notnd
|a|
=
x,
|b|
=
y,
|c|
=
z,
x,
y,
z

R
,
inegalit
atea este echiva+

lent
a cu x2 + 1 y 2 + 1 z 2 + 1 (x + y) (y + z) (z + x), x, y, z R+ .Avem
p
2

2
x + 1 y 2 + 1 = x2 y 2 +x2 +y 2 +1 x2 +y 2 +2xy = (x + y) , deci (x2 + 1) (y 2 + 1)
p
p
x + y. La fel, au loc:
(x2 + 1) (z 2 + 1) x + z si (y 2 + 1) (z 2 + 1) y + z.
Prin nmultirea ultimelor trei relatii se obtine inegalitatea dorit
a.
Egalitate se obtine pentru xy = 1, yz = 1, zx = 1, deci pentru x = y = z = 1 sau
|a| = |b| = |c| = 1.
VII.41. Rezolvati n N2 ecuatia

VII.43. Pentru n N, notam cu s (n) numarul de reprezentari distincte ale


lui n ca suma de doua numere naturale (n = a + b si n = b + a constituie aceeasi
reprezentare). Sa se arate ca:
n
hni hn + 1i
X
1
a) s (m + n) = s (m) + s (n) [1 + (1)mn ]; b)
s (k) =

.
2
2
2
k=0
Petru Minut, Iasi
hni
Solutie. Ar
at
am c
a s (n) =
+ 1. ntr-adev
ar, pentru n par, n = 2k, avem:
2
hni
+ 1. Pentru
n = 0 + n = 1 + (n 1) = . . . = k + k, deci s (n) = k + 1 =
2
n impar, n = 2k
h n+i 1, avem: n = 0 + n = 1 + (n 1) = . . . = k + (k + 1), deci
s (n) = k + 1 =
+ 1.
2
a) Relatia se demonstreaza analiznd paritatea numerelor m si n.
b) Daca n = 2k + 1, avem: s (0) + s (1) + . . . + s (n) = 1 + 1 + 2 + 2 + . . . +
hni hn + 1i
+k + k = k (k + 1) =

. Dac
a n = 2k, avem s (0) + s (1) + . . . + s (n) =
2
2
hni hn + 1i
= 2 (1 + 2 + . . . + (k 1)) + k = k2 =

.
2
2
134

VII.44. Fie [AB] diametru al cercului C de centru O; N, M C astfel nct


\
\
m(AON ) = 36 , iar [OM este bisectoare pentru N
OB. Daca T este simetricul lui
O fata de M N , sa se arate ca proiectia lui T pe AB este mijlocul lui [AO].
Valentina Blendea, Iasi
\
Solutie. n triunghiul T N O isoscel, m(T
N O) = 180
T
1 \
\
2m(N
OT ) = 180 m(N
OB) = 108 . n triunghiul
2
M
1

\
isoscel OAN , m(AN O) =
(180 36 ) = 72 , deci
N
2

\
\
m(T
N O) + m(AN
O) = 180 , prin urmare T , N , A sunt
coliniare. Cum m(T[
AO) = m(T[
OA) = 72 , triunghiul T AO
O
B
A
este isoscel si concluzia este imediat
a.
VII.45. Fie 4ABC echilateral, iar P (BC). Notam cu D, E simetricele lui P
fata de AC, respectiv AB. Sa se arate ca dreptele AP , BD si CE sunt concurente.
Constantin Cocea si Julieta Grigoras, Iasi
Solutie. Fie {M } = BD AC, {N } = CE AB;
A
AB = a, BP = x, P C = a x. Din asem
anarea triCM
CD
ax
D
unghiurilor DM C si BM A g
asim
=
=
,
MA
AB
a
M
iar din asemanarea triunghiurilor BEN si ACN gasim E N
BN
x
BE
=
= .
AC
AN
a
B
P
C
BP CM AN
n triunghiul ABC avem

= 1; conform reciprocei teoremei lui


P C MA NB
Ceva, dreptele AP , BM , CN sunt concurente.

Clasa a VIII-a
VIII.41. Fie f1 , f2 , f3 functii liniare ale caror grafice sunt drepte concurente
doua cte doua. Cele trei drepte sunt concurente daca si numai daca exista unic
R si exista u 6= v R astfel ca
f1 (u)
f2 (u)
f3 (u)
=
=
,
f1 (v)
f2 (v)
f3 (v)

cu fi (v) 6= , i {1, 2, 3} .

Claudiu-
Stefan Popa, Iasi
Solutie. Fie fi : R R, fi (x) = ai x + bi , i = 1, 3. Graficele functiilor sunt
concurente doua cte doua daca si numai daca a1 , a2 , a3 sunt distincte.
b2
b1
=
=
Prin proportii derivate relatia din enunt este echivalent
a cu
a1
a2
b3
=
, cu conventia c
a, dac
a unul dintre numerele ai este zero, atunci si bi
a3
este zero.
Graficele functiilor fi sunt concurente dac
a si numai dac
a exist
a (, ), -unic,
astfel nct fi () = , i = 1, 3, adic
a a1 + b1 = a2 + b2 = a3 + b3 = sau nc
a
b2
b3
b1
=
=
, ceea ce ncheie rezolvarea problemei.
=
a1
a2
a3
VIII.42. Fie x, y, z (0, ). Sa se arate ca
135

yz + zx
xy + xz
p
p
+
+

yz + zx + x2 + y 2 + z 2 yz zx
xy + xz + x2 + y 2 + z 2 xy xz

xy + yz
p
+
2.

xy + yz + x2 + y 2 + z 2 xy yz

Lucian Tutescu, Craiova


Solutie. Avem x2 +
y 2 + z 2
xy yz zx 0 deci x2+ y 2 +z 2 yz zx xy
yz + zx
yz + zx
p

si atunci

. Sumnd

2
2
2
yz + zx + xy
yz + zx + x + y + z yz zx
aceasta cu nc
a dou
a inegalit
ati asem
an
atoare, obtinem concluzia problemei.

VIII.43. Daca un triunghi dreptunghic are laturile numere naturale, iar suma
catetelor este patrat perfect, atunci suma cuburilor catetelor este suma de doua patrate perfecte.
Andrei Nedelcu, Iasi
Solutie. Fie (a, b, c) un triplet de numere naturale nenule care satisfac relatia
a2 + b2 = c2 . Urmeaz
a c
a toate trei sunt pare sau numai unul este par; deci
cb+a c+ba

,
N .
2
2

2
2
1
1
Avem a2 + b2 ab =
(c b + a) +
(c + b a) . Deoarece a + b = p2 si
2
2
hp
i2 h p
i2
2

3
3
2
3
a +b = (a + b) a + b ab , rezulta ca a +b3 = (c b + a) + (c + b a) .
2
2
VIII.44. Pe laturile [AB], [CD], [BC], [AD], [AC] si [BD] ale tetraedruBP
AQ
lui ABCD se iau respectiv punctele M , N , P , Q, R, S astfel ca
=
,
BC
AD
DN AR
DS
AM
=
,
=
. Notam cu V1 , V2 , V3 , V4 , V respectiv volumele tetraedrelor
AB
DC AC
BD
AM RQ, BP M S, CP N R, DN QS si ABCD. Sa se arate ca 212 V1 V2 V3 V4 V 4 .
Viorel Cornea si Dan S
tefan Marinescu, Hunedoara
V1
V2
Solutie. Notnd cele trei rapoarte cu , , , avem:
= ,
= (1 )
V
V
V3
V4
(1 ),
= (1 ) (1 ) (1 ),
= (1 ) . Atunci
V
V
2 2 2
1
1
1
1
V1 V2 V3 V4
2 2
2 2
2
2
=

(1

(1

(1

= 12 ,
4
V
4
4
4
2

1
, x [0, 1].
4
VIII.45. Fie A1 , A2 , . . . , Ak puncte pe un cerc C. Sa se determine o conditie
necesara si suficienta pentru a putea nscrie n C un poligon regulat ce admite punctele
date ca vrfuri (nu neaparat consecutive).
Irina Mustata
a, Iasi
, elev
Solutie. Presupunem c
a P este un poligon regulat cu n laturi ce are punctele
A1 , A2 , . . . , Ak ca vrfuri. Dac
a ntre Ai si Ai+1 sunt mi laturi ale lui P , cum
2
unghiul la centru dintre dou
a vrfuri consecutive ale lui P este
, deducem c
a
n
unde am folosit inegalitatea x (1 x)

136

2
m(Ai\
OAi+1 )
m(Ai\
OAi+1 ) = mi
, deci
Q, i = 1, k (cu conventia Ak+1 = A1 ).
n
2
m(Ai\
OAi+1 ) mi
S
a ar
at
am c
a aceast
a conditie este si suficient
a. Fie
, (mi , ni ) = 1,
=
2
ni
a c
a
i = 1, k. Alegnd n multiplu comun al numerelor n1 , n2 , . . . , nk rezult
A1 B2 . . . Bn este poligon regulat care contine vrfurile A1 , A2 , . . . , Ak (deoarece
m1
nm1 2
nm1
si
2 =

N, deci A2 P etc.).
n aceast
a situatie m(A\
1 OA2 ) =
n1
n1
n
n1

Clasa a IX-a
IX.41. Pentru n N, n 10, notam cu u2 (n) numarul format din ultimele
doua cifre ale lui n. Sa se arate ca:
a) u2 a20k+p = u2 (ap ), p {4, 5, . . . , 23}, k N, a {2, 3, 8};
b) u2 a10k+p = u2 (ap ), p {2, 3, . . . , 11}, k N, a {4, 9};
n
c) u2 (5
) = 25, n N;
5k+p
d) u2 6
= u2 (6p ), p {2, 3, . . . , 6}, k N;

e) u2 74k+p = u2 (7p ), p {2, 3, 4, 5}, k N.


Ovidiu Pop, Satu Mare
Solutie. n rezolvarea problemei vom folosi faptul c
a u2 (a) = u2 (b) dac
a si
numai daca a b se divide cu 100.
2k
a) 220k+p = 210
2p = 10242k 2p = (1025 1)2k 2p = (M25 + 1) 2p =
p
= M100 + 2 ;
k
k
320k+p = 910k 3p = 815 3p = 01 3p = (M100 + 1) 3p = M100 + 3p ;
820k+p = 260k+3p = 10246k 23p = (M25 1)6k 8p = (M25 + 1)8p = M100+8p .
Egalit
atile b) e) se demonstreaz
a analog.
1 1 1
IX.42. Fie a, b, c R astfel nct a + b + c = + + . Sa se afle numerele
a
b
c
a, b, c, daca |abc| = 1. (enunt scurtat)
Marius Pachitariu, elev, Iasi
am c
a
Solutie. Ecuatia |abc| = 1 este echivalent
a cu a2 b2 c2 = 1. Observ
(1 ab) (1 bc) (1 ca) = 1 ab bc ca + abc (a + b + c) a2 b2 c2 = 1 a2 b2 c2

(s-a tinut seama de relatia din enunt). Ca urmare, suntem condusi la ecuatia
(1 ab) (1 bc) (1 ca) = 0. Dac
a ab = 1, g
asim c = 1 si, deci, tripletele
(x, 1/x, 1), x R , sunt solutii ale problemei. Celelalte solutii se obtin din acestea
prin permutari circulare.
IX.43. Fie functia f : R R, iar a (1, ). Stiind
ca


1
2
2 1
f x + ax a f x f x + 1, x (, 0) ,

sa se arate ca f nu este injectiva.

Titu Zvonaru, Bucuresti


1
Solutie. Ecuatia x + ax a = , x (, 0) este echivalent
a cu (x 1)
x

2
2
x + ax + x + 1 = 0 sau cu x + x (a + 1) + 1 = 0, x (, 0). Cum > 0,
S < 0, P > 0 ea are dou
a r
ad
acini reale, distincte,
negative

x1 ,x2 .
1
2
Pentru x = x1 , relatia din enunt devine f x1 f x11 f x11 + 1, deci
2

137



2

f x11 1 0, de unde deducem f x11 = 1. La fel f x12 = 1. Prin urmare


f x11 = f x12 , x1 6= x2 ; deci f nu este injectiv
a.

IX.44. Daca 4ABC


este ascutitunghic,
sa se gaseasca maximul expresiei
E = sin A cos A + sin B cos B + sin C cos C.
Cezar Lupu, elev, si Tudorel Lupu, Constan
ta

Solu
t
ie.
Putem
presupune
A

C,
deci
sin
A

sin
B

sin
C
s
i
cos
A

cos B cos C. Folosind inegalitatea lui Cebsev, vom avea

sin A + sin B + sin C


E
cos A + cos B + cos C .

3 3
Dar sin A+sin B+sin C
, cos A+ cos B+ cos C 3(cos A+cos B +cos C)
2
r

3
3 6
3 6
3
. Cum pentru A = B = C = 60 obtinem E =
,
3 = . Deci E
2
4
4
2
3 6
.
deducem c
a maximul lui E este
4
IX.45. Demonstrati ca 4ABC n care are loc egalitatea
X
ha hb mc
= 1,
ma mb mc + ha hb mc + ma mb ic
suma fiind obtinuta prin permutari circulare, iar notatiile fiind cele uzuale, este
echilateral.
Iuliana Georgescu si Paul Georgescu, Iasi
Solutie. Avem ha ia ma si analoagele; deci suma din enunt este mai mica
X
ha hb mc
ha
hb
. Notnd x =
, y =
,
sau egal
a cu S 0 =
ma mb mc + ha hb mc + ma mb hc
ma
mb
X
hc
xy
z=
, avem 0 x, y, z 1 si S 0 =
.
mc
1 + xy + z
X
xy
Cum 1 + xy x + y pentru x, y [0, 1] si analoagele, avem S 0

x+y+z
X
x
= 1; egalitate are loc daca si numai daca x = y = z = 1, deci

x+y+z
ha = ma , hb = mb , hc = mc , adica daca si numai daca triunghiul este echilateral.

Clasa a X-a


2
F2n
2n
X.41. Prove the inequality

, where the Fibonacci numbers Fn


n
Fn1 Fn
are defined by F0 = F1 = 1, Fn+1 = Fn + Fn1 , n 1.
Zdravko Starc, Vrac, Serbia and Montenegro
Solutie. Conform inegalit
atii Cauchy-Schwarz, avem
"
2 #

2

2
2

n
n 2
n
n
n
n
F0 + F12 + + Fn2
+
+ +
F1 + +
F
.
F0 +
1
n
1
n n
0
0
n
2

n
n
P
P 2
P
2n
n
n
=
,
Fk = Fn Fn+1 ,
Fk = F2n (v. RecMat Deoarece
n
k
k=0
k=0 k
k=0
2n
2
2/2002, p. 70), obtinem
Fn Fn+1 F2n
. Egalitate apare pentru n = 1.
n
138

X.42. Sa se rezolve ecuatia 2[x] + 6[x] + 7[x] = 3[x] + 4[x] + 8[x] .


Daniel Jinga, Pitesti
Solutie. Notam [x] = y, y Z. Avem de rezolvat n Z ecuatia 2y + 6y + 7y =
= 3y + 4y + 8y . Dac
a y < 0 avem 7y > 8y si 2y + 6y > 3y + 4y (deoarece aceasta este
echivalent
a cu (2y 1) (3y 2y ) > 0, y < 0). Prin urmare nu avem solutii cu y < 0.
Se verifica usor faptul ca y = 0, y = 1, y = 2 sunt solutii.
Pentru n 3, n N, demonstram ca 3n + 4n + 8n > 2n + 6n + 7n . Este suficient
s
a ar
at
am prin inductie c
a 8n + 4n > 6n + 7n , n 3, care este un exercitiu de rutin
a.
Prin urmare y {0, 1, 2} si atunci x [0, 1) [1, 2) [2, 3). Multimea solutiilor
ecuatiei este intervalul [0, 3).
X.43. Fie f o functie reala nenula cu proprietatea ca
f (x + y xy) = f (x + y) f (x) f (y) ,

2003
Sa se calculeze f
.
2002

x, y R.

Adrian Zanoschi, Iasi


Solutie. Rezolvam problema n mai multe etape.
1. Pentru x = y = 0, din relatia dat
a g
asim f (0) = 0.
2. Pentru y = x = 1, deducem f (1) + f (1) f (1) = 0, adic
a f (1) = 0, sau
f (1) = 1. Dac
a f (1) = 0, pentru y = 1 relatia dat
a implic
a f (x + 1) = 0,
x R, adica f = 0, contrar ipotezei.
3. Ne ocup
am de cazul f (1) = 1 si f (1) = a 6= 0. Pentru y = 1, g
asim
f (x + 1) = af (x) + a,

x R.

(1)

Pentru y = 1 si x x + 1, obtinem f (2x + 1) = f (x + 1) + f (x) sau nc


a
af (2x) + a = af (x) + a + f (x), deci
a+1
f (2x) =
f (x) , x R.
(2)
a
Din (1) si (2) pentru x = 1, deducem f (2) = a2 + a = a + 1, de unde a = 1.
Daca a = 1, relatia (2) implica f (2x) = 0, x R, fals. Prin urmare a = f (1) = 1.
4. Din (1), gasim f (x + 1) = f (x) + 1, x R. Prin inductie, gasim f (x + k) =
= f (x) + k, x R si k Z. Pentru x = 0 deducem c
a f (k) = k, k Z.

p
p
p
5. n relatia dat
a nlocuim x cu si y cu q (p, q Z, q 6= 0) si g
asim f
= ,
q
q
q

2003
2003
adic
a f (x) = x, x Q. Prin urmare, f
=
.
2002
2002
X.44. Urnele U1 , U2 , . . . , Un contin fiecare cte a bile albe si b bile negre. Din
fiecare urna se extrage cte o bila care se depune ntr-o alta urna U . Din urna U
se scoate o bila si se constata ca este alba. Care este compozitia cea mai probabila a
urnei U ?
Petru Minut, Iasi
Solutie. Notam cu Ek evenimentul constnd n faptul ca n U ar fi depuse k bile

k
nk
a
b
k
albe (si n k bile negre). Avem P (Ek ) = Cn
, k = 0, n. Fie
a+b
a+b
E evenimentul ca bila extrasa din U sa fie alba. Conform formulei lui Bayes
139

P (Ek ) PEk (E)


kCnk ak bnk
kC k ak bnk
PE (Ek ) = Pn
.
= Pn n i i ni =
na (a + b)n1
i=1 P (Ei ) PEi (E)
i=1 iCn a b
Aceasta probabilitate este maxima cnd f (k) = kCnk ak bnk , k = 0, n este maxima.
f (k + 1)
na
f (k + 1)
na
si
Observam ca
>1k <
<1k>
. Daca k0
f (k)
a+b
f (k)
a+b
este punctul de maxim pentru f (k), avem
f (k0 + 1) f (k0 ) si f (k0 ) f (k0 1),
na
na
na
na
din care deducem k0
,
+ 1 . Prin urmare, k0 =
, dac
a
N
a+b a+b
a+b
a+b

na
na
si k0 =
+ 1, dac
a

/ N.
a+b
a+b
X.45. Se considera triunghiul ascutitunghic ABC. Sa se arate ca exista un
0B0) =
\
triunghi A0 B 0 C 0 astfel nct A0 (BC), B 0 (AC), C 0 (AB), iar m(AC
0 C 0 ) = m(CB
0 A0 ) = (0, 90]. Daca
\
\
= m(BA
n plus 4ABC este echilateral, sa se
calculeze lungimile laturilor 4A0 B 0 C 0 n functie de a = BC si . (n legatura cu o
problema propusa la O. N. M., 2002)
Dan Popescu, Suceava
Solutie. Fie D proiectia punctului C pe AB, deci D (AB). Fie C 00 (DA) si
00 A) =
00 A00 B) = . De asemenea,
\
E (AC) nct m(EC
si A00 (BC) nct m(C\
00 F C ) = .
fie A1 si C1 , A (BA1 ), C (BC1 ) si F (A1 C1 ) cu A1 C1 k AC si m(A\
1
n aceste conditii, (C 00 E (A00 F = {B 00 }, (BB 00 (AC) = {B 0 }, iar omotetia de

a A00 n A0 (BC)
centru B si raport , raport definit de BB 0 = BB 00 , transform
00
0
0
0
0
si C n C (AB). Prin urmare A B C satisface enuntul.
n cazul n care triunghiul ABC este echilateral se deduce relativ usor ca 4A0 B 0 C 0
este echilateral si (AB 0 ) (CA0 ) (BC 0 ). Cu teorema sinusurilor n 4AB 0 C 0
a

deducem B 0 C 0 =
.
cos + 3 sin

Clasa a XI-a
XI.41. Fie A1 , A2 , . . . , Ak Mn (Z) astfel nct

A(1) A(2) . . . A(k) = In ,


.
unde Sk este multimea permutarilor de ordin k. Sa se arate ca n .. k!.
Vladimir Martinusi, Iasi
Solu
t
ie.
Deoarece
tr
(AB)
=
tr
(BA),
A,
B

M
ca
n (C), inductiv se arata

tr A(1) A(2) A(k) = tr (A1 A2 Ak ), Sk . Tinnd


cont c
a tr (A + B) =
= tr A + tr B, A, B Mn (C) deducem c
a

!
!

P
P
tr
A(1) A(k) = tr (In ) tr
A(1) A(k) = n
Sk

Sk

Sk

Sk

tr A(1) A(k) = n k! tr (A1 A2 Ak ) = n

si cum tr (A1 A2 Ak ) Z, concluzia problemei este imediata.

XI.42. Prin punctele M1 si M2 ale unei elipse se duc normalele la elipsa, care
intersecteaza una din axele de simetrie ale acesteia n M10 , respectiv M20 . Sa se
arate ca mediatoarea segmentului [M1 M2 ] trece prin mijlocul lui [M10 M20 ]. Ramne
proprietatea adevarata pentru hiperbola sau pentru parabola?
Gheorghe Costovici, Iasi
140

x2
y2
+ 2 = 1, = 1 si Mi (xi , yi ) pe curba,
2
a
b
a2 yi
(x xi ), i = 1, 2, si
i = 1, 2. Normala n Mi la curba are ecuatia y yi = 2
b xi

2
a b2
0
xi , 0 , i = 1, 2.
intersecteaza una din axele simetrie, de exemplu Ox, n Mi
a2
2

a b2
Mijlocul M 0 al segmentului [M10 , M20 ] are coordonatele
(x
+
x
)
,
0
. Me1
2
2a2

y1 + y2
x1 + x2
x2 x1
diatoarea segmentului M1 M2 are ecuatia y
x
=
.
2
y2 y1
2
Se verifica usor ca M 0 este situat pe mediatoarea segmentului [M1 M2 ].
Prin calcul se verifica ca proprietatea ramne valabila si pentru parabola.
XI.43. Consideram sirul de functii fn : (0, ) R, fn (x) = nx + ln x (n 1)
si fie xn solutia unica a ecuatiei fn (x) = 0. Sa se calculeze limitele sirurilor (xn )n1
si ((xn )n )n1 .
aeru, Suceava
Angela Tig
atoare si surjective, ceea ce asigur
a
Solutie. Evident, functiile fn sunt strict cresc
existenta si unicitatea solutiei xn .
x1
Folosim n continuare inegalitatile
ln x x 1, x (0, ). Avem:
x

n
1
n
1
n
=
+ ln

+
1 = 0 si
fn
n+1
n+1
n+1
n+1 n+1

n+1
n+1
2
n+1
n+2
2
fn
= ln
+

1
+
> 0, n N .
n+2
n+2 n+2
n+2
n+1 n+2

n+1
n
n+1
n
si atunci
fn (xn ) fn
xn
,
Prin urmare, fn
n+1
n+2
n+1
n+2
1
n 1. G
asim lim xn = 1 si lim xnn = .
n
n
e
XI.44.Sa se determine
func
t
iile
continue
f : (0, ) (0, ) pentru care

f (x) = f
2x2 2x + 1 , x > 0.
Marian Urs
arescu, Roman
p
Solutie. Fie x0 (0, 1] fixat si sirul (xn )n0 definit prin xn+1 = 2x2n 2xn + 1,
n N. Se verifica faptul ca xn (0, 1], n N, (xn )n0 este crescator si lim xn = 1.
p
n
Pentru x = xn , relatia din enunt devine f (xn ) = f
2x2n 2xn + 1 = f (xn+1 ),
n N, deci f (x0 ) = f (xn ). Ca urmare, f (x0 ) = lim f (xn ). Rezult
a c
a f (x) =
Solutie. Fie curba de ecuatie

= f (1), x (0, 1].

p
2x2n 1
, n N. Se arata ca
2
xn > 1, n N. (xn )n0 este descrescator si lim xn = 1. Cum relatia de ren
q
curenta se poate scrie si n forma xn = 2x2n+1 2xn+1 + 1, obtinem f (xn+1 ) =
q

=f
2x2n+1 2xn+1 + 1 = f (xn ), apoi f (x0 ) = f (xn ) si f (x) = f (1), x > 1.
Fie x0 > 1 si sirul definit prin xn+1 =

1+

n concluzie, f (x) = f (1), x (0, ), deci f este functie constanta.


141

XI.45. Fie k N, k 2 si numerele


p reale pozitive a1 , a2 , . . . , ak , b1 , b2 , . . . , bk
cu a1 < a2 < < ak . Definim xn = n b1 an1 + b2 an2 + . . . + bk ank .
a) Sa se demonstreze ca lim xn = ak ;
n
b) Sa se arate ca lim n (xn ak ) = ak ln bk ;
n

n
ak
c) Daca bk = 1, are loc lim n
(xn ak ) = ak bk1 .
n
ak1
p

Marian Tetiva, Brlad


Solutie. a) Avem ak n bk n an1 b1 + + ank bk ak n b1 + b2 + + bk , n N.
p
Deducem ca lim n b1 an1 + b2 an2 + + bk ank = ak .
n

b)

n
ln xn
xn
e ak 1 xn
xn
1 = ak lim n
ln
=
a
lim
ln
=
lim n (xn ak ) = ak lim n
k
x
n
n
n
n
n
ak
ln ak
ak
ak
n
n

a1
a2
ak1
= ak lim ln b1
+ b2
+ + bk1
+ bk = ak ln bk .
n
ak
ak
ak
c)

n
n ln xan
ak
ak
e k 1 xn
lim n
(xn ak ) = lim ak n
ln
=
n
n
ak1
ak1
ln xank
ak

n n
n

n
ak
a1
a2
ak1
ln b1
+ b2
+ + bk1
+1 =
= ak lim
n ak1
ak
ak
ak

n n
n

n
ak
a1
a2
ak1
= ak lim
+ b2
+ + bk1
b1
=
n ak1
ak
ak
ak

n

a1
a2
= ak lim b1
+ b2
+ + bk1 = ak bk1 .
n
ak1
ak1

Clasa a XII-a
XII.41. Sa se calculeze

nN .

Z (1 + x) 1 + x2 . . . 1 + x2n1
x2n

dx, unde x [1, ),

Oana Marangoci,
student
a, Iasi


2
2n1
Solutie. Pentru x (1, ), avem (1 + x) 1 + x 1 + x
=
2

1 x2
1 x2
1 x2 1 x2
n

.
.
.

=
=

= 1 + x + x2 + + x2 1 si atunci
n1
1 x 1 x2
1
x
1 x2

n1
(1 + x) 1 + x2 1 + x2
1
1
1
= 2n + 2n 1 + + ,
n
2
x
x
x
x
relatie care se verific
a si pentru x = 1. Primitivele functiei sunt

1
1
1
+
+ +
+ ln x + C.
(2n 1) x2n 1 (2n 2) x2n 2
x

Z 5
4

5
f (x) sin 2x dx = .
R o functie continua pentru care
XII.42. Fie f : ,
4
4

142

Sa se arate ca exista c

5
,
astfel nct f (c) (1, 2).
4

Mihai Haivas, Iasi


Solutie. Avem
5
Z 5
Z 5
4
4
4
sin 2x
(cos 2x)0
= .
dx
=

dx
=

arctg
(cos
2x)
2

1
2
1
+
cos
2x
4
1

sin
2x

5
5
Z
Z
Z 5
4
4
4
2
sin 2x
dx
sin 2x dx = 0 .
f (x) sin 2x dx =
f (x)
2
2
1
1

sin
2x
2

sin
x

5
Aplicnd teorema de medie exista c ,
astfel nct
4

2
sin 2c = 0 f (c) =
.
f (c)
2
4
2 sin 2c
2 sin2 2c

2
5
Dar

(1,
2)
pentru
c

,
, adica f (c) (1, 2).
4
2 sin2 2c
XII.43. Sa se arate ca

Z 1
arctg ln a
ln a
x2

, a > 1.
a
dx
1

3
ln a
0
Petru R
aducanu, Iasi
Solutie. Se arat
a c
a ax x ln a + 1, x R si a > 1. Obtinem astfel
2
1
1
x2 ln a + 1 ax = x2 2
, x R, a > 1.
x ln a + 1
a
Integrnd ntre 0 si 1, obtinem inegalitatile cerute.
XII.44. Sa se afle numarul radacinilor reale ale polinomului P Z [X] de grad
minim, care admite radacina 2 + , unde verifica ecuatia x3 x + 1 = 0.
Laurentiu Modan, Bucuresti
Solutie. Polinomul n y care se cere, apare prin eliminarea lui x ntre ecuatiile
x3 x + 1 = 0 si y = x2 + x. Deoarece x2 = y x, g
asim x (y x) x + 1 = 0 si
y

1
(y 1)2 y 1
asim x =
+
(evident y 6= 0). Deducem y =
nlocuind din nou x2 g
y
y2
y
3
2
si, n final, P (y) = y 2y + 3y 1. Folosind sirul lui Rolle, polinomul P (y) admite
o singur
a r
ad
acin
a real
a y (0, 1).
XII.45. Fie S5 . Sa se arate ca 2 are puncte fixe daca si numai daca 3
are puncte fixe.
Paul Georgescu si Gabriel Popa, Iasi
Solutie. Folosind descompunerea unei permutari n produs de cicli disjuncti,
se poate observa ca, daca k , 1 k 5, are puncte fixe, atunci contine n
descompunerea sa cicli de lungime l, cu l | k.
Presupunem c
a 2 are puncte fixe. Atunci contine n descompunerea sa cicli de
lungime 1 (puncte fixe) sau 2. Daca are puncte fixe, 3 are de asemenea puncte
fixe. Daca contine n descompunerea sa cicli de lungime 2, fara a avea puncte fixe,
atunci contine n descompunerea sa si un ciclu de lungime 3. Elementele acestui
ciclu vor fi toate puncte fixe pentru 3 . Implicatia reciproc
a se demonstreaz
a analog.
143

Solutiile problemelor pentru preg


atirea concursurilor
din nr. 2 / 2003
A. Nivel gimnazial
G46. Determinati ultimele cinci cifre ale numarului
A = 72000 + 72001 + 72002 + 72003 .
Viorel Cornea si Dan S
tefan Marinescu, Hunedoara
Solutia I. Scriem A = 72000 400. Se constant
a c
a 720 = M1000 + 1. Ca urmare,
2000
7
= M1000 + 1 si A = (M1000 + 1) 400 = M100000 + 400, deci ultimele cinci
cifre ale lui A sunt 00400.
Solutia II (Irina Mustata
, eleva, Iasi). Conform teoremei lui Euler, a(n)
1(mod n) pentru (a, n) = 1. Considernd a = 7, n = 1000, avem (1000) = 400, deci

5
7400 1(mod 1000), adic
a 72000 = 7400 1(mod 1000), de unde A = 72000 400
400(mod 100000), deci num
arul A se termin
a cu 00400.
G47. Determinati valorile parametrilor a, b Z pentru care solutiile sistemului
y
x
x=a
; y=b
y+1
x+1
sunt n Z Z.
Temistocle Brsan, Iasi
Solutie. Fie M multimea perechilor (a, b) Z Z pentru care este ndeplinit
a
cerinta problemei. Observ
am c
a (0, 0) Z Z este solutie a sistemului si c
a aceasta
este unica solutie daca a = 0 sau b = 0. Ca urmare, (a, 0) M , a Z si (0, b) M ,
b Z.
Pe (R \ {1}) (R \ {1}) sistemul dat, dac
a facem abstractie de solutia (0, 0),
este echivalent cu
(b + 1) x = ab 1, (a + 1) y = ab 1.
(1)
ab 1
ab 1
I. a 6= 1, b 6= 1. Obtinem x =
,y=
. Atunci (x, y) Z Z
b+1
a+1
b + 1 | ab 1 si a + 1 | ab 1. Cum ab 1 = (a + 1) (b + 1) (a + 1) (b + 1),
deducem c
a (x, y) Z Z b + 1 | a + 1 si a + 1 | b + 1 b + 1 = a + 1 sau
b + 1 = (a + 1). Prin urmare (a, a) M , a Z, a 6= 1 si (a, a 2) M ,
Z, a 6= 1.
II. a = b = 1. Cum rezult
a ab 1 = 0, sistemul (1) are o infinitate de solutii si
nu toate sunt n Z Z. Deci (1, 1)
/ M.
III. a = 1, b 6= 1 (analog a 6= 1, b = 1). Avem ab 1 6= 0, deci (1) nu
are solutii, iar sistemul dat are (0, 0) ca unic
a solutie. Asadar, (a, 1) M , a Z,
a 6= 1 si (1, b) Z, b Z, b 6= 1.
n concluzie:
M = {(a, 0) , a Z} {(0, b) , b Z} {(a, 1) , a Z} {(1, b) , b Z}
{(a, a) , a Z} {(a, a 2) , a Z} \ {(1, 1)} .
2002
G48. Fie A (0, ) o multime care contine
si avnd proprietatea ca,
2003
a
a+1
a
daca A (a, b N ), atunci
A si
A. Sa se arate ca A Q+ .
b
b
2b
Gheorghe Iurea, Iasi
144

Solutie. Consider
am transform
arile
a+1
a
A
A
b
b

(1)

si
a
a
A
A.
b
2b

(2)

Avem:
2002
(2) 1001
(1) 1002
(2) 501
(1) 502
(2) 251
(1)
(2) 1
A
A
A
A
A
A
A.
2003
2003
2003
2003
2003
2003
2003
2
1
1
(1)
(1)
(1) 2003
A
A
A, deci
A.
2003
2003
2003
1
1
a
b
1
a
a
Fie
Q+ . Cum A A A A, rezulta ca A, adica
b
1
b
b
b
b
Q+ A.

G49. Fie x1 , x2 , . . . , xn+1 R+ astfel nct x1 + x2 + . . . + xn+1 (n + 2) m,


n+4 2
iar x21 + x22 + . . . + x2n+1
M , unde m = min xi , M = max xi . Sa se arate
4
ca exact n dintre numerele date sunt egale.
Eugen Jecan, Dej
Solutie. Daca toate numerele ar fi egale, relatia a doua din enunt ar fi
n+4 2
(n + 1) M 2
M , imposibil. Exista cel putin un numar egal cu M , fie acesta
4
xn+1 . Deducem ca (n + 2) m x1 + + xn + M deci 2m M . De asemenea,
n+4 2
a M = 2m si
M x21 + + x2n + M 2 nm2 + M 2 sau M 2m. Rezult
4
prima relatie devine x1 + x2 + +xn +2m (n + 2) m, deci x1 +x2 + + xn nm
sau nca (x1 m)+ +(xn m) 0. Cum x1 m 0, x2 m 0, . . . , xn m 0,
deducem ca x1 = x2 = = xn = m, valori care verifica si a doua conditie.
n concluzie exact n numere sunt egale cu m si unul egal cu M = 2m.
G50. Fie a N, a 3. Sa se arate ca

an + 1 =
an + 2 = . . . =
an + a 1 , n N a {3, 4} .

Pop, Satu
Mare

Ovidiu
1 =
2 = =
a1 ,
Solutie. Pentru n = 0 relatia din enunt devine

a 1 = 1, echivalent cu 2 a < 5, si cum a N, a 3, avem a {3, 4}.


deci
Reciproc, arat
am ca, dac
a {3, 4},
a
au loc egalitatile din enunt. Fie a = 3;
trebuie ar
atat c
a
3n + 1 =
3n + 2 , n N.
Pentru un num
ar n N exist
a k N (k depinde de n) astfel nct k2 3n + 1 <
2
< (k + 1) . Considernd numarul 3n+2 avem 3n+2 < (k + 1)2 sau 3n+2 = (k + 1)2 .
2
Cum 3n + 2 nu este
atrat perfect,
deducem c
a k2 3n + 1 <
+ 2 < (k+ 1) , de
3n
p

unde rezulta k 3n + 1 < 3n + 2 < k + 1, deci


3n + 1 =
3n + 2 .

4n + 1 =
4n + 2 =
4n + 3 ,
Pentru a = 4, trebuie demonstrat c
a

2
n N. Considernd k =
4n + 1 avem k 2 4n + 1 < (k + 1) si cum 4n + 2,
2
4n+3 nu sunt p
atrate perfecte
pentru nici
un
n N, deducem
k 4n+1 < 4n+2 <
< 4n + 3 < (k + 1)2 si atunci
4n + 1 =
4n + 2 =
4n + 3 = k.
145

3
, cu a + b + c = 1. Sa se arate ca
10

3
2
a a + bc + b b + ca + c c + ab < .
3
4
Gabriel Dospinescu, elev, Onesti
p

a+b+a+c
1+a
si atunci
=
Solutia. Avem a + bc = (a + b) (a + c)
2
2

2
Xa +a
X
X
6
1
2
=
1+
a2 . Cum a = 1 (b + c) < 1
=
a a + bc
2
2
10
5

X 1
1
4
1 37
3
etc., rezulta ca
1+
a2 <
1+3
=
< .
2
2
25
2 25
4

2
Pentru inegalitatea din stnga avem: a + bc = (a + b) (a + c) a + bc si
X
X
X
X

a a + bc = 1 2
ab +
a bc. R
amne s
a ar
at
am
atunci
a a + bc >
2

X (a b)
X
X
X
2
1
a b . Este
c
a 2
ab+
a bc 0 sau, echivalent,

c
3
3
2

2
2
(a b)
a b , echivalenta cu
a + b 3c
suficient sa aratam ca
c
3

sau cu a + b + 2 ab 3 3a 3b.
Ultima
inegalitate se scrie 4a + 4b + 2 ab 3 si
3
este adevarata n conditia a, b
, .
10
G52. Se considera o piramida formata din patrate 1 1,
avnd n trepte, pe treapta k existnd 2k 1 patrate (n figura,
n = 4). Aflati numarul minim de dreptunghiuri, fiecare alcatuit
numai din casute ntregi, n care poate fi mpartita tabla.
Adrian Zahariuc, elev, Bac
au
Solutia I (a autorului). Color
am c
asutele tablei alternativ n alb si negru
(c
asuta din vrf este neagr
a). Cum modulul diferentei dintre num
arul de c
asute albe
si num
arul de c
asute negre dintr-un dreptunghi este cel mult 1 si cum sunt cu n
casute negre mai multe dect albe, trebuie sa avem cel putin n dreptunghiuri. Acest
num
ar poate fi obtinut t
aind pe nivele. Asadar, r
aspunsul este n.
Solutia II (Irina Mustata
a, Iasi). Unim, ca n figu, elev
ra, vrfurile din stnga sus ale patratelor marginale din stnga;
se observa ca deasupra acestei drepte nu mai exista vrfuri
ale piramidei. Oricare dou
a p
atrate marginale din stnga nu
pot apartine aceluiasi dreptunghi, deoarece ar nsemna c
a acel
dreptunghi va avea coltul din stnga sus deasupra dreptei considerate; prin urmare,
numarul minim de dreptunghiuri este n, minim atins pentru mpartirea pe trepte.
Solutia III (Marius Pachitariu, elev, Iasi). Prin inductie complet
a.
G53. FieABCD un patrat de latura 70.
S
a
se
arate
c
a
exist
a
o
multime de

patrate Pk = Ai Bi Ci Di | Ai Bi = i, i = 1, k care sa aiba suma ariilor egala cu aria


patratului dat. Putem acoperi patratul ABCD cu elementele multimii Pk ?
Petru Asaftei, Iasi
k(k+1)(2k+1)
Solutie. Trebuie s
a avem 12 +22 + +k2 = 702 , echivalent cu
=
6
G51. Fie a, b, c

146

= 702 , de unde k = 24. Solutia este unic


a deoarece k > 24 implic
a 12 +22 + +k2 >
> 702 iar k < 24, 12 + 22 + + k 2 < 702 .
Aratam ca multimea Pk nu poate acoperi patratul ABCD. Pentru aceasta, sa
observ
am c
a p
atratele Ai Bi Ci Di , pentru a acoperi Pk , nu au puncte comune, exceptie
f
acnd laturile (altfel aria acoperit
a de acestea este mai mic
a dect 12 +22 + +242 =
= 702 ); de asemenea, patratele Ai Bi Ci Di nu lasa "spatii goale" ntre ele (altfel aria
acoperita de ele plus ariile "spatiilor goale" este mai mare ca 12 +22 + +242 = 702 ).
Analiznd pozitia p
atratului de latur
a 1 n p
atratul ABCD, observ
am c
a r
amne
o suprafata ce nu mai poate fi acoperit
a (r
amne o suprafata dreptunghiular
a de
laturi 1 si l 1 n care nu "ncape" nici unul din patratele ramase, care au laturile
mai mari sau egale cu 2).
G54. Sa se arate ca nu putem alege nici un punct n interiorul triunghiului
echilateral ABC de latura l 10, care sa aiba distantele la vrfuri numere prime
distincte.
Doru Buzac, Iasi
Solutie.
Presupunem, prin absurd, c
a exist
a
A
M Int (ABC) astfel nct M A, M B, M C s
a fie numere prime.
Cum M A, M B, M C < l si M A, M B, M C pot fi
7
laturile unui triunghi (teorema lui Pompeiu), deducem
c
a AM, BM, CM {3, 5, 7}. Fie AM = 7, BM = 3,
M
3
5
CM = 5.

\
Fie N astfel nct BN = 3 si m(M
BN ) = 60 . Evi- B
C
dent, triunghiul BM N este echilateral. Din congruenta
3
7
triunghiurilor AM B si CN B deducem N C = 7. Atunci
N
M N 2 +M C 2 N C 2
1
\
\
\
C) = 180 ,
cos N
MC =
= , deci m(N
M C) = 120 si m(BM
2 MN MC
2
fals. Prin urmare, nu exista M Int (ABC) cu proprietatea ceruta.
G55. Printr-un punct situat n interiorul unui tetraedru se duc planele paralele cu
fetele tetraedrului. Daca V1 , V2 , V3 , V4 sunt volumele tetraedrelor unic determinate
de aceste plane, iar V este volumul tetraedrului dat, sa se arate ca
27V 16 (V1 + V2 + V3 + V4 ) .
Neculai Roman, Mircesti (Iasi)
Solutie. Not
am cu xi distanta de la punctul considerat la fata tetraedrului
A1 A2 A3 A4 opusa vrfului Ai si cu hi naltimea corespunzatoare acestei fete. Fie
4
p
P
hi xi
xi
ai = 3 Vi /V , i = 1, 4. Avem ai =
= 1 , i = 1, 4, deci
ai =
hi
hi
i=1
4 x
4 S x
4
P
P
1 P
i
i i
=4
=4
Vi = 4 1 = 3.
=4
V i=1
i=1 hi
i=1 Si hi

Relatia de demonstrat se scrie 27 16 a31 + a32 + a33 + a34 si decurge din faptul

3
a3 + a32 + a33 + a34
33
27
a1 + a2 + a3 + a4
c
a 1
= 3 =

.
4
4
4
64

B. Nivel liceal

b si B
b se
L46. Fie ABCD un patrulater inscriptibil. Bisectoarele unghiurilor A
147

intersecteaza ntr-un punct situat pe latura [CD]. Sa se arate ca CD = AD + BC.


Mircea Becheanu, Bucuresti
b = 2,
Solutia I (Irina Mustata
,
elev
a
,
Ia
s
i).
Fie
m(
A)

D
E P
b
m(B) = 2, iar P (CD) astfel nct AD = DP ; avem ca

C
1
b = . Deosebim dou
\
a cazuri,
180 m(D)
m(AP
D) =
2
dup
a cum E (DP ) sau E (P C); ne plas
am n prima
[
\ patrulaterul ABP E este
situatie. Deoarece EP
A EBA,
B
\
\) = 180 A
inscriptibil, deci m(CP
B) = . Atunci m(CBP
b = , deci 4CP B este isoscel cu CP = CB, de unde concluzia.
m(C)
Solutia II. Folosind teorema sinusurilor n triunghiurile ADE, BEC si AEB,
obtinem:
B
A
sin
sin
2
2
AB, CE =
AB,
DE =
B
A
A+B
A+B
2 cos sin
2 cos sin
2
2
2
2

A
B
sin B
sin A
2
2
AD =
AB, BC =
AB,
B
A
A+B
A+B
2 cos sin
2 cos sin
2
2
2
2
din care deducem
sin A + sin B
CD = DE + CE = AD + BC =
AB.
A
B
A+B
4 cos cos sin
2
2
2
Not
a. S-a mai primit solutie corecta de la Marius Pachitariu, elev, Iasi.
L47. Daca un triunghi are patratele laturilor n progresie aritmetica, atunci
simetricul centrului de greutate fata de latura mijlocie se afla pe cercul circumscris
triunghiului.
Gabriel Popa si Paul Georgescu, Iasi
Solutie. Fie a, b, c lungimile laturilor triunghiului ABC, c < a < b cu 2a2 = b2 +c2 .
Simetricul cercului circumscris triunghiului ABC fata de BC este cercul circumscris triunghiului BHC si atunci simetricul lui G fata de BC este pe cercul circumscris daca si numai daca BHGC este patrulater inscriptibil, echivalent cu faptul ca
b
\ = m(BHC)
\ = m(A).
m(BGC)
2
\ = a (se foloseste relatia 2a2 = b2 +c2
n triunghiul BGC se determina cos BGC
2bc
a2
\ = cos A, deci
si formula medianei). Cum cos A =
, deducem c
a avem cos BGC
2bc
b = , ceea ce ncheie solutia.
\ + m(A)
m(BGC)
Not
a. Solutii corecte au dat Irina Mustata
si Marius Pachitariu, elevi, Iasi.
L48. Fie R, r, R1 raza cercului circumscris 4ABC, raza cercului nscris 4ABC,
respectiv raza cercului circumscris 4DEF determinat de picioarele bisectoarelor interioare ale 4ABC. Sa se arate ca R/2 R1 r.
Marian Tetiva, Brlad
Solutie (Titu Zvonaru, Com
anesti (Bac
au)). Inegalitatea r R1 este adevarata oricare ar fi punctele D (BC), E (CA), F (AB) (Liliana Niculescu 148

O metoda de demonstrare a unor inegalitati geometrice, GM - 2-3/1993; Teorema 1,


p. 51). ntr-adevar, fie Q centrul cercului circumscris 4DEF si d1 , d2 , d3 distantele
de la Q la laturile BC, CA, AB. Cum fiecare dintre dreptele BC, CA si AB este
secant
a sau tangent
a cercului circumscris 4DEF , avem d1 R1 , d2 R1 , d3 R1 ,
deci
ad1 bd2 cd3 aR1 bR1 cR1
pr = A[ABC] = A[QBC] +A[QCA] +A[QAB] =
+
+

+
+
= pR1 ,
2
2
2
2
2
2
adica r R1 .
Inegalitatea R1 R/2 este adev
arat
a, dac
a punctele D, E, F apartin segmentelor
determinate de picioarele n
altimilor si mijloacele laturilor respective (asa cum se
ntmpl
a cu picioarele bisectoarelor). n acest caz aceste puncte sunt n interiorul
cercului lui Euler al 4ABC, a carui raza este R/2, si rezulta R1 R/2.

L49. ntr-un patrat 10 10 se nscriu numerele 1, 2, 3, . . . , 100 n asa fel nct


oricare doua numere consecutive sa se afle n casute vecine. Demonstrati ca exista
o linie sau o coloana ce contine macar doua patrate perfecte.
Adrian Zahariuc, elev, Bac
au
Solutie. Observam ca avem 10 patrate perfecte dintre care 5 sunt pare. Presupunem ca patratele perfecte sunt situate pe linii si coloane diferite. Coloram tabla
ca pe o tabl
a de sah n alb si negru. Numerele pare vor fi situate pe c
asute de aceasi
culoare, la fel numerele impare.
Fie (x1 , y1 ), (x2 , y2 ), . . . , (x10 , y10 ) coordonatele casutelor n care sunt situate
p
atratele perfecte. Datorit
a presupunerii c
a p
atratele perfecte sunt pe linii si coloane
diferite, numerele xi , i = 1, 10 snt diferite dou
a cte dou
a si {x1 , x2 , . . . , x10 } =
= {1, 2, . . . , 10}; la fel pentru numerele yi , i = 1, 10. Obtinem (x1 + y1 ) + (x2 + y2 ) +
+ + (x10 + y10 ) = 2 (1 + 2 + + 10) = numar par si rezulta ca un numar par de
perechi are suma par
a. Cum perechile cu suma combinatiilor par
a au aceasi culoare,
deducem c
a exist
a un num
ar par de p
atrate perfecte pare, absurd.
Not
a. Solutie corect
a a dat Marius Pachitariu, elev, Iasi.

L50. Fie (an )n1 o progresie aritmetica avnd a1 = 5, r = 2002. Pentru un


element b al progresiei, sa se arate ca bm apartine progresiei daca si numai daca
60 | m 1.
Mihai Piticari, C-lung Moldovenesc
Solutie. S
a demonstr
am mai nti urm
atoarea
Lem
a. Daca a, m, n sunt numere naturale nenule astfel nct n | am 1 si m
este cel mai mic numar cu aceasta proprietate, atunci n | ak 1 daca si numai daca
m | k.
ntr-adev
ar, dac
a m | k, atunci k = sm si din n | am 1 | asm 1 rezult
a
k
n | a 1. Presupunem n | ak 1. Conform teoremei mp
artirii cu rest, exist
a
c si r numere naturale astfel nct k = mc + r, r < m. Din n | ak 1, rezult
a
n | amc+r 1 = ar (amc 1) + ar 1 si deducem n | ar 1. Cum r < m, pe baza
minimalit
atii lui m rezult
a r = 0, deci m | k.

Revenim la problema data. Evident, bm apartine progresiei daca si numai daca


2002 | bm b. Cum b = 5+2002p, rezult
a c
a 2002 este prim cu b si atunci bm apartine
m1
progresiei daca si numai daca 2002 | b
1. Din bm1 1 = (5 + 2002p)m1
149

5m1 + 5m1 1, urmeaz


a c
a 2002 | bm1 1 dac
a si numai dac
a 2002 | 5m1 1.
Avem 2002 = 2 7 11 13 si 6, 10, 4 sunt minime cu proprietatile 7 | 56 1,
11 | 510 1, 13 | 54 1. Deci, pe baza lemei, 2002 | 5m1 1 daca si numai daca
6 | m 1, 10 | m 1, 4 | m 1, adic
a dac
a si numai dac
a 60 | m 1.
Not
a. Solutie corect
a a dat Marius Pachitariu, elev, Iasi.

L51. Fie A, B M2 (R) doua matrice care comuta si pentru care det A2 + B 2 <
< (det A + det B)2 . Sa se arate ca xA + yB este matrice nesingulara, x, y R .
C
at
alin Calistru, Iasi
Solutie. Se arat
a cu usurinta c
a
det (xA + yB) = x2 det A + y 2 det B + xy [det (A + B) det A det B] , x, y C,

prin urmare

det (A + iB) det (A iB) = (det A det B)2 + [det (A + B) det A det B]2 .

Cum AB = BA, avem (A + iB) (A iB) = A2 + B 2 si relatia anterioar


a devine

2
2
2
det A + B = (det A det B) + (det (A + B) det B det A)2 .

Conditia din enunt se rezum


a la a ar
ata c
a x, y R , avem det (xA + yB) 6= 0, deci
2
2
ecuatia x det A + y det B + xy [det (A + B) det A det B] = 0 nu admite solutii
reale nebanale. ntr-adev
ar, discriminantul acestei ecuatii este

= [det (A + B) det A det B]2 4 det A det B = det A2 + B 2

2
2
(det A det B) 4 det A det B = det A2 + B 2 (det A + det B) < 0.

L52. Fie Q C [X] un polinom de grad m avnd radacinile distincte. Sa se


determine cardinalul multimii
E = {P C [X] | A Mn (C) a. . Q (A) = On si P (X) = det (XIn A)} .

Ovidiu Munteanu, Brasov


Solutie. Fie P E. Exist
a A Mn (C), Q (A) = On si P (x) = det (xIn A).
Fie C o radacina a lui P . Rezulta ca sistemul AX = X are si o solutie nebanala,
notat
a X0 . Este usor de v
azut c
a Q (A) X0 = Q () X0 si prin urmare Q () = 0, deci
ad
acinile
r
ad
acinile lui P sunt n numere din multimea {1 , 2 . . . m }, unde i sunt r
lui Q. Prin urmare, P este determinat de n numere, nu neap
arat distincte, 1 , 2 ,
. . . , n din multimea {1 , 2 , . . . , m }.
Invers, dnd un polinom P care are ca r
ad
acini n numere 1 , 2 , . . . , n ca
mai sus, fie A matricea care are pe diagonala principal
a 1 , 2 , . . . , n si 0 n rest.
Evident, det (xIn A) = (x 1 ) (x n ) = P (x), iar Q (A) este o matrice
care are pe diagonala principala Q (i ) = 0 si 0 n rest, deci Q (A) = On . Rezulta ca
P E. Prin urmare, num
arul de elemente al lui E este egal cu num
arul de posibilit
ati
de a alege n numere oarecare dintr-o multime cu m elemente, f
ar
a a conta ordinea,
n
adic
a Cm
.
L53. Fie n 2 si (A, +, ) un inel comutativ cu n2 elemente, care are cel mult
n 2 divizori ai lui zero. Sa se arate ca A este corp.
Gabriel Dospinescu, elev, Onesti
Solutie. Presupunem c
a A nu este corp. Fie T multimea divizorilor lui zero;
urmeaza ca T 6= . Presupunem ca T are k elemente, 1 k n 2. Consideram
150

x T si a A \T . Exist
a d 6= 0 nct xd = dx = 0. Atunci (ax) d = d (ax) = 0, deci
ax = 0 sau ax T . Daca ax = 0, avem a T , fals. Deci ax T si prin urmare putem
defini f : A \T T , f (a) = ax. Cum A \T are n2 k1 elemente, T are k elemente
n2 k 1
si
> n, exist
a elementele diferite a1 , a2 , . . . , an+1 A \ T astfel nct
k
f (a1 ) = f (a2 ) = = f (an+1 ). Deducem ca (an+1 ai ) x = x (an+1 ai ) = 0,
i = 1, n. Deci an+1 ai T pentru i = 1, n. Asadar T are cel putin n elemente,
contradictie.
L54. Fie f : R R o functie cu derivata continua pentru care f (x) 6= 0, x 6= 0.
Sa se determine functiile continue : R R care satisfac identitatea
Z y

Z x

1
1
f (x)
(t) dt (y) = f (y)
(t) dt (x) , x, y R,
a
a
0
0
unde a 6= 0 este o constanta data.
Adrian Corduneanu, Iasi
Solutie. Relatia dat
a pote fi scris
a sub forma
Rx
Ry
1
1
(t) dt (y)
(t) dt (x)
0
0
a
a
=
, x, y6=0.
f (y)
f (x)
Rx
1
Z x
(t) dt (x)
0
1
a
(t) dt (x) = cf (x),
Prin urmare,
= c, c R, deci
f (x)
a
0
egalitatea avnd loc si pentru x = 0, n baza continuitatii. Pentru x = 0 rezulta
(0) = acf (0).
1
Prin derivare obtinem ecuatia (x) 0 (x) = cf 0 (x), ce are solutia
a

Z x
eat (acf 0 (t)) dt , unde k = (0) = acf (0). Asadar,
(x) = eax k +

0
Z x
at 0
ax
(x) = ace
e f (t) dt , c R.
f (0) +
0

a1
a
L55. Fie a (0, ) \ {1}. Definim sirul (xn )n1 prin x0 =
; xn =

ln a
ln a
n

xn1 , n 1. Aratati ca sirul este convergent si calculati lim xn si lim nxn .


n
n
ln a
Gheorghe
Iurea,
Iasi
Z 1
x n
Solutie. Prin inductie matematic
a se arat
a c
a xn =
a x dx, n N ,
0

a1
. Pentru a (0, 1) avem xn a xn ax xn , x [0, 1]; integrnd,
ln a
Z 1
a
1
obtinem
ax xn dx

. Deci lim xn = 0. Pentru a (1, ), din


n
n+1
n+1
0
1
a
xn
, n N. Ca urmare,
xn xn ax xn a, x [0, 1], deducem
n+1
n+1
lim xn = 0.
x0 =

n concluzie, pentru orice a (0, ) \ {1}, lim xn = 0. Din relatia de recurenta


n
rezult
a lim nxn1 = a, de unde deducem c
a lim nxn = a.
n

151

Probleme propuse1
Clasele primare
P.74. Descopera regula de formare, apoi completeaza sirurile urmatoare:
a) 1,2,3; 2,3,5; 3,,; 5,,.
b) 11,10,12; 13,12,14; 15,,; 17,,.
c) 2,6,4; 3,7,5; 4,8,6; 5,,; 6,,.
( Clasa I )
nv. Maria Racu, Iasi
P.75. R
aspundeti la urm
atoarele ntreb
ari:
a) De cte suprafete este m
arginit cubul?
b) Ce forma au fetele cuboidului?
c) Ce forma are un obiect care se aseamana cu sfera?
( Clasa I )
Aliona Loghin, elev
a, Iasi
P.76. Completati casetele din expresia 654321 cu semnele grafice "+"
sau "" pentru a obtine cel mai mic rezultat posibil.
( Clasa a II-a)
nv. Gheorghe Toma, Muncelu de Sus (Iasi)
P.77. Un corp este format din trei cuburi a, b, c ca n
4
5
2
figura al
aturat
a. Fiecare cub are fetele numerotate de la
6
1 la 6, iar suma numerelor de pe oricare dou
a fete opuse
3
2
4
ale sale este 7. S
tiind c
a pe fetele lipite ale cuburilor a si
b este scris acelasi numar si ca aceeasi proprietate o au si
a
b
c
cuburile b si c, s
a se afle suma tuturor numerelor scrise pe fetele corpului care nu se
v
ad.
( Clasa a II-a)
Oxana Pascal, elev
a, Iasi

P.78. a) Verific
a egalit
atile: 1 + 3 + 5 + 7 = 4 4, 1 + 3 + 5 + 7 + 9 + 11 = 6 6;
b) Scrie rezultatul la fel ca la punctul a) pentru 1 + 3 + 5 + 7 + + 19.
( Clasa a III-a)
Andreea Surugiu, student
a, Iasi
P.79. 7 elevi m
anc
a 7 inghetate n 6 minute. Cti elevi vor mnca 24 nghetate
n 36 minute?
( Clasa a III-a)
Alexandru Tudorache, elev, Iasi
P.80. Dou
a orase sunt legate printr-o linie de cale ferat
a. La fiecare or
a pleac
a
un tren din fiecare oras c
atre cel
alalt. Toate trenurile merg cu aceeasi vitez
a si fiecare
calatorie de la un oras la altul dureaza 6 ore. De cte ori fiecare tren, care parcurge
distanta dintre orase, se ntlneste cu trenuri care merg n sens opus?
( Clasa a IV-a)
Alexandru Tudorache, elev, Iasi
P.81. Sa se arate ca din fetele unui cub confectionat din carton putem construi,
f
ar
a resturi, fetele a sase cuburi.
( Clasa a IV-a)
Petru Asaftei, Iasi
P.82. Sa se afle cel mai mare numar natural de forma abcd cu proprietatile:
a 6= d, b + c = 5 (a + d).
( Clasa a IV-a)
Adrian Andronic, elev, Iasi
P.83. Mircea mpreuna cu fratele sau au un numar de bomboane mai mic dect
30. Mircea are de 3 ori mai multe dect fratele s
au. Aflati cte bomboane trebuie s
a
1

Se primesc solutii pn
a la data de 1 iunie 2005.

152

i dea Mircea fratelui s


au pentru a r
amne cu un num
ar de dou
a ori mai mare dect
al fratelui. Cte bomboane avea Mircea la nceput si cu cte a ramas?
( Clasa a IV-a)
Inst. Tudor Tudorache, Craiova

Clasa a V-a
V.51. ntre oricare doua numere naturale definim operatia a b = ab + a.
a) Sa se rezolve ecuatia 2 (x + 1) = 34.
b) Este operatia dat
a comutativ
a?
Vasile Solcanu, Bogd
anesti (Suceava)
V.52. Un dreptunghi se poate descompune n 1344 patrate de arie 25 cm2 . Aflati
perimetrul dreptunghiului daca acesta este: a) maxim posibil; b) minim posibil.
Romanta Ghita
si Ioan Ghita
, Blaj
V.53. Determinati n N pentru care

3
1
1
1 + + + n
2 + 6 + + 98
2
3
3
=

.
1
1
5
1 + 3 + + 17
1 + + + n
4
5
5
Viorel Cornea, Hunedoara
V.54. S
a se arate c
a nu exist
a numere rationale pozitive a, b, c astfel nct
b+c
c+a
a+b
= 22003 ,
= 22004 si
= 22005 .
ab
bc
ca
Andrei - Sorin Cozma, elev, Iasi
2
3
2004
1
+
+
+ +
, unde ai N ,
V.55. Fie num
arul rational N =
a1
a2
a3
a2004
1
i = 1, 2004. Sa se arate ca exista a1 , a2 , . . . , a2004 astfel nct N =
. Gene2005
ralizare.
Petru Asaftei, Iasi

Clasa a VI-a
VI.51. Pentru efectuarea unei lucr
ari, trei muncitori au fost retribuiti cu sume
de bani direct proportionale cu numerele 16, 14, 17. Unul dintre muncitori constata
ca daca sumele primite ar fi fost invers proportionale cu numerele 3, 4, 5, el ar fi
primit mai putin cu 1000000 lei. Aflati ce sum
a de bani a primit fiecare muncitor.
Ion Visan, Craiova
1 2 3
VI.52. Determinati numerele ntregi n care pot fi scrise sub forma n = + + ,
a
b c
cu a, b, c Z .
Gheorghe Iurea, Iasi
d
VI.53. Se da unghiul ascutit xOy si punctele A, B (Ox, C, D (Oy astfel
nct A (OB), C (OD), AB 6= CD si t OA + s AB = t OC + s CD, cu
d sunt
s, t R . Atunci mediatoarele segmentelor [AB] si [CD] si bisectoarea lui xOy
trei drepte concurente daca si numai daca t = 2s.
Ioan S
ac
aleanu, Hrl
au
VI.54. Fie 4ABC isoscel (AB = AC), N mijlocul lui [AC], iar D un punct pe
prelungirea lui [BC] astfel nct CD < BC. S
a se arate c
a ntre triunghiurile ABN
si N CD nu exist
a nici o congruenta.
Romanta Ghita
si Ioan Ghita
, Blaj
153

VI.55. Fie punctele O, A1 , A2 , A3 , . . . astfel nct OA1 = OA2 = OA3 = =

\
\
= 1 cm, iar m(A\
1 OA2 ) = 1 , m(A2 OA3 ) = 2 , m(A3 OA4 ) = 3 etc. (toate unghiurile se consider
a n sens orar). S
a se arate c
a exist
a k 6= l astfel nct Ak = Al .
Cristian Laz
ar, Iasi

Clasa a VII-a
VII.51. Fie a, b N astfel nct n2004 a se divide cu n b, pentru orice n N,
n 6= b. Sa se arate ca a = b2004 .
Alexandru Negrescu, elev, Botosani
VII.52. Fie a, b, c R cu a + b + c = 0; s
a se arate c
a
3

3
3
3
a b3 + b3 c3 + c3 a3 = 3(a b)(b c)(c a) a2 bc b2 ac c2 ab .
Anca Tutescu, elev
a, Craiova
VII.53. Determinati m, n, p Z astfel nct solutia inecuatiei |mx 1| n s
a
fie [p, p + m + 1].
Ciprian Baghiu, Iasi
d de masura 10 si un segment [M N ] de lungime
VII.54. Se considera unghiul xOy
a. S
a se construiasc
a, folosind numai rigla si compasul, un triunghi dreptunghic
OAB, A (Ox, B (Oy, avnd o catet
a de lungime a.
Florin As
avoaie, elev, Iasi
VII.55. Fie ABCD patrulater convex, iar {O} = AC BD. Bisectoarele in\ BOC,
\ COD,
\ DOA
\ taie laturile (AB), (BC), (CD),
terioare ale unghiurilor AOB,
respectiv (DA) n M , N , P , respectiv Q. Sa se arate ca dreptele M Q, N P si BD
sunt concurente sau paralele.
Constantin Cocea si Dumitru Neagu, Iasi

Clasa a VIII-a
VIII.51. Se considera functiile f, g, h : R R definite prin f (x) = x, g (x) =
x
= , h (x) = 3. Not
am {A} = Gg Gh , {B} = Gf Gh , iar C si D sunt punctele de
3
intersectie ale dreptei x = 2 cu Gf , respectiv Gg . Determinati masurile unghiurilor,
perimetrul si aria patrulaterului ABCD.
Dumitru - Dominic Bucescu, Iasi
VIII.52. Fie E (x, y) = 2004 2x2 5y 2 + 2xy + 6y, cu x, y N. Determinati
valoarea maxima a lui E.
Gheorghe Iurea, Iasi
VIII.53. S
a se arate c
a pentru orice a, b, c R, are loc inegalitatea

a4 + b4 + c4 + 3a2 b2 + 3a2 c2 + 3b2 c2 2 a3 b + ab3 + a3 c + ac3 + b3 c + bc3 .


Marian Tetiva, Brlad
VIII.54. Pentru a, b, c (0, ), s
a se demonstreze inegalitatea

p
p
p
2a b2 + c2
2
2
2
2
2
a+ b +c a +b +c <
< a + b2 + c2 .
a2 + b2 + c2
Radu Frunz
a si Mircea Cosbuc, elevi, Iasi
VIII.55. O piramida triunghiulara regulata este tetraedru regulat daca si numai
dac
a unghiurile f
acute de o fata lateral
a cu planul bazei, respectiv cu o alt
a fata
lateral
a, sunt congruente.
Claudiu - S
tefan Popa, Iasi
154

Clasa a IX-a
IX.51. Fie sirul (an )n1 definit prin: a1 = 1 + 2 3; a2 = a1 + 4 + 5 + 6 7 8;
a3 = a2 + 9 + 10 + 11 + 12 13 14 15 etc.
a) S
a se determine semnele cu care apar 100 n a100 , respectiv 91 n a91 .
b) S
a se afle formula termenului general al sirului.
Lidia Nicola, Craiova
IX.52. S
a se determine functiile f, g : Z R cu propriet
atile: f (0) = 2004,

f este par
a
,
g
este
impar
a
s
i
exist
a
a,
b

N
astfel
ncat
f
(x)
=
f
x
+
x
+
a
,
2

g (x) = g x + x + b , x Z.
D. M. B
atinetu-Giurgiu, Bucuresti
IX.53. Exista functii f : R R pentru care
|f (x + y + z + t) + cos x + cos y + cos z + cos t| < 4,

x, y, z, t R?

Lucian Tutescu, Craiova


[ ) = .
IX.54. Fie ABCD un patrat de latura a, iar T (AD) astfel nct m(ABT
Notam {S} = AC BT si fie R punctul n care perpendiculara n S pe BT intersecteaz
a AB.
a) S
a se arate c
a 4RST este isoscel.
b) Sa se exprime RS functie de a si .
Gheorghe Costovici, Iasi
IX.55. Fie ABC un triunghi cu c < b. Not
am cu M si N mijloacele laturilor
[AB], respectiv [AC] si cu D si E punctele de tangenta a cercurilor nscris si respectiv
A- exnscris triunghiului cu latura [BC]. Aratati ca
(i) M E si N D se intersecteaza pe mediana din vrful A;
(ii) M D k N E a = 2 (b c);
(iii) dac
a a 6= 2 (b c), atunci M D si N E se intersecteaz
a pe prelungirea medianei
din A.
Temistocle Brsan, Iasi

Clasa a X-a
X.51. Fie OABC un tetraedru cu OA OB OC, circumscris
unei sfere de
3+1
R

.
raz
a r. Dac
a R este raza cercului circumscris 4ABC, atunci
r
2
Cezar Lupu, elev, Constanta
X.52. Fie polinomul
X
6n+1 12n+2

=
ak X k , n N.
P (X) = 1 + X + X 2

S
a se arate c
a

2n
P

k=0

a6k =

2n
P

k=0

a6k+2 .

k=0

C
at
alin Calistru, Iasi
X.53. Fie a, b, c (1, ) astfel nct a + b + c = 9. S
a se arate c
a
3

3
3
3
loga 2b + c + logb 2c + a + logc 2a + b 12.
aeru, Suceava
Angela Tig
X.54. Definim multimile Ak , k 1, prin
n n
o

A1 =
| n = 1, 2, . . . , 10000 \ {1} ; Ak = (Ak1 \ {a, b}) a2 lg b ,
2003
155

cu a, b Ak1 arbitrare, k 2. S
a se determine A9999 .
Marius Pachitariu, elev, Iasi
2
X.55. Fie a, b Z cu a 4b < 0, iar o solutie a ecuatiei x2 + ax + b = 0.
Definim functia f : Z Z Z, f (x, y) = x2 axy + by 2 . Pentru orice pereche
1
(x, y) f 1 (1), sa se arate ca (x + y)card f (1) = 1.
Andrei Nedelcu, Iasi

Clasa a XI-a
XI.51. S
a se calculeze determinantul unei matrice p
atratice de ordinul patru care
are toti minorii de ordin trei egali.
Lucian - Georges L
adunc
a, Iasi

XI.52. Fie functia f : M2 (R) [0, ), f (A) = det A2 + I2 , A M2 (R).


2
2
a) S
a se arate c
a f (A) = (det A 1) + (tr A) , A M2 (R).
b) Sa se demonstreze ca f este surjectiva, dar nu este injectiva.
Ovidiu Pop, Satu Mare
XI.53. Fie R; pentru n 3, definim

cos + 2k cos
sin
+ 2k

sin

n
n

k =
, k N .
2(k+1)
cos sin + 2(k+1)
sin
cos +
n
n

S
a se calculeze limita sirului (an )n3 , an =

n2
P
k=1

|k |.

Gheorghe Croitoru si Gabriel Popa, Iasi


XI.54. Fie k N ; sa se arate ca ecuatia xn+k xn xn1 x 1 = 0 are
o singura solutie pozitiva, pe care o notam xn . Sa se arate apoi ca sirul (xn )n1 este
convergent; ce se poate spune despre limita sa?
Dumitru Mihalache si Marian Tetiva, Brlad
XI.55. Determinati toate functiile f : R R pentru care

f x2n+1 + x x f 2n+1 (x) + f (x) , x R,

unde n N. (n leg
atur
a cu problema 2811 din Crux Mathematicorum, nr. 1/2003)
Titu Zvonaru, Com
anesti

Clasa a XII-a

sin x
dx, p < 2, este convergent.
p
1/n x
Rodica Luca Tudorache, Iasi
XII.52. Fie f : [0, 1] R derivabil
a, cu derivata continu
a, astfel nct
f (x) + f 0 (x) = 0, x [0, 1]. S
a se arate c
a
Z
Z 1
(2e 5) f (1) 4 2 xex f (x 1)
f (x) dx
dx.
+
5
e 1 (x2 + 1)2
0
Mihail Bencze, Brasov
XII.53. Prove that
Z x t
2 (ex 1)
e

dt x, x 0.
t+1
ex + 1
e
x
Zdravko Starc, Vrsac, Serbia and Montenegro
XII.51. S
a se arate c
a sirul (an )n1 , an =

156

XII.54. S
a se afle functiile continue u = u (t), solutii ale ecuatiei
Z t
Z a
u (t) = +
b (s) u (s) ds +
b (s) u (s) ds, 0 t a,
0

unde este constant


a, iar b = b (t) este continu
a pe [0, a].
Adrian Corduneanu, Iasi
XII.55. S
a se arate c
a pentru orice n N , exist
a monoizi care nu sunt grupuri
si care contin exact n elemente inversabile.
Paul Georgescu si Gabriel Popa, Iasi

Probleme pentru preg


atirea concursurilor
A. Nivel gimnazial
G66. Se considera multimea A = {1, n + 1, 2n + 1, . . . , mn + 1}, m, n N ,
m > n. S
a se afle cte valori distincte poate lua suma a1 + a2 + + an , unde
a1 , a2 , . . . , an A.
Petru Asaftei, Iasi
G67. Fie b N, b 2. Spunem ca un numar natural este decompozabil daca se
poate scrie ca suma a dou
a numere cu aceesi sum
a a cifrelor n baza b. S
a se arate
c
a exist
a o infinitate de numere care nu sunt decompozabile.
Adrian Zahariuc, elev, Bac
au
G68. Fie N N ; s
a se arate c
a exist
a n N astfel nct factorialul niciunui
num
ar natural s
a nu se termine cu n, n + 1, . . . , n + N zerouri.
Iuliana Georgescu, Iasi
G69. Fie E (x) = ax2 + bx + c, a, b, c Q, x R. Dac
a a + b + c Z, ar
atati c
a
exist
a o infinitate de numere ntregi n astfel nct E (n) s
a fie num
ar ntreg.
Gheorghe Iurea, Iasi
G70. S
a se arate c
a ecuatia x2 + y 2 + 3x + y 707 = 0 nu are solutii n Q2 .
Dan Popescu, Suceava
a se demonstreze inegalitatea
G71. Fie (m, n) N2 \ {(0, 0)}. S
a
b
c
+
+

2
2
2
2
2
2
2
(m + n) a + mb + nc
(m + n) b + mc + na
(m + n) c + ma2 + nb2

1 1 1
1
+ +
, a, b, c (0, ) .

2 (m + n) a b
c
Titu Zvonaru, Com
anesti
G72. Fie 4ABC circumscris cercului de centru I. Cercul de diametru [AI]
b si C
b n M , respectiv N . Sa se arate ca M si
intersecteaza bisectoarele unghiurilor B
N se afl
a pe dreapta suport a liniei mijlocii paralele cu BC.
Doru Buzac, Iasi
G73. Fie ABCD un dreptunghi de centru O. Consider
am N (AO), M mijlocul
lui [AD], {P } = M N CD, {E} = OP BC. S
a se arate c
a N E BC.
Andrei Nedelcu, Iasi
157

G74. Fie n puncte n spatiu astfel nct oricare patru s


a formeze tetraedre de
volum cel mult 1. Sa se arate ca exista un tetraedru de volum cel mult 27 care sa
contina n interior toate cele n puncte.
Tudor Chiril
a, elev, Iasi
G75. Fie A1 A2 . . . An un poligon regulat de latur
a 1, n 4. Pe latura [A1 A2 ]
se considera punctul P1 cu P1 A1 = a (0, 1). Din punctul P1 se propaga o raza de
lumina care se reflecta de laturile [A2 A3 ], [A3 A4 ], . . . , genernd pe laturi punctele
de incidenta P2 , P3 , . . . (presupunnd c
a raza de lumin
niciodat
a ntr-un
a nu ajunge

2
vrf al poligonului) astfel nct m(A\
,
. S
a se afle valoarea
2 P1 P2 ) =
n n
minima a lui l pentru care Pl si Pl+1 nu apartin la doua laturi consecutive.
Irina Mustata
a, Iasi
, elev

B. Nivel liceal
L66. Fie ABC un triunghi, D si Da punctele n care cercurile nscris si Aexnscris sunt tangente la BC si Eb , Fc punctele n care cercurile B-exnscris si Cexnscris sunt tangente la AC si respectiv AB. Sa se arate ca punctele D, Da , Eb ,
b = 90 .
Fc sunt conciclice dac
a si numai dac
a AB = AC sau m(A)
Temistocle Brsan, Iasi

L67. Dreptele paralele t1 si t2 sunt tangente cercului C de centru O. Cercul C1


de centru O1 este tangent la t1 si C, iar cercul C2 de centru O2 este tangent la t2 , C
si C1 ; cele trei cercuri sunt exterioare unul celuilalt. Sa se arate ca unghiul O\
1 OO2
este ascutit si s
a se afle valoarea maxim
a a m
asurii acestuia.
Neculai Roman, Mircesti (Iasi)
L68. a) Pentru x, y, z (0, ), sa se demonstreze inegalitatea
s
r

r
1
1 1
x
y
(x + y + z)
+ +
1+
+
.
x y z
y
x
b) Folosind eventual a), s
a se arate c
a n orice triunghi, cu notatiile uzuale, are
loc inegalitatea
s
r
r
pa
pb
R
1+4 1+
+
.
r
pb
pa
Marian Tetiva, Brlad
L69. Pentru ce numere naturale n 3, exista n plan n puncte albastre si n
puncte rosii, oricare trei necoliniare, astfel nct n interiorul oric
arui triunghi cu
vrfurile albastre s
a existe cel putin un punct rosu, iar n interiorul oric
arui triunghi
cu vrfurile rosii s
a existe cel putin un punct albastru?
Adrian Zahariuc, elev, Bac
au
L70. Fie k, p N si un dreptunghi de dimensiuni 82k 2p, acoperit complet si
fara suprapuneri cu dreptunghiuri 7 5 si 6 4. Sa se arate ca numarul patratelelor
(x, y), x par, y impar, ale dreptunghiului mare, care sunt colturi n dreptunghiuri
75, este egal cu num
arul de dreptunghiuri 75. (Prin dreptunghiuri 75 ntelegem
dreptunghiuri cu lungimea egal
a cu 7 si l
atimea egal
a cu 5.)
Marius Pachitariu, elev, Iasi
158

L71. Fie n N, n 2 fixat. S


a se determine cea mai tare inegalitate de forma
n
n
q
X
X
a2k + n2 1 m
ak + M,
k=1

k=1

unde m, M nu depind de a1 , a2 , . . . , an , valabil


a pentru orice numere a1 , a2 , . . . ,
an pozitive si cu produsul 1.
Gabriel Dospinescu, Bucuresti
L72. Fie a, b numere rationale, pozitive, distincte, astfel nct an bn Z pentru
o infinitate de numere naturale n. Sa se arate ca a si b sunt ntregi.
Gabriel Dospinescu, Bucuresti

L73. Fie k N , k 3. S
a se determine n N \ {0, 1} pentru care
r
q

a1 + a2 + + ak n a1 a2 . . . ak , a1 , a2 , . . . , ak [0, ).

Gabriel Popa si Paul Georgescu, Iasi


a f : [a, b] R este continu
a si
L74. Fie n N , n 2 si a, b R, a < b. Dac
Rb k
x
f
(x)
dx
=
0
pentru
0

n,
atunci
f
are
cel
pu
t
in
n
+
1
zerouri
distincte
n
a
(a, b).
Andrei Nedelcu, Iasi
L75. Sa se determine n N pentru care este adevarata inegalitatea
i
1
cos < p
,

0,
.
8
2
1 + n sin4

C
at
alin Calistru, Iasi

Training problems for mathematical contests


Junior high school level
G66. Considering the set A = {1, n+1, 2n+1, . . . , mn+1}, m, n N , m > n, find
the number of distinct values taken by the sum a1+a2+. . .+an , when a1 , a2 , . . . , an A.
Petru Asaftei, Iasi
G67. Let b N, b 2. It is said that G N is decomposable if we can write G as
a sum of two numbers such that their expansions in the basis b have the same sum of
digits. Prove that there exist infinitely many numbers which are not decomposable.
Adrian Zahariuc, high school student, Bac
au

G68. Let N N . Prove that there is n N such that no factorial ends in n,


n + 1, . . . , n + N zeros.
Iuliana Georgescu, Iasi
2
G69. Let E (x) = ax + bx + c, a, b, c Q, x R. If a + b + c is an integer, prove
that there exist infinitely many integers n such that E (n) is also an integer.
Gheorghe Iurea, Iasi
G70. Prove that the equation x2 + y 2 + 3x + y 707 = 0 has no solutions in Q2 .
Dan Popescu, Iasi
159

G71. Let (m, n) N2 \ {(0, 0)}. Prove that


(m +

n) a2

b
c
a
+
+

2
2
2
2
2
2
+ mb + nc
(m + n) b + mc + na
(m + n) c + ma2 + nb2

1 1 1
1
+ +
, a, b, c (0, ) .

2 (m + n) a b
c
Titu Zvonaru, Com
anesti

G72. Let I be the incenter of a triangle ABC. The circle with diameter [AI]
b and C
b in M , respectively N . Prove that M and N lie on
meets the bisectors of B
the line joining the midpoints of [AB] and [AC].
Doru Buzac, Iasi
G73. Let ABCD be a rectangle with center O. Let N (AO), let M be the
midpoint of [AD] and let {P } = M N CD, {E} = OP BC. Prove that N E BC.
Andrei Nedelcu, Iasi
G74. Let us consider n points such that any given four are the vertices of a
tethraedron with volume at most 1. Prove that there is a tethraedron with volume
at most 27 which contains all n points in its interior.
Tudor Chiril
a, high school student, Iasi
G75. Let A1 A2 . . . An be a regular n-gon with side 1, n 4. We consider P1
on the side [A1 A2 ] such that P1 A1 = a (0, 1). A ray of light is emitted from P1
towards and is reflected by the sides [A2 A3 ], [A3 A4 ], . . . , generating the incidence
points P2 , . . . , Pn (supposingthat the
ray never meets the vertices A1 , A2 , . . . , An )
2
\
such that m(A2 P1 P2 ) =
,
. Find the minimal value of l such that Pl and
n n
Pl+1 do not belong to adjacent sides.
Irina Mustata
, high school student, Iasi

High school level


L66. Let ABC be a given triangle and let D, Da be the points in which the
incircle, respectively the A-escribed circle are tangent to the side BC. Let also Eb ,
Fc be the points in which the B-escribed and C-escribed circles are tangent to the
side AC, respectively to the side AB. Prove that D, Da , Eb , Fc are concyclic if and
b = 90 .
only if AB = AC or m(A)
Temistocle Brsan, Iasi

L67. The parallel lines t1 and t2 are tangent to the circle C with center O, the
circle C1 with center O1 is tangent to t1 and C and the circle C2 with center O2 is
tangent to t2 , C and C1 ; C, C1 and C2 being exterior to each other. Prove that the
angle O\
1 OO2 is acute and find the minimum value of its measure.
Neculai Roman, Mircesti (Iasi)
L68. a) Prove that, for x, y, z (0, ),
s
r

r
x
y
1
1 1
(x + y + z)
+ +
1+
+
.
x y z
y
x
160

b) Using a), prove that


r

R
1+
1+4
r

pa
+
pb

for any given triangle with the usual notations.

pb
pa

Marian Tetiva, Brlad


L69. Find n N, n 3, such that there are n blue points and n red points in
the same plane, no three points being collinear, such that the interior of any triangle
with blue vertices contains at least one red point and the interior of any triangle with
red vertices contains at least one blue point.
Adrian Zahariuc, high school student, Bac
au
L70. Let k, p N and let a 82k 2p rectangle which is completely covered with
7 5 and 6 4 rectangles with no superpositions. Prove that the number of squares
(x, y) with side 1, x even, y odd, which are vertices of 7 5 rectangles equals the
total number of 7 5 rectangles. (By a 7 5 rectangle we mean a rectangle with
length 7 and height 5).
Marius Pachitariu, high school student, Iasi
L71. Let n N, n 2. Find the best constants m, M such that
n q
n
X
X
a2k + n2 1 m
ak + M
k=1

k=1

for any a1 , a2 , . . . , an > 0 satisfying a1 a2 . . . an = 1.


Gabriel Dospinescu, Bucuresti
n
L72. Let a, b Q, a, b > 0, a 6= b such that a bn Z for infinitely many n N.
Prove that a, b Z.
Gabriel Dospinescu, Bucuresti

L73. Let k N , k 3. Find n N \ {0, 1} such that


r
q

a1 + a2 + + ak n a1 a2 . . . ak , a1 , a2 , . . . , ak [0, ).
Gabriel Popa and Paul Georgescu, Iasi
L74. Let n N , n 2 and a, b R, a < b. If f : [a, b] R is a continuous
Rb
function such that a xk f (x) dx = 0 for any k N, 0 k n, then f has at least
n + 1 distinct zeros in (a, b).
Andrei Nedelcu, Iasi
L75. Find n N such that
i
1
cos < p
,

0,
.
8
2
1 + n sin4
C
at
alin Calistru, Iasi

161

Pagina rezolvitorilor
BOTO
SANI
Scoala
nr. 7 "O. Bancila". Clasa a IV-a. IFTODE Cozmin: P(54-59,61,63,64,

67,68,70).
Colegiul National "A. T. Laurian". Clasa a IX-a. NEGRESCU Alexandru:
VII(43,47,48), VIII(47,49), IX(42,46,48), X.42, G46.
CRAIOVA
Angela). STANCIU
Scoala
nr. 22 "M. Eliade". Clasa a IV-a (inst. VANTU

Ioan: P (64-73)
IA
SI
Scoala
nr. 3 "Al. Vlahuta". Clasa a V-a. COJOCARU Ioana: P.71, V(47
49),VI.46; DODU Corina: P.71, V(47-49),VI.46; IRIMIA Andreea: P.71, V(4749),VI.46; S
TIRBAN Ioana: P.71, V(47-49),VI.46; UNGURU George Claudiu: P.71,
V(47-49),VI.46.
Scoala
nr. 7 "N. Tonitza". Clasa a II-a (nv. TUDOSE Elena). DOBRIN Diana
Simona - AlexanMaria: P(64-68,70,72); LEONTE Anca: P(64-68,70,72); POSTICA
dra: P(64-68,70,72); ROTARU Larisa-Maria: P(64-68,70,72); SAVIN Razvan: P(6468,70,72). Clasa a II-a (nv. MELINTE Rodica). BACIU Ciprian: P(64-69,71,72);
BRZU Constantin: P(64-69,71,72); BOTOSANU Bianca-Mihaela: P(64-69,71,72);
D
BUZDUGAN Petru-C
at
alin: P(64-69,71,72); CEUCA
anut-Vasilic
a: P(64-69,71,
72); CONSTANTINESCU Diana-Gabriela: P(64-69,71,72); CUCUTEANU PaulC
at
alin: P(64-69,71,72); GUSOVATE Diana-Stefana: P(64-69,71,72); LEOGAN
Larisa-Diana: P(64-69,71,72); MIRON Vlad-Stefan: P(64-69,71,72); MOTAN Geanina-Diana: P(64-69,71,72); ROTARIU Marian: P(64-69,71,72); SUCIUC Raluca:
P(64-69,71,72); TEIU-COSTIN Andra-Mihaela: P(64-69,71,72). Clasa a III-a
AROIU

(nv. PASANIUC Maria). ATASIEI Vl


adut: P(64,65,67,68,72); PAS
Bogdan:
P(66-68,70,71); SOLOMON Ana-Maria: (65,68-71); TINCU Andrei: P(64,65,68
70,72); ZAMFIR Loredana-Cristiana: P(64,65,69-71). Clasa a III-a (nv. GEAMAN
ALI
TEI

P(65,68-71); GHIARASIM Olivia: P(64,65,67,


Gabriela). ADASC
IONUT:
68,72); MACOVEI Alina: P(64,65,68-70,72); NEGRESCU Vlad-Petru: P(64,65,6871); TROCIN Monica-Andreea: P(66,68-71).
Scoala
nr. 13 "Alexandru cel Bun". Clasa a IV-a (inst. COJOCARIU Ana). CO
C
A
BILIT
at
alina-Elena: P(64-68,70); CURMEI Renata-Maria: P(64-68,70); DALAS
Radu: P(64-68,70); ILIE Laura: P(64-68,70); MAGDICI Magda-Otilia: P(64-68,70);
MARDARI Claudiu: P(64-68,70); LUCAN Mihaela-Alexandra; P(64-68,70); NECHIFOR S
tefan-Marian: P(64-68,70); NICULAE-GRIGORESCU Andrei: P(64-68,70);
PRISACARU Carmen-Georgiana: P(64-68,70); PETREA Silvia: P(64-68,70); S
TE
FAN Teodora-Ioana: P(64-68,70); TRLAGEANU
Ingrid-Maria: P(64-68,70).
Scoala
nr. 17 "I. Creanga". Clasa a VIII-a. DUMITRIU Vlad: VI(46-48),VII.48,

VIII.46.

CoScoala
nr. 22 "B. P. Hasdeu". Clasa I (nv. S
TEFAN Liviu). DANIL
A

drin P(55,64,65,67,71); IGNAT Andreea: P(54-56,64,65); NICOLA Delia-Corina:


P(55,64,65,67,71); PURICE Dumitru-Ciprian: P(64-67,71). Clasa a II-a (nv. DO162

HOTARU Liliana). TURCU Andrei-Daniel: P(64-67,71); Clasa a II-a (nv. TRZI ALI
TEI

ORU Iuliana). ADASC


Victor: P(64-69,71,72); APOSTOL Ana-Maria:
Catalina: P(64-68,72);
P(64-68,71,72); BALAN Andrei: P(64-68,71,72); BURUIANA
CUBERSCHI Paul: P(64-67,70-72); ESANU Georgiana: P(64-68,71); GREIEROSU
ATIC

Claudiu: P(64-68,72); GNDU Alexandra-Livia: P(64-66,68,71,72); LAM


Ioana: P(61,64-68,71); MOGA Alexandru: P(64-68,71,72); REBEGEA Andrada: P(6468,71); UNGUREANU Teofana: P(64-68,71,72). Clasa a II-a (nv. TUTU Laura).
AILENEI-OPREA Adriana: P(64-68); ANDRONICIUC Ana-Miruna: P(64-68); BR
LADEANU
Claus-Alex: P(64-66,68,71); BOARU Adrian: P(64-66,68,71); BURUIA Sebastian-Andrei: P(64-68,71): BUHU Vlad: P(64-68,71); CEOBANU AndreiNA
Alexandra-Elena: P(64-68,71); COSTACHESCU

Nicolae: P(64-67,71); CHICHIRAU

Alexandru: P(64-66,68,71); DIACONESCU Matei:


Ivona: P(64-68,71); DANIL
A
P(64-68,71); DOROHOI Ovidiu: P(64-68,71); GHERAN Ana-Maria: P(64-66,68,71);
Raluca-Claudia: P(64-68,71); HATESGRIGORE Georgiana: P(64-68,71); GURAU

CU Iustina: P(64-68,71); HORBOVANU Bianca-Alexandra: P(64-68,71); NASTASE


Andrei-Ionut: P(64-68,71); ONOFREI Liviana Ana-Maria: P(64-68,71); RADU
Andrei: P(64-66,68,71); SIMIRAD Andrei: P(54,55,57-59,61,62,64-72); SMEREA
Alexandra-Arina: P(64-66,68,71). Clasa a V-a. PINTILIE Mina-Liviu: P(61,62,7173); PINTILIE Nicoleta-Livia: P(61,62,71-73); S
TERBULEAC Daniel: P(61-63,7173),V(42,43).
Beatrice). TUDOScoala
nr. 23 "T. Maiorescu". Clasa a IV-a (nv. CHIRILA

RACHE Alexandru-Gabriel: P(64-73).


Scoala
nr. 26 "G. Cosbuc". Clasa a II-a (nv. BUCATARIU Rica). IACOB

Robert-Ionut: P(54-57,64-67); IVANCIUC Dumitru-Florin: P(55,64-67); MOISA


Adrian-Bogdan: P(54-57,64-67); SANDU Ioana-Luiza: P(54-57,64-67); SCUTARU
Ionela-Cristina: P(54-57,64-67). Clasa a III-a (nv. RACU Maria). BULGARU

Ionela-Alexandra: P(64-68,70-72); BURLACU S


tefan-Claudiu: P(64-68,70,72); CA

LIN Andreea-Georgiana: P(64-68,70-72); IFROSA Adriana: P(64,66-72); IOJA Petru-Alexandru: P(64-70,72); MOISA Bogdan: P(64-68,70-72); PINTILIE R
azvan
Florin: P(64-68,70-72); RAZLOG
Ionut: P(64-68,70-72). Clasa a III-a (nv. GALIA
Oana-Catalina:
Paraschiva). ALUPEI Andra-Madalina: P(64-67,68,70,71); CIOABA

P(64-68,70,71); GHERCA Marius-Catalin: P(64-68,70,71); HOMEA Liviu: P(6468,70,71); HUIDES Gina: P(64-68,70,71); MANOLIU M
ad
alina: P(64-68,70,71);

Alexandru: P(64-68,70,71); POPA


MIHAILESCU
Laura; P(64-68,70,71); PISICA
Florin: P(64-68,70,71); SCUTARU Constantin: P(64-68,70,71,73).
Colegiul National "C. Negruzzi". Clasa a V-a. ANDRIESCU Gabriela: P(56,58,
59,61,63),V.42; TIBA Marius: P(71-73),V(46,47,49,50),VI.46.
Colegiul National Iasi. Clasa a V-a. VLCU Maria Caterina: V(40,41,43-45).
Liceul "M. Eminescu". Clasa a VI-a. CIURARU Ionela: V(46,48), VI(47,48,50);
IPATE Cristina: V(46,48),VI(47,48,50). Clasa a IX-a. AVRAM Mircea: VII(41,43,
44,46,48). Clasa a X-a. COMAN S
tefan: VIII(46,49), IX(46,47), X.47.

163

Premii acordate rezolvitorilor


Pentru aparitia de trei ori la rubrica "Pagina rezolvitorilor" redactia revistei
"Recreatii matematice" acorda o diplom
a si un premiu n carti elevilor urmatori:
Scoala
nr. 22 "M. Eliade", Craiova

STANCIU Ioan (cl. a IV-a): 2/2003 (9pb), 1/2004 (10pb), 2/2004 (10pb).
Scoala
nr. 7 "N. Tonitza"

BACIU Ciprian (cl. a II-a): 2/2003 (5pb), 1/2004 (5pb), 2/2004 (8pb);
BRZU Constantin (cl. a II-a): 2/2003 (5pb), 1/2004 (5pb), 2/2004 (7pb);
BOTO
SANU Bianca-Mihaela (cl. a II-a): 2/2003 (5pb), 1/2004 (5pb), 2/2004
(7pb);
BUZDUGAN Petru-C
at
alin (cl. a II-a): 2/2003 (5pb), 1/2004 (5pb), 2/2004
(7pb);
D
CEUCA
anut-Vasilic
a (cl. a II-a): 2/2003 (5pb), 1/2004 (5pb), 2/2004 (8pb)
CONSTANTINESCU Diana-Gabriela (cl. a II-a): 2/2003 (5pb), 1/2004 (5pb),
2/2004 (8pb);
CUCUTEANU Paul-C
at
alin (cl. a II-a): 2/2003 (5pb), 1/2004 (5pb), 2/2004
(8pb);
DOBRIN Diana-Maria (cl. a II-a): 2/2003 (5pb), 1/2004 (5pb), 2/2004 (7pb);
GU
SOVATE Diana-
Stefana (cl. a II-a): 2/2003 (5pb), 1/2004 (5pb), 2/2004
(8pb);
LEOGAN Larisa-Diana (cl. a II-a): 2/2003 (5pb), 1/2004 (5pb), 2/2004 (8pb);
LEONTE Anca (cl. a II-a): 2/2003 (5pb), 1/2004 (5pb), 2/2004 (7pb);
MIRON Vlad-
Stefan (cl. a II-a): 2/2003 (5pb), 1/2004 (5pb), 2/2004 (8pb);
MOTAN Geanina-Diana (cl. a II-a): 2/2003 (5pb), 1/2004 (5pb), 2/2004 (8pb);
Simona-Alexandra (cl. a II-a): 2/2003 (5pb), 1/2004 (5pb), 2/2004
POSTICA
(7pb);
ROTARIU Larisa-Maria (cl. a II-a): 2/2003 (5pb), 1/2004 (5pb), 2/2004 (7pb);
ROTARIU Marian (cl. a II-a): 2/2003 (5pb), 1/2004 (5pb), 2/2004 (8pb);
SUCIUC Raluca (cl. a II-a): 2/2003 (5pb), 1/2004 (5pb), 2/2004 (8pb);
TEIU-COSTIN Andrada-Mihaela (cl. a II-a): 2/2003 (5pb), 1/2004 (5pb),
2/2004 (8pb).
Scoala
nr. 22 "B. P. Hasdeu"

TEI

ADASC
ALI
Victor (cl. a II-a): 2/2003 (5pb), 1/2004, (5pb), 2/2004 (9pb);
BALAN Andrei (cl. a II-a): 2/2003 (5pb), 1/2004 (5pb), 2/2004 (8pb);
BUHU Vlad (cl. a II-a): 2/2003 (5pb), 1/2004 (5pb), 2/2004 (6pb);
Alexandra-Elena (cl. a II-a): 2/2003 (5pb), 1/2004 (6pb), 2/2004
CHICHIRAU
(6pb);
CUBERSCHI Paul (cl. a II-a): 2/2003 (5pb), 1/2004 (6pb), 2/2004 (7pb);
E
SANU Georgiana (cl. a II-a): 2/2003 (5pb), 1/2004 (5pb), 2/2004 (5pb);
GREIEROSU Claudiu (cl. a II-a): 2/2003 (5pb), 1/2004 (6pb), 2/2004 (6pb);
Raluca-Claudia (cl. a II-a): 2/2003 (5pb), 1/2004 (5pb), 2/2004 (6pb);
GURAU
164


NASTASE
Andrei-Ionut (cl. a II-a): 2/2003 (5pb), 1/2004 (6pb), 2/2004 (6pb);
HATESCU Iustina (cl. a II-a): 2/2003 (5pb), 1/2004 (5pb), 2/2004 (6pb);
ATIC

LAM
Ioana (cl. a II-a): 2/2003 (5pb), 1/2004 (6pb), 2/2004 (7pb);
REBEGEA Andrada (cl. a II-a): 2/2003 (5pb), 1/2004 (5pb), 2/2004 (6pb).
Scoala
nr. 23 "Titu Maiorescu"

TUDORACHE Alexandru-Gabriel (cl. a IV-a): 2/2003 (10pb), 1/2004 (10pb),


2/2004 (10pb).
Scoala
nr. 26 "G. Cosbuc"

ALUPEI Andra-M
ad
alina (cl. a III-a): 2/2003 (5pb), 1/2004 (5pb), 2/2004 (7pb);
BULGARU Ionela-Alexandra (cl. a III-a): 2/2003 (5pb), 1/2004 (5pb), 2/2004
(8 pb);
BURLACU S
tefan-Claudiu (cl. a III-a): 1/2003 (5pb), 1/2004 (5pb), 2/2004 (8pb);
Marius-C
GHERCA
at
alin (cl. a III-a): 2/2003 (5pb), 1/2004 (5pb), 2/2004 (7pb);
HOMEA Liviu (cl. a III-a): 2/2003 (5pb), 1/2004 (5pb), 2/2004 (7pb);
HUIDE
S Gina (cl. a III-a): 2/2003 (5pb), 1/2004 (5pb), 2/2004 (7pb);
Adriana (cl. a III-a): 2/2003 (5pb), 1/2004 (5pb), 2/2004 (8pb);
IFROSA
Petru-Alexandru (cl. a III-a): 2/2003 (5pb), 1/2004 (5pb), 2/2004 (8pb);
IOJA
Alexandru (cl. a III-a): 2/2003 (5pb), 1/2004 (5pb), 2/2004 (7pb).
PISICA
Colegiul National "C. Negruzzi"
TIBA Marius (cl. V-a): 1/2003 (6pb), 2/2003 (7pb), 1/2004 (7pb).
Liceul "M. Eminescu"
CIURARU Ionela (cl. VI-a): 2/2003 (11pb), 1/2004 (5pb), 2/2004 (5pb).

(continuarea tabelului din p. 128)


178.
179.
180.
181.
182.
183.

PIFTOR Rositta
SPIRIDON Doina
MARTINUSI Vladimir
TIMOHE TUMAC Gabriel
ANTON Florina Cristiana
PRICOP Vasile

Liceul economic nr. 2, Iasi


Liceul de arta, Iasi
Colegiul National "E. Racovita", Iasi
Liceul "G. Ibraaileanu", Iasi
Colegiul National "E. Racovita", Iasi
Pascani

165

Premiile pe anul 2004 acordate


"POIANA"
de FUNDA
TIA CULTURALA
Fundatia Cultural
a "Poiana" (director d-l Dan Tiba) acord
a anual premii
elevilor - colaboratori ai revistei "Recreatii matematice" care se disting prin calitatea
articolelor, notelor si problemelor originale publicate n paginile acesteia.
Redactia revistei decide ca pentru anul 2004 premiile oferite, n valoare de cte
1 000 000 lei, s
a fie atribuite urm
atorilor elevi:
1. LUPU Cezar (Colegiul National "Mircea cel Batrn", Constanta)
Asupra unei inegalit
ati conditionate (RecMat 1/2004, 27-28),
probleme propuse: IX.36 (1/2003), IX.44 (2/2003), IX.48, XI.46 (1/2004),
X.51 (2/2004).
2. NEGRESCU Alexandru (Colegiul National "A. T. Laurian", Botosani )
Asupra unei inegalitati (RecMat 2/2004, 106-108),
probleme propuse: VI.38 (1/2003), VII.41 (2/2003), VIII.46 (1/2004),
VII.51 (2/2204).
Premiile se pot ridica direct de la redactie sau pot fi trimise prin mandat postal
la adresa elevului premiat.

IMPORTANT
n scopul unei leg
aturi rapide cu redactia revistei, pot fi utilizate urm
atoarele adrese e-mail: tbi@math.tuiasi.ro, popagabriel@go.com .
Pe aceast
a cale colaboratorii pot purta cu redactia un dialog privitor la
materialele trimise acesteia, procurarea numerelor revistei etc.
La problemele de tip L se primesc solutii de la orice iubitor de matematici
elementare (indiferent de preocupare profesionala sau vrsta ). Fiecare
dintre solutiile acestor probleme - ce sunt publicate n revist
a dup
a un
an - va fi urmat
a de numele tuturor celor care au rezolvat-o.
Adres
am cu insistenta
amintea ca materialele trimise re rug
vistei s
a nu fie (s
a nu fi fost) trimise si altor publicatii.

166

Revista RECREAII MATEMATICE apare de dou ori pe an (la


datele de 1 martie i 1 septembrie) i se adreseaz elevilor, profesorilor,
studenilor i tuturor celor pasionai de matematicile elementare.
n atenia tuturor colaboratorilor
Materialele trimise redaciei spre publicare (note i articole, chestiuni de
metodic, probleme propuse etc.) trebuie prezentate ngrijit, clar i concis; ele
trebuie s prezinte interes pentru un cerc ct mai larg de cititori. Se recomand
ca textele s nu depeasc patru pagini. Evident, ele trebuie s fie originale
i s nu fi aprut sau s fi fost trimise spre publicare altor reviste.
Problemele originale destinate rubricii Probleme propuse vor fi
redactate pe foi separate cte una pe fiecare foaie, cu enun i
demonstraie/rezolvare, fiind nsoite de numele autorului, coala i localitatea
unde lucreaz/nva.
Redacia revistei va decide asupra oportunitii publicrii materialelor
primite.
n atenia elevilor
Numele elevilor care vor trimite redaciei soluii corecte la exerciiile i
problemele din rubrica Probleme propuse vor fi menionate n Pagina
rezolvitorilor. Elevii vor ine seama de urmtoarele reguli:
1. Pot trimite soluii la minimum cinci probleme propuse n numrul
prezent i cel anterior al revistei; pe o foaie va fi redactat soluia unei
singure probleme.
2. Elevii din clasele VI-XII au dreptul s trimit soluii la problemele
propuse pentru clasa lor, pentru orice clas mai mare, din dou clase mai mici i
imediat anterioare. Elevii din clasa a V-a pot trimite soluii la problemele
propuse pentru clasele a IV-a, a V-a i orice clas mai mare, iar elevii claselor
I-IV pot trimite soluii la problemele propuse pentru oricare din clasele primare
i orice clas mai mare. Orice elev poate trimite soluii la problemele de
concurs (de tip G i L).
3. Vor fi menionate urmtoarele date personale: numele i prenumele,
clasa, coala i localitatea.
4. Plicul cu probleme rezolvate se va trimite prin pot (sau va fi adus
direct) la adresa Redaciei:
Prof. dr. Temistocle Brsan
Catedra de Matematic
Universitatea Tehnic Gh. Asachi Iai
Bulevardul Carol I nr. 11, 700506, Iai
E-mail: tbi@math.tuiasi.ro

CUPRINS
Ctre cititori .................................................................................................................. 85
ALEXANDRU MYLLER, ctitorul colii matematice ieene.................................. 87
HENRI POINCAR la 150 de ani de la naterea sa ........................................ 89
Trecerea planetei Venus prin faa Soarelui ............................................................... 91

ARTICOLE I NOTE
G. DOSPINESCU Cteva noi aplicaii ale unei idei consacrate ........................ 94
T. BRSAN Cteva proprieti ale medianelor....................................................... 99
C. - t. POPA O construcie geometric a mediilor (II)..................................... 102
F. POPOVICI O generalizare a teoremelor de baz ale calculului diferenial ........ 104

NOTA ELEVULUI
A. NEGRESCU Asupra unei inegaliti................................................................ 106
***
Asupra problemei VII.41 din RecMat 2/2003 ....................... 109

CHESTIUNI METODICE
D. MIHALACHE, M. TETIVA Asupra unei probleme de concurs.................... 111

CHESTIUNI COMPLEMENTARE MANUALELOR


M. CRCIUN Exponentul numrului natural a n produsul n!......................... 114

CONCURSURI I EXAMENE
Concursul
Concursul
Concursul
Concursul
Olimpiada

Alexandru Myller ed. a II-a, Iai, 2004 ............................................. 116


Florica T. Cmpan, ed. a IV-a, 2004 ................................................. 121
Traian Lalescu, ed. a V-a, Iai, 2004 ................................................ 123
Adolf Haimovici, ed. a VIII-a, 2004 .................................................. 124
Balcanic de Matematic (juniori), ed. a VIII-a, 2004 ....................... 126

CORESPONDENE
H. STEPHAN Probleme pentru clasa a VIII-a ..................................................... 129

PROBLEME I SOLUII
Soluiile problemelor propuse n nr. 2/2003............................................................. 130
Soluiile problemelor pentru pregtirea concursurilor din nr. 2/2003 .................... 144
Probleme propuse........................................................................................................ 152
Probleme pentru pregtirea concursurilor ................................................................. 157
Training problems for mathematical contests .......................................................... 159
Pagina rezolvitorilor.................................................................................................... 162

S-ar putea să vă placă și